Text
                    THE RED BOOK
OF MATHEMATICAL
PROBLEMS
KENNETH S. "VIILIAMS
KENNETH HARDY
Carleton University, Ottawa
Dover Publications, Inc.
Mineola, New York


PREFACE TO THE FIRST EDITION Copyright Copyright @ 1988 by Integer Press. , All rights re.served under Pan AmeriC'dn and International Copy- nght Gmventlons. Published in Canad.a by Genual Publishing Company, LId.. 3f! Lesmlll Road, Don :\fIlh, Toronto. Ontario. II has become the fashion for some authors to include li.eraryquotations in Ihf'ilmathematical texts, presumably with the aim of connecting mathematics illl<I lilt' humanities. The preface of The Green Book*' of 100 practice problems ,." undergraduate mathematics competitions hinted at connections between jllllhh-IIJ-solving and all the traditional elements of a fairy tale: mystery, "';11.-11. discovery, and finally resolution. Althouh The Red Book may s('em 10 1.,1'" polIucal overtones, rest assured, de3r reader, that the quot:Jtions (labelled IVI,.. x, Pushkin and Trotsky, just for fun) are merely an in'ipiration for your I' ""IH'V through the enchantl:tl realms of mathematics. Bibliographical Note This. Do."er edition, first publishr.d in 1996, is a slightly corrcl'led republIcauon of he work originally published hy Integer Press, Ottawa, Canada, In 1988 under the title The Red Book: 100 Practice Problems for Undergraduate Mathematics Competitions. A section of the original page 97 has been deleted and all subsequent copy repaged thereafter. J'ht! Red Book contains 100 problems for undergraduate swdents training fill mathematics competitions, particularly the William Lowell Putnam Mathematical Competition. Along with the problems come useful hints. and . tllllpicte solutions. The book will also be useful to anyone interested in the posing and solving of mathematic.al problems at the undergraduate level. Many of the problems were suggested by ideas originating in a variety of ources, including Crux Mathematicorum, Mathematics Magazine and the American Mathematical Monthly, as well as various mathemaLic;. competi. tiolls. Where possible, acknowledgement to known sources is given at the end of t he book. Libraty of Congmss Cataloging-in-Pu.blication Data Williams, Kenneth S, The red book of mathematical problems! Kenneth S. Williams, Kenneth Hardy. p, em. . '1\ slightly corncted I'epublication of te work originally pub- hshed b}' Imeger Press, Ouawa, Canada, 111 1988 under the title: The. red book: 100 practice problems for undergraduat( mathe- matIc;. competitions"-Tp. verso. Includes bibliographical references. ISBN 0-486-69413-1 (pbk.) l. Mathemalics-Problems, exercises, elc. I. Ilardy, Kenneth. n. Title. QA43.W55 1996 510'. 76-dc20 96-43820 CIP Once again, we would be interested in your reaction to The Red Book, and illvitecomments, alternate solutions, and even corrections. We make no claim thill the solutions are the "best possible" solut.ions, but we trust that. you will find them elegant enough, and that The Red Book will be a practical tool in training undergraduate competitors. We wish to thank our typeseuer and our literary adviser at Integer Press for their valuable assistanl"C in this project. Kmneth S. Williams and Kenneth Hardy Ottawa, Canada May, 1988 -To be reprinted by Dover Publications in 1997. Manufactured in the United States of America Dover Publications, Inc., 31 East 2nd Street, Mineola, KY. 11501 
CONTENTS I-'age Notation The Problems The Hints The Solutions The Sources ]X  f :"{ 1 '7" 
[x] lnx exp x q,(n) GCD(a,b) () () deg U(x» Y.. t T(n) 1'( x) dct A z Q,R,C NOTATION denotes the greatest integer  $, where $ is a real number. denotes the natural logarithm of x. denotes the exponential function e"'. denotes Euler's totient function defined for any naLllra.1 num- bcr n. denotes the greatest common divisor of the integers a a,nd b. denotes the binomial coefficient n!fk! (n - k)!, where nand k are non-negative integers (the symbol having value zero when n < k) . denotes Legendre's symbol which has value +1 (resp. -1) if the integer n is a quadratic residue (resp. nonresidue) modulo the odd prime p . denotes the degree of the polynomial f( x) . denotp.s the transpose of the row vector Y.. . denotes the number of distinct prime divisors of the positive integer n. denotes the derivative of the function f( x) with respect to x. denotes the determinant of the square matrix A. denotes the domain of rational integers. denote the fields of rational, real, complex numbers respec- tively. 
THE PROBLEMS Mankind always sets itself only sudt problems t1. ii, mn _-ob.'{,; .,. it will always bt: found lhal tlu lask ilsdf fI1'iM'S only wlu 11 III.( l1W,- terial conditions for its solution already exist 01' are at least in the proc.ess of formation. Earl Marx (1818-1883) 1. Let p denote an odd prime and set w =: exp(27ri/p). Evaluate the product (1.0) E(p) (W T1 + w T ' + . . . + d(P-l)!. )(w nt + w n . + . . . + W"{P-l)!2), where 1'1,.. . ,r(p-I)/2 denote the (p - 1 )/2 quadratic residues modulo p and n},.. ., n(p-l)/2 denote the (p 1)/2 quadratic nOIlfesidues modulo p. 2. Let k denote a positive integer. Determine the number N(k) of triples (x,y,z) of integers satisfying { Ixl s: k, Iyl s: k, Izi s: k, Ix yl s: k, Iy zl s: k, Iz xl s: k. (2.0) 3. Let p 1 (mod 4) be prime. It is known that there exists a unique integer w == w(p) such that w 2 == -1 (mod p), 0 < w < p/2. 
2 PROBLEMS (For example, -w(5) 2,w(13) = 5.) Prove that there exist integers a,b,c,d with ad - be 1 such that pX 2 + 2wXY + (w 2 + 1) 1'2 == (aX + by)2 + (eX + dY)2. P (For example, when 1) =- 5 we have 5.\'2+ 4 xy+y2 ,y21-(2,y+n 2 , and wlwn I' ,= I: WI' hay" 13X 2 + lOX}' + 2y2 (3)( + Y)2 + (2X + Y)2.) 4. Let d,,(n), T = 0,1,2,3, denote the number of positive integral divisors of n which are of the form 4k + T. Let m denote a positive integer. Prove that (4.0) f(d1(n) - ds(n» = f( -lY [ 2 ': 1 ] . n=l 3=0 J 5. Prove that the equation (5.0) y2 = x 3 + 23 has no solutions in integers x and y. 6. Let f(x, y) -= a.x 2 +2bxy+cy2 be a positive-definite quadratidorm. Prove that (flxl,ydf(x2,Y2))l/2f(xl - X2,Yl - Y2) (6.0)  (ac- b 2 )(XI1/'l X2Yl?, PROBLEMS 3 for all real numbers Xl,X2,YI,Y2. 7. Let R, S, T be three real numbers, not all the same. Give a condi- tion whkh is satisfied by one and only one of the three triples (7.0) { (R,S,1'), (1',-8 + 21',R .') +1'), (R 5' + T,2R 8,R). 8. Let ax 2 + bxy + cy2 and Ax2 + Bxy + Cy2 be two positive-definite quadratic forms, which are not proportional. Prove that the form (8.0) (aB-bA)x 2 +2(aC cA)xy+(bC cB)y2 is indefinite. 9. Evaluate the limit I (9.0) L . n n 2 k lim - ,,- . n_OO 2 n L..i k k=l I Prove that there does not exist a constant c  1 such that 10. (10.0) n C 4>(n)  mct/>(m), for all positive integers nand m satisfying n  m. I 11. Let D be a squarefree integer greater than 1 for which there exist positive integers AI,A2,Bl,B2 such that { D= Ai+Bi =A+B, (11.0) (AhBd :f:. (A 2 ,B 2 ). 
4 PROBU'}MS PROBLEMS !) Prove that neither 2D(D + il]A 2 + B]B 2 ) nor (15.0) 2D(D + A]ft 2 RIBz) is the sqllare of an integ;er. 12. [",t Q and R deno!." the fields of rational and re<!l numhers rl?f>pcctivf'ly, Lf't K ;md L 1)(' 1.11f' smallf'st SII bfif'lds of n. which contain both Q a.nd the real numbers (16.0) J lf)85 + 31 Y1985 and J 3970+ 64V1985 , respectively. Prove that K L. 13. Let k a.nd 1 be positive integers such that 15. Evaluate the integral I follnXlIl(1 x)dx. 16. Solve the nXUfn'nce r(']ation E (;)a(k) 'II 11 + 1 ' n 1,2,. '. 17. Let n alld k be positive integers. Let p be a prime such tha.t p> (n2+n+k?+k. GCD(k,5) GCD(l,5) GCD(k,l) 1 (17.0) Prove that the sequence n 2 , n 2 + 1, n 2 + 2, . . . , n 2 + 1 , and where 1 that _k 2 + 3kl- t p2 , where GCD(}5) 1. Prove that tllt pair of equations (13.0) { k x 2 + y2, 1 = x 2 + 2xy + 2y2, has exa.ctly two solutions in integers x and y. (n2+n+k)2 n 2 + k, conta.ins a pair of integers lm., m. + k) such ( : ) ( m; k) = 1. 18. Let 14. Let rand B be non-zero integers. Prove that the equation Docs the infinite series 2::;:'=0 an converge, and if so, what is its sum? an 1 1 -+- 4n + 1 4n + 3 n=O,l,... . (14.0) (r 2 _ s2)x 2 4rsxy (r 2 _ B2)y2 1 has no solutions in integers x a,nd y. 1 2n+2 ' 19. Let a],..., am be m ( 2) real numbers. Set An 0,1 + 0,2 + . . . + an, n = 1,2,.., , m. . 
6 PROBLE:M.8 PROBLEMS 7 Prove that Let P be a real-valued function of the n(n - 1)/2 variables Xij such that the inequality (19.0) f ( n r  12fa;. n=2 n=1 (no) n F(:CIl,:f12,..' ,:1'" 111)  2::x% 1.:=1 s -. 'n () - L-. ( 2n-l ) k=O k holds for all Xl,"" Xn. Prove lItal. equality eaIJllot hold ill (23.0) jf 7::k=1 :1:).. f O. 20. Evaluate the sum for all positive integers n. 24. Let. Uh''',U", he 11! (;2: 1) real 11111111)('1'" whidt an' "uelr that 2':::.;'=1 an f O. Prove tILe inequality 21. Let a and b be coprime positive integers. For k a positive integer, let N(k) denote the number of integral solutions to t,he equation (24.0) ( t na. ) / ( t 0", ) 2 > 2:m 1't_1 'n_l (21.0) ax + by ;;::; k, x 2': 0, y 2': 0 . Evaluate the limit L = lim N(k) . k-+oo k 25. Prove that there exist infinitely many positive integers which are not expressible in the form n 2 + p, where 'II is a positive integer a.nd p is a prime. 22. Let a, d and 'I' be positive integers. For k = 0,1,. .. set 26. Evaluate the infinite series (22.0) Uk 1 Uk( a d T) ;;::; . " (a+kd)(a+(k+1)d)...(a+(k+r)d) s = farctan ( :2 )' n=1 Evaluate the sum " s 2:: 1Lk , k=O 27. Let Pl"",Pn denote 'II (2': 1) distinct integers and let fn(x) be the polynomial of degree 'II given by where n is a positive integer. fn(x):::: (x - pd(x - 112)...(x - Pn). 23. Let Xl,''',X n be 'II (> 1) real numbers. Set Xij Xi - Xj (1  i < j  n). Prove that the polynomial gn(X) (fn(x))2 + 1 
8 PROBLEMS PROBLEMS 9 cannot be expressed as the product of two non-constant polynomials with integral coefficients. 33. Let I denote the closed interval [a,b], a < b. Two functions f( x), g( x) are said to be completely different on I if f( x) =I g( x) for all x in I. Let q(x) and rex) be functions defined on I such that the differentia] equation 28. Two people, A and B, playa game in which the probability that A wins is p, t.he probability that B wins is q, and the probability of a draw is r. At the beginning, A ha.<; m dollars and B has 11 dollars. At the end of each game th(' winn('r takes a dollar from the loser. If .1 and 11 agree to play until one of them Inses aU his/her money, what is the probabilty of A winning all the money'! dx y2 + q(x)y + rex) ha.<; three solutions Yl (:1'), Y2t.:r), Y3(X) which ar(> pa.irwise completely different on .T. If z(x) is a fourth solution such that the pairs offunctions z(:r.), y.(x) a.re completely different for ii, 2, :1, prove that then' exists a wl1st.ant K (cf 0,1) such that 29. Let f(x) be a monic polynomial of degree n  1 with complex co- efficients. Let x}, . .. ,X n denote the n complex roots of f( x). The discriminant DU) of the polynomial f( x) is the complex number (33.0) Yl(I(Y:/. Y3) + (1- J()Y2Y3 (I( 1)Yl+(Y2-J(Y3) (29.0) DU) = n (Xi - Xj)2. l$i<j$n 34. Let an, n = 2,3,..., denote t.he number of ways tlw product b 1 b 2 ... b ll can be bracketed so that only two of the bi are multiplied together at a.ny one time. For e.xample, a2 = 1 since b 1 b 2 can only be bracketed as (hb 2 ), whereas a3 = "l. as blb2b; can be bracketed in two ways, namely, (b l (b 2 b 3 )) and «b 1 b 2 )b 3 ). Obtain a formula for an. Express the discriminant of f(x 2 ) in terms of DU) . 30. Prove that for each positive integer n there exists a circle in t.he xy-plane which contains exactly '/1. lattice points. 31. t '(). be a given non-negative integer. Determine the number S( n) of solut.ions of the equation (35.0) 35. Evaluate the limit L = lim  r tan(ysinx) dx. !I_ O Y 10 (31.0) x + 2y + 2z = n in non-negative integers x, y, z. 36. Let £ be a real number with 0 < £ < 1. Prove that there are infinitely many integers n for w.hich 32. Let n be a fixed integer  2. Determine all functions f(x), which ItffJ bounded for 0 < x < a, and which satisfy the functional equation (36.0) cosn 2: 1- £ . (:12.0) 1 ( ( X ) ( x+a ) ( x+(n-1)a )) f(x) = n 2 f ;; + f --;;- +... + f n . 
10 PROBLEMS PROBLEMS 11 37. Determine a.1I the functions f, which a.re everywhere differentiable a.nd satisfy 41. A,B,C, D are four points lying on a circle such that ABCD i5 a convex quadrilateral. Determine a formula for the radius ofthe circle in terms of a = 1.4.BI, b = IBGI, c ,CDI and d = IDAI. ( X+Y ) (37.0) f(:I:) + fey) = f 1- :r.y for aU real x a.nd y with X.1/ f L 42. Let ABCD be a convex quadl'ila.l.eral. Let P be the point outside ABCD slIch that j.4.PI IPHI a,nd LAPB 90°. The points Q,1l,8 are similarly defmed. Prove that the lines P Rand Q,<; an oJ e!luallcngth and perpendi cular. 38. A point X is chosen inside or on a circle. Two perpendicular chords .4C and BD of the circle are drawn through X. (In I;he case when X is on the circle, the degenerate case, when one ('.hord is a diameter and the other is reduced to a point, is allowed.) Find the greatest and least values which the sum S = IACI + IBDI can take for all possible choices ofthe point X. 43. Determine polynomials p(x,y,z,w) and q(x,y,z,w) with real coefficients such that (43.0) (xy + z + w? - (x 2 - 2z)(y2 - 2w) (p(x, y, z, w»)2 - ($2 _ 2z)(q(x, y,z, w))2 . 39. For n An == { 1,2,. .. define the set An by {0,2,4,6,8,...}, ifn {0,3,6,...,3(n- 1)f2}, if n o (mod 2) , 1 (mod 2) . nQ1 C01 An+k) nQ1 CQ An+k) ? 44. Let C denote the field of complex numbers. Let f ; C --+ C be a function satisfying Is it true that ( 44.0) { f(O) == 0, If(z) f(w)1 = Iz wi, 40. A sequence of repeated independent trials is performed. Each trial has probability p of being successful and probability q ] - p of failing. The trials are continued until an uninterrupted sequence of n successes is obtained. The variable X denotes the number of trials required to achieve this goal. If Pk Prob(X == k), determine the probability generating function P( x) defined by for all z in C and 1lJ = 0,1, i. Prove that fez) = f(l)z or f(l)z, where If(l)1 1. ( 40.0) 00 P(x) L:l)kXk. k=O 45. If $ and y are rational numbers such that (45.0) tan 1!'X = y, 
12 PROBLEMS PROBLEMS 13 prove that x = k/1 for some integer k not congruent to 2 (mod :1). has a solution in integers Xl, X2,..., x"' not all zero, satisfying 46. Let I' be a point inside the triangle ABC. Let AI' mcct BC at 0, Hi' llleet C A at ,f;, and CJ> meet :1.B at P. prove that IXjl ::; [(2na) nn' ], 1::; j ::; n . ( 46.0) IPAi IPB: IPBIIPCI IPCIIPAI tpDllPHI + j PEI TpFI + iPFl lPDI 2': 12. 19. Liouville proved tIwl; if 1:5: 1 < n, GCD(l,n) 1. J f!:r)t '1(.r)d.r. is an elementary fUllction, where f(x) and g(x) are rational functions with degree of g(x) > 0, then 47. Let I a.nd n be positive integers such that Define the integer J;; uniquely by l::;k<n, kl=-1(modn). Let M be the k X 1 matrix whose (i,j)-th entry is J f(x)(!I(x)dx h(x)e 9 (x) , where h(x) is a rational function. Use Liouville's result to prove that ( i 1 )l + j . J e- x2 d:r Let N be the k x I matrix formed by taking the columns of M hI reverse order and writing the entries as the rows of N. What is the relationship between the (i,j)-th entry of M and the (i,j)-th entry of N modulo n? is not an elementary function. 48. Let m and n be integers SUdl that 1 ::; m < n. Let a'ij, i = 1,2,.. ., fn.; j = 1,2, . . . , n, be mnintegers which are not all zero, and set 50. The sequence Xo, Xl>' .. is defined by the conditions a max laijl. l<'<m l$.1$n (50.0) Xo 0, Xl = 1, Xn+l = X" + nX n -1 n+1 n 2': 1. Determine Prove that the system of equations L = lim X n . n-+oo (48.0) { anXl + a12 x 2 + ... + alnX n 0,21 Xl + a22 X 2 + ... + a2nXn 0, =0, 51. Prove that the only integers N 2': 3 with the following property: amI Xl + a m 2 X 2 + . . . + amnX n 0, (,) 1.0) if 1 < k::; Nand GCD(k,N) = 1 then J.. is prime, 
14 PROBLEMS PROBLEMS 15 are Prove that N 3,4,6,8,12,18,24,30. IZjl ::; 1 + A, j = 1,2,. .., n. 52. Find the sum of the infinite series 56. If m. and n are positive integers with m odd, determine s 1 1 1 .1 + 6' 1 1 9' + 11 1 'j- .. . 1--1 it = GCD('.?!" - 1, 2'n T ]) . 53. Semicircles are drawn externally to the sides of a. given triangle. The lengths of the common tangents to these semicircles are 1, m, and n. Relate the quantity 57. If f( x) is a polynomial of degree 2m + 1 with integral coefficients for which there are 2m. r 1 integers k 1 ,. . . ,k 2m +1 such that 1m mn nl -+ +- n 1 m to the lengths of the sides of the triangle. (57.0) f(k 1 ) = .., = f(k 2m + 1 ) = 1, prove that f( x) is not the product of two non-constant polynOInials with integral coefficients. 54. Determine all the functions H : R-I -> R having the properties 58. that Prove that there do not exist integers a, b, c, d (not all zero) such (i) ( ii) ( iii) (iv) H(l,O,O,l) 1, H(>'a,b,Ac,d) >'H(a,b,c,d), Hla,b,c,d) = -H(b,a,d,c), H(a + e,b,c or J,d) H(a,b,c,d) + H(e,b,J, d), (58.0) a 2 + 5b 2 - 2c 2 - 2cd - 3d 2 = 0 . where a,b,c,d,e,J,A are real numbers. 59. Prove that there exist infinitely many positive integers which are not representable as sums offewer than ten squares of odd natural numbers. 55. Let Z1,..., Zn be the complex roots of the equation zn +alz n - l +.... + an 0, 60. Evaluate the integral wllCre 0,1,... ,an are n (?:: 1) complex numbers. Set (60.0) I(k) L oo sin kx cosh x dx ox' A = max lakl . 1 :$ksn where k is a positive integer. 
16 PROBLEMS 61. Prove that 1 ( 2n ) n+ 1 n is an integcr for n 1,2,3,. .. , 62. Find the sum of the infinite scries .'i "" 2 n )'--  a" + 1 . whcff a > 1. 63. Let k be an integer. Prove that the formal power series VI + kx == 1 + alx + a2x2 + ... has integral coefficients if and only if k 0 (mod 4). 64. Let m be a positive integer. Evaluate the determinant of the m X m matrix Urn whose (i,j)-th entry is GCD(i,j). 65. Let land m be positive integers with 1 odd and for which there arc integers x and y with { x2 + y2, x 2 + 8xy + 17y2. Prove that there do not exist integers u and v with (65.0) { 1 == u 2 + v 2 , m == 5u 2 + 16uv + 13v 2 . r PROBLEMS 17 66. Let an 1 1 1 -+ -...+ 2 3 n -ln2. Provc that an cOIlverges and determine its sum. 67. Let .4 :::- {a.i I (J <: i :S; G} he a. S(qllence of sevell iutep;ers satisfying o 'CC Ill) :::: (1.1 :S; _ . . :S; (Iii  6. For i 0,1,...,61et Ni numbcr of aj (0::: j :S; 6) S]Jch that aj . Determine a.U sequences A stich that (67.0) Ni , ==a6-i i==0,1,...,6. 68. Let G be a finite group with identity e. If G contains elements g and h such that (68.0) g5 = e, ghg- l h 2 , determine the order of h. 69. Let a and b be positive integers such that GCD(a,b) == 1, a:;t b (mod 2) . If the set S has the following two properties: (i) a,b E S, (ii) x,y,z E S implies x + y+ z E S, 
18 PROBLEMS PROBLEMS 19 prove that every integer > 2ab belongs to S. 70. Prove that every integer can be expressed in the form x 2 +y2_5z2, where x, y, z are integers. 75. Evaluate the sum of the infinite series 00 1 mn( m + n) . S I: 71. [<;.valuate the SUIIl of the iuIiuit(' serie:-- m>T=J r;CD(m,u=l In2 " L, In3 In4 In 5 + -  +... , ,I ,) 76. A cross-country racer runs a IO-mile race in .')0 minutes. Prove that somewhere along the course the racer raIl 2 miles in exactly 10 minutes. 72. Determine constants a, b aIld c such that Vii 'f ,/Jak 3 + bk 2 + ck + 1 - vak 3 + bk 2 + ck , k=O 77. IJet AB be a line segment with midpoint O. I.J€t R be a point on AB between A and O. Three semicircles are constructed on the same side of AB as follows: S} is the semicircle with centre 0 and radillsl0AI = lOBI; S2 is the semicircle with centre R and radius IARI, meeting RB at C; S3 is the semicircle with centre S (the midpoint of CB) and radius ICSI = ISBI. The common tangent to S2 and S3 touches Sz at P and S3 at Q. The perpendicular to AB through C meets S} at D. Prove that PCQD is a rectangle. for n 1,2, . . . . 73. J.J€t n be a positive integer and a,b integers such that GCD(a,b,n) 1. 78. Determine the inverse of the n x n matrix a} == a (mod n), b 1 == b (mod n), GCD(al,b}) = 1 . (78.0) [    S 1 1 0 1 1 1 1 1 1 Prove that there exist integers a}, b 1 with o where n 2:: 2. 74. For n = 1,2,... let o5( n) denote the sum of the digits of 2 n . Thus, for example, as 2 8 256 we have 05(8) 2+5+6 = 13. Determine all positive Integers n such that 79. Evaluate the sum (71.0) o5( n) = o5( n + 1) . (79.0) n-l S(n) = I:(-llcosn(k1rln) , k=O 
20 PROBLEMS PROBLEMS 21 where n is a positive integer. 85. (0.0) [ -1 1 J B'J + C'J . o -2 Let G be a group which has the following two properties: (i) G has no element of order 2, (ii) (xy)2 (yx?, for all x,y E G. Prove that G is ablian. 80. Determine 2 x 2 matrices [] and C with integr;J entries such that (85.0) 86. Let A [a;j] be an n X n real symmetric matrix whose entries satisfy 81. Find two non-congruent similar triangles with sides of integral length having the lengths of two sides of OIle triangle equal to the lengths of two sides of the other. (86.0) ai. = 1, L laijl S 2 , j=l for all iI, 2, . . . ,n. Prove that 0 S det A S 1. 82. Let a,b,c be three real numbers with a < b < c. The function I(x) is continuous on [a,c] and differentiahle on (a,c). The derivative f'ex) is strictly increasing on (a, c). Prove that 87. Let R be a finite ring containing an element T which is not a divisor of zero. Prove that R mllst have a multiplicative identity. (82.0) (c - b)f(a) + (b - a)f(c) > (c - a)f(b). 88. Set I n = {1,2,..., n}. For each non-empty subset S of I n define w(S) = maxS - minS. sES sES Detennine the average of w(S) over all non-empty subsets S of I n . 83. The sequence {a".. I Tn 1,2,...} is such that am > am+! > o , m = 1,2" .. , and :L=l am converges. Prove that = L m(a m - am+IJ m=1 89. Prove that the number of odd binomial coefficients in each row of I'ascal's triangle is a power of 2. converges and determine its sum. 90. From the n X n array 1 2 3 2: ] n+l n+2 n+3 2n+ 1 2n+2 2n+3 3n (n-l)n+ 1 (n-l)n+2 (n-l)n+3 n 2 84. The continued fraction of VD, where D is an odd nons quare integer > 5, has a period of length one. What is the length of the period of the continued fraction of t(1 + .JJ5)? 
22 PROBLEMS PROBLEMS is irreducible ov('r Z for n ;::: 4. a number Xl is sdected. The row and column containing Xl are then deIcted. From the resulting array a number X2 is selected, and its row and column deleted as before. The sclection is continued until only onc number X T . remains available for selection. Determine the sum .r-l + :e,t + . .. + Xn. 23 95. Let aI:"', an b(' n (;::: 4) distinct real numbers. Determine the eneral solution of the system of n 2 !inl'ar equations 91. Suppose that p X's and q O's a.re plac(>d on tIle drcumferenc r . of a circle. The number of occurrences of two a.djacent X's is tI and the number of occurrences of two adja.cent O's is b. D(tennille a b in terms of J1 and q. (95.0) 92. In the triangular array 1 1 1 1 1 2 3 2 1 1 3 6 7 6 3 1 1 4 10 16 ]9 16 10 4 1 96. 1 Xl+ X2+"'+ x" tl] "1 + {/2:e:l + . .. + anx" 2 a'Ix] + aX2 +... + (l.,."X", 3 n-3 + + n-:J x a- Xl + lL2 X2 . . . an n in the n Ullknowns Xl,'" ,X. n . (92.0) every entry (except Hie top 1) is the slim of the entry a immediately above it, and the entries band c immediately to the left and right of a. Ab5ence of an entry indicates zero. Prove that every row aft('r the second row contains an entry which is even. 97. 93. A se<luence of n real numbers Xl, . . . , X n satisfies (93.0) { Xl = 0, IXil = IXi-l + cl (2:::; i:::; n), (97.0) where c is a positive real number. Determine a lower bound for the average of Xl,'" , X n as a function of conly. 98. 94. (94.0) (98.0) Prove that the polynomial f(x) x"'+x3+x2+X+5 Evaluate the sum 1 0, 0, c tJ, 0, N.:= 2,3,.... B(N) = mn l:::;m<n:::;N m+n>N GGD(m.n)=1 Evaluate the limit I r, n j L = lim - L L '2 + k2 . n-oo 11 j=l k=l J Prove that 311" . 211" r;< tan +4sm = v11. 11 11 
21 99. For 11 =: 1,2,.. . let Ewtluatc the sum C n 1 1 1 1 +-+-+...+- 2 3 n' OG S'- L C n  - r.=l n(n+ 1) . 100. PROBLEMS For x > .I determine the f;um of the infinite series X x 2 x4 .1:+1 + (x+1}(x 2 +1) + (x +1)(x 2 + 1)(x 4 + 1) +.... THE HINTS I H/.ill shrou(led ,in llu fiaY'kt, $1 niyht, me look to lh f)oBI lI,ilh .'2:]1('('- tation: a hint of a bright new day. Aleksander Sergeevich Pushkin (1799-1837) 1. Let (p-I)/2 N(k) L 1 , i,j=l r,+n, =.k (mod p) k = O,l,...,p-l, and prove that N(k) N(l), k l,2,...,p 1. Next, evaluate N(O) and N(l), and then deduce the value of E(p) from p-I E(p) = Lw k N(k) . k=O 2. Prove that N(k) LLL 1, x 'Y Z where the variable x is summed from -k to k; the variable y is summed from max(-k,x k) to min(k,x + k)j and the variable z is summed from 
26 HINTS max(-k,x k,y k)tomin(k,x+k,y+k). Thencxpressthctriplesumas the iHl1ll of six sums specified aecording to the relative sizes of O,x and y. 3. First U8e the fact that w 2 - 1 (mod p) to prove that there are integNs a and c sucll tha.t p = (2 + c 2 . Then lei, sand t be integers such tha.t at -.. cs 1. Prow tha.t as + ct f/J1 (mod p), whr(' f :1:1, and deduce that a.n intt'!!;c1' !J can be fi.>und o that b (-' S a.fJ) and d (=- t - tg) satisfy ll.b+ed =- /117, nd be = 1 and b + d 2 c. (w 2 + 1)/p. 4. Prove that m m L (d1(n) - d 3 (n)) = L L (_1)<d-1)/2, n=l n=l din dodd and then intercha.nge the order of summation of the SIl1llS on the right side. 5. Rule out the possibilities x ° (mod 2) and x 3 (mod ,1) by congruence considera.tions. If x 1 (mod 4), prove that there is at least one prime p == 3 (mod 4) dividing x 2 - 3x + 9. Deduce that p divides x 3 + 27, and then obtain a contradiction. 6. "Gse the identity f(Xbydf(X2,yJ == (axlx2 + bXIY2 + 1JX2YI + CYlY2)2 + (ae - b)2(X1Y2 - X2Y1? together with simple inequalities. 7. Prove that exactly one of the triples (a,b,c) = (R, S,T), (T,-S + 2T,R S + T), (R - S + T,2R - S,R), satisfies a ::; b < c, or a 2: b> c , I lINTS 27 hy considering cases depending upon the relative sizes of R, Sand T. 8. C..msilkr the sign of the discriminant of (a11 bA)x 2 + 2(aC - cA)xy + (bG - clJ)y2 . 9. Prove th"l the quantity I nn 2" 2 n L T .k=1 n-1 1 I L2 k .k=0 tends to zero as n -> 00. 10. Consider the Iase when n = p+ 1 and m = p, where p is a prime suitably large compared with c. 11. Assume that 2D( D + Al A2 + f.B 1 B2) is a square, where ( = :1:1. If D is odd, show that { D + Al , A2 + f.B 1 B2 == 2D1l 2 , D - AI A2 tBIB2 2D\.r2 A 1 B2 - f.A2Bl = 2DllV Deduce that 112 + V 2 = 1. Then consider the four possibilities (If, V) (:1:1,0), (0,:1:1). The case D even ca.n be treated similarly. 12. Set a::l:: = V 1985:1: 31 V'1985 , {1::1:: = V 3 970:1: 64 V'1985 , and prove that a+ + a_ = fJ+, a+ a_ ;3-. 
28 HINTS 13. If (x, y) is a :solution of (13.0), prove that x 2 + xy y2 = :l::F , and then solve tIJe system of equations { x2 + y2 x 2 +2xy +2y2 = x 2 + xy _ y2 k, I, :l::F, for x 2 , xy and y2. 14. F'actor the left side of (14.0). 15. Make the following argument mathematically rigorous: L\nx In(1- x) dx = L 1 00" In x Ldx o ";:1 k 00 1 1 - L k { x" In x dx ";:1 10 00 1 t; k(k + 1)2 00 1 00 1 t; k(k+ 1) - t; (k + 1)2 1 _ ( 2 1) = 71"2 2 - 6' IllNTS 29 16. Taking n = 1,2,. . . ,6 in (16.0), we obtain 0,(1) = 1/2, a(2) = -1/3, a(3) 1/4, a(4) = -1/5, a(5) = 1/6, a(6) = -1/7 . This suggests that a( n) ( -1 )n+1/( n + 1), which can be proved by induction on n. 17. Consider three cases accordin to thf' foJ1owio!!; va.1ues of tlH Legendre symbol: ( n 2 : k ) = 1 or ( n + ;2 + k ) = 1 or ( n 2 : k ) = ( n + ;2 + k ) = -1. In the third case, the identity (n 2 + n + k)2 + k = (n 2 + k) (n + 1)2 + k) is useful. 18. Rearrange the terms of the partial sum N ( 1 1 1 )  4n + 1 + 4n + 3 - 2n + 2 ' and then let N --> 00. 19. Use ( An ) 2 ( An ) 2 2 2 ( An -;;- = an + -;- - an S; 2a n + -;;- anr 
30 to prove that  ( An ) 2 m  n 5 4 E a + 2 E ( n r - 4 , a:An . Then use -2a" It,,:= ( .112 , . " A2 ) n-L ., " a;' 5 -(rt;. A;'_d 1.0 pJ'O\" t.bat -2 ;,;"'- u"A1I .  .IF L., < -, L., n n=l n - n=t n(n + 1) . Putting these two incquaHties together, deduce that m ( 2 ) ( A 2 m L , 1 - - -..2: ) 5 ,t "'- 0,2 . n=t n + 1 n L., n n=1 20. Use the identity () -2 ( m e n k - l ) - e') (kt» ) ( 2n ) . .k+l 21. All int.egral solutions of ax + by k are given by x 0;:: 9 + bt, y = h - at, t = 0, where (g, h) is a particular solution of ax + by = k. ... , 22. Prove that Uk = Vk-l - l'k, k=O,l,..., HINT IIINTS 31 wherc 1 l'k = , (a + (k + l)d)... (a + (k + r)d)1'd , -1,0,1,... 23. Prove that the st.l'Ollg'r inf-'qu<tlity 11 , F(xu"rt:"...,.1'"nJn) < L:ri.; k=l ( . \ 2 ] './'I. 1 n <--.. ) k=l holds by replacing ea.ch Xi by Xi - M for suita.ble AI (23.0). AfVb..., xn) in 24. Apply the Cauchy-Schwarz inequality to r::. 1 \ an -..rn -..rn . L-." n n=l 25. Consider the integers (3m + 2)2, m = 1,2.... . 26. Use the identity arct.an ( :'2 ) arcta.n ( n  1 ) arctan (  ) , n = 2.3, . .. . n+1 27. Suppose that. gn(x) = h(x)k(x), where hex) and k(x) a.re non- constant polynomials with integral coefficients. Show that h( x) and k( x) can be taken to be positive for all reaJ x, and that h(p;) :(Pi) 1, i = 1,2,..., n. Deduce that hex) and k(x) are both of degree n, and determine the 
32 HINTS form of both hex) and k(x). Obtain a contradiction by equating appropriate coefficients in g",( x) and h( x )A( x). 28. Let l)(k), k = 0,1,... , denote the probability that 11 wins when A has k dollars. Prove the recurrence relation ap(k+2)-(a+b)p(k+1)+bp(k) O. 29. If xl:..., X n are the n roots of f(x), the 2n roots of f(x 2 ) are f:y'Xl, f:y'X2, .... f:ft; . 30. Find a point P such that any two different la.ttice point must be at different distances from P. Then consider the lattice points sequentially according to their increasing distanc.es from P. 31. Determine the generating function 00 L S(n)t n . n=O 32. such that As I(x) is bounded on (O,a) there exists a positive constant K If(x)1< J(, O<x<a. Use (32.0) to deduce successively that { If(x)1 < Kfn, x)1 < Kfn 2 , etc. O<x<a, O<x<a, II iNTS 33. 33 Consider the derivative of the function (YI - Y2)(Y3 z) f(x) = (YI - Y,,)(Y2 - z) 34. Set al 1. Prove the recurrence relation + tl a I + . . . + a",-I tt2 + a.,aI , an+I aIUn 2 n- . . Ii ( ) - '\.-00 a x n satbiles and use it to show that the generatmg functIOn. x - L"",=l " A(X)2 A(x) - x. Then solve for 11(x). 35. Use L'H6pital's rule, or use the inequality t S tan t S t + t\ to estimate the integral J; tan(y sin x) dx. Osts1, . el if e is an irrational number, 36. Use a result due to HurwItz, nam y: 0 and GCD(a b) = 1 there are infinitely many rational numbers afb with b> ., such that 37. Ie - afbl < 1f(../5b 2 ). Differentiate (37.0) with respect to x and Y to obtain (1 + x 2 )f'(x) (1 + y2)f'(y) . 
:H HINTS 38. Introduce a coordinate svst d . that max S 4R d . , , -. ern an use sImple inequalities to show t an mIn.5 = 2R, where R iR the radius of the circle. 39. Prove that u ( n An+k ) == X n Y 11.=1 k=1 and n ( u ,'lnH ) = ,- U Y , n=I k=1 where x -= {O, 2. 4,. .. }, y = {O, 3, (j, .. . } . 40. Prove that Pk = { n qp" ,0<k<n-1 , k: n - , , , n + 1  k  2n , and Pk = (1 kt l Pi ) qpn, k > 2n . \ .=0 Use these to find a linear equation satisfied by P( x). 41. First prove that the circu ad . . 1 m and n . . b 1111' IUS of a tnangle with side.<; oflen g th , IS gIven y lm n -J( l + m + n)(l + m - nJ(l Next show that m + n)( -l + m + n) . IACI  bd)(ad+ be) (ab + cd) I HINTS 3.') FinaUy, apply the above two results to !:"AHC. I 42. Consider the quadrilateral ABCD as lying in the complex plane. RepreseIll the vertices A, B, C. f) hy t.he complex numbNs l!, b,c, d respec- tively_ Prove that. P, Q, R, .9 are represented by the numbers I (J.:x i ) la + ib , )' e2' )lc+d), rcspediv{.ly. 'l'll"11 rdat" l' - 1- and q  43. Try a solution of the form p=xy+X, (i) (b + ic), (t;i ) (d+ ilL), q=y+Y, where X and Yare polynomials in x, wand z. Substitute in (43.0) and solve the resulting equations for X and Y. 44. Setn =- f(l)and/ f(i). Provethatlal = 1t31 = 1,la-t31 =.:-12. Deduce that a 2 + {32 - 0 so that t3 = W, f :l:i. Next from (4<1.0) deduce that { af(z) + af(z) 'iif(z) af(z) Now solve for fez). = z+z, -fiz + fiz . 45. JJet x be a. rational number such that Y = tan 7rX is rational. Prove that z = 2 cos27rx is a rational root of a monic polynomial with integral coefficients. Deduce that z = 0, :1:1, :1:2. 46. Let SI,SZ,S3 denote the areas of b.PBC,!:"PCA,!:"PAB'respec- tively. Prove that !PAl _ S2+ S3 IPDI - Sl 
37 36 HINTS HINTS 47. Prove that the (i,j)-th entry of N is 1 times the (i,j)-th entry of AI modulo n, 51. Let Pk denote the k-th prime. Suppose that N > 121 is an integer with the property (51.0). Let Pn be the largest prime less tha or equ.al to.J!i, th 1. > 5 nd N < P 2 Use property (51.0) to obtam the mequallty so a n _ ,a n+l' N  PIP2 .. . Pw Then use Bertrand's postulate Pk+1  2Pk, k =:: 1,2,... , . h . . 1 . r If.m d IPCI WIt slml ar expreSSIOns lor IPEI an IPFI' 48. There are (N + 1)" vectors (Yl,Y2,. .., y,,) of intef!;ers satisfying o  Yj  N, 1  j  n. For each of these vectors the corresponding value of to obtain PIP2 . . . Pn-2 < 8 .' 2 Deduce the contradiction n from the mequahty 1'11-'2'" Pn < Pn+I' . Check property (51.0) for the integers N 3,4,...,121 dIrectly.  4. Li Li(YbY2"" ,Yn) = aiIYl +... +ainYn, 1  i  m, satisfies -naN  Li  naN, so the vector (L b L2"'" Lm) of integers can take on at most (2naN + 1)m different values. Choose N appropriately and apply Diricblet's box principle. 52. Prove that 49. Suppose that f e- x2 dx is an elementary function, so that by Liouville's result, there is a rational function p(x)/q(x), where p(x) and q(x) are polynomials witb no common factor, such that t x 2 + x + 1 d S =:: 10 x4 + x3 + x2 + x + 1 x and then use partial fractions to evaluate the integral. J _x2 d p(x) _",2 e x == -e q(x) 53. Let IABI -= 2c, IB GI == 2a, IGAI == 2b. S how that 1== J (a-b+c)(a+b-c) with similar expressions for m and n. Differentiate both sides to obtain p'(x)q(x) - p(x)q'(x) 2xp(x)q(x) -= q(x? , 54. Evaluate and deduce that q(x) is a nonconstant polynomial. Let c denote one of the complex rQots of q(x) and obtain a contradiction by expressing q(x) in the form q(x) == (x - c)mr(x), with rex) not divisible by (x - c). H(1,1,0,O), H(0,0,1,1), H(O,l,l,O), H(1,0,0,1), using (i) and (iii). Then express H(a,b,c,d) in terms ofthese quantities by means of (i), (ii), (iii) and (iv). 50. Prove that n-l (-1)i X n ==?= i+ l ' n == 1,2,... . .=0 55. Set fez) == z"+alz"-l+...+a" and note that for z =1= ° we have If(z)1 =:: Izn (1+ a; +...+ :: )1 
38 HINTS HINTS 39 Iz"1 1 1+ +."+ an i z z1t ;;:: Iznl ( 1 _  .. . _ la n l ) Izi Izln ;;:: Iznl (1- 1;1 I,n ) , 59. Consider the integers 72k: + 42, k = 0,1,. ., . 60. Use the identity 56. Ddinr- intr-gns /; ami I b.I' k ( k- ) 2 k sin k:x cos k x 'L: r t;i n 27':/; . r=1 2 m 1 l;d, 2 n + 1 = ld , and then consider 61. Express 2 mn = (kd + l)n -, (ld _ l)m . n: 1 e:) as the difference of two binomial coefficients. 57. Suppose that f(x) g(x)h(x), where g(x) and hex) are non con- stant polynomials with integral coefficients chosen so that 62. Use the identity Deduce that def?; (g(;1.:») :::; 'In and that g(k,) = :1::1, i = 1,2...., 2m + 1. Let ( +1 (resp. -1) if +1 (resp. -1) occurs at lea.<;t 'In + 1 times among the values g( k:i) :I:: 1, i = 1,2,..., 2m + 1. Then consider the polynomial gee) - (. + 1 a 2n - 1 2n+i a 2n + 1 _ 1 ' a> 1. deg(g(x):::; deg(h(x)). 2" 2" 63. Prove that 58. Suppose a, b, c, d a.re integers, not all zero, sa.tisfying (58.0). Show that without loss of generality a, b, c, d may be ta.ken to satisfy 1 ( 2n - 2 ) ( k ) " a,,=2(-1)n-I;;; n-1 4' GCD(a,b,c,d) = 1 . and appeal to Problem 61. By considering (58.0) modulo 5 prove t.hat a == b == c == d == 0 (mod 5) . 64 L t C C C denote the columns of Mm. Determine a . e 1, 2, .. .. .., .1m .. linear combination of C h C2,"" Cm-I which when added to C m gIves the column (O,O,...,4>(m)t. Deduce that det U m = 4>('111) det Mrn-i' 
40 HINTS IUNTS 41 65.. . Assume (65.0) holds and use congruences modulo 8 t bt ' contradictIOn. 0 0 am a for suitable constants a, b, .. . ,f. The case m odd is treated similarly. 66. Prove that 71. Note that 1 1 (_l)n-Ixn a'n d I + ,x, o x alld use this reprt'.sentation of an to de(Juce that In2 2 In 3 In 4 In 2n 3+1 ...+ 2n = In 2 ( 1-1 1 + . .. + 1 ) + t I n k 2 n k=l k 2n Ink L-- k=1 k ' I N 1 I ]; an - l (1: x )2 dx $ 1 +2 . and estiul'l.tl. Lk=l(lnk)fk fur largp 'I/. using I.IH' Euler-Ma.t'!,a1lI'in Rumma.tinl! formula. the 67 . b et A b a sequnce of the required type, and let k denote o num er 0 zeros III A. FIrst prove that k ::: 3. Deduce that A { ,O,O,a 3 ,a4,a s ,3},wherel<a3<a 4 <a s < 3 ' fh th N - - - -' en prove at 1 = 2. 72. Express (v'k+1- Vk)3 in the form Vp (k) + 1 Vp(k), where p( k) is a cubic polynomial in k. 68. }'rove that 73. Choose al to be any nonzero integer such that a] a (mod n). Then set b l = b + rn, where T is the product of those primes which divide a] but do not divide either b or n. Prove that GCD(a},bd = L g nh g -n == h 2n , 71.=1,2,...,5. 74. Prove that s(n + 1) == 2s(n) (mod 3), and use this conguence to show that there are no positive integers n satisfying s(n) s(n + 1). 69. 75. Show that 00 00 1 s== L f L m,n=l d=1 Prove that every integer N > 2ab is of the form N==xa+ y b , x>O > 0 - , y - , x + y == 1 (mod 2) , and that all integers of this form belong to S. 70. If m is even, say m == 271., show that by collecting together those m,n in the sum A L:.n=llf(mn(m + n)) having the same value for GC D( m, 71.). Then evaluate the sum A by proving that it is equal to the integral m = (an + b)2 + (cn + d)2 - 5(en + f)2 , l lln 2 (1 - x) dx, o x 
42 HINTS HINTS 1.3 which can be ('!valuated by means of the transformation J: = 1 _ e--". ar('! similar if a/b = bfc c/d. Choose positive integers to satisfy this relation remembering that the triangle inequalities c < a + b, etc must be satisfied. 76. App!y the int,ermediate v<tlu('! theorem to the function T( x) de- fined to be the tIme taken in minutes by the racer to :run from th . 1. . '1 I th . . ('! pom .c Ill! es a ong ('! C01}fS('! to the point x + 2 miles along the course. 82. (b,c). Apply the mean vdlue thmrem to f(T-) on th('!intervals (tl,b) a.1l.! 77. Choose a coordinate syst'rn so that 83. First show that linIn__;."x. 7W n --' O. Then [pt 1/, . = in A (-1,0), 0 (0,0), 1) "" (1,0). Then R = (-a,O) with 0 < a < L Deduce that n L k«(Ik - akH) k=l n Lak k=l na ll +1. C = (1 - 2a,0) , S = (1 - a,O), D = f l- 2a,2 V( I.(1 - a») , P = 2a 2 - 4a + 1 ,2(1- a)va (1 - a») Q 1 2a 2 ,2a va (l- a») , a.nd calculate the slopes of PC, PD, QC and QD. 84. Use the fact that the length of the period of the continued fraction of ..;J5is one, and that D is an odd nonsquare integer > 5, to show that D = 4c 2 + 1, C ?:: 2. Then determine the continued fraction of HI + ..;J5). 85. lor x, y E G prove that (XY:I--ly-l)2 1. 78. that U 2 Let I denote the n X n identity matrix. Set [J = 8 + I. Prove nll. Seek an inverse of S of the form cU - I. 86. Let).. denote one of the eigenvalues of A and let ;£ be a nonzero eigenvector of A corresponding to)... By applying simple inequalities to an appropriate row of A;£ = )..;£, deduce that I).. - 11 ::; 1. Then use the fact that A is real symmetric and the relationship between det A and the eigenvalues of A. 79.. R:eplace cos(k1r/n) by (w k +w-- k )/2, where w = exp(7ri/n), and use the bmOIllIal theorem. 80. Let A [ -1 1 ] o -2 and show that A3 + 3.4 2 + 2A = O. Then 87. Show that there exists an integer k ?:: 2 such that r = r k . Then prove that r k -- l is a multiplicative identity for R. consider (A + 1)3. 81. Let the sides of the trian g les be a b c and b C d ' I " he t t . 1 ' , . ,.,. . wo flang es 88. For 1 ::; k ::; 1 ::; n let 8(k,l) denote the set of subsets of J" with minsEs S k and maxsES S = I. Evaluate IS(k,l)1 and then compute 
44 HINTS HINTS 45 ESPn w(S) using is repeated down the left edge of the array from the fourth row down. L w(S') 4>#-St;;J n L (l- k)/S(k,l)1 lk<I", 93 L t be any real number such that I X n + l l = IX n + cl, and . e Xn+l consider Efjl x; . 89. Write n in binary notation, say, n = 2 G1 + 2<>2 + . . . + 2 Gk , 94. lfwe have f(x) = g(x)h(x) then without loss of generality g(O) ::1:1, 1£(0) ::1:5. Prove that one of the comple..x roots,6 of g{ x) satisfies 1,61 ::; 1, and then dedl1n! that If(f:i) 1 2': ]. where aI, . . . ,ak are integers such that al > a2 > ... > ak 2': 0, and then use (1+xf'Q (1 + x)'" 1 + x 2Q (mod 2) , = (1+x)2Ql(1+x)2Q2...(1+x)2Qk. 95. Set f(x) = (x al)(X - a2)"'(x - an)' 90. Suppose that Xi, 1::; i ::; n belongs to the Ti-th row and the si-th column. Show that Prove that n LXi .=1 n '" '" 2 '" == n L.i Ti - n + L.i s; , ;=1 ;=1 ( P(l)' '''' f'("'» ) and ( Pl) '"'' Pn» ) are two solutions of (95.0). Deduce the general. solution of (95.0) from these two solutions. and then use the fact that both { Tl,..., Tn} and {81,..., 8n} aTe permuta- tions of {1,2,.. .,n}. 96. By picking out the terms with n = N in the sum s(N), show that s(N) = s(N - 1) for N ;?: 3. 91. Let N:r;:r;,N:r;o, No""N oo denote the number of occurrences of XX, XO, OX, 00 respectively. Relate N",:r;, N",o, No."N"" to a,b,p,q. Prove that No", = N:r;o, and deduce the value of a - b in terms of p and q. 97. Prove that 92. Consider the entries of the triangular array modulo 2. Show that the pattern L= {I {I dxdY, 10 10 x + Y and evaluate the double integral using polar coordinates. 1 101 1 000 1 1 1 0 101 0 98. For convenience set p = '!rIll, and let c = cosp, s =: smp. Use the imaginary part of (C+iS)l1 = -1, 
46 HINTS to prove that (Us - 44s: J + 3285)2 == llc 2 (1 48 2 Y,!. Then show that tan3p+ 4sin2p " 2 5 " . r,-:. , , . =:tvll. THE SOLUTIONS l)...duc!' that the + sign holds by confiiderinp; the sign of tile left side. 99. ists. Use partial summation aud the fact that limk-+oo(c,,: Ink) ex- Some pwple think we are W1'01/.Y, but only time will tell: given all the alternativES, 'I.ve have the solution. 100. Use the identity 1 x 2 "  (x + 1)(x 2 4;-,1)(x 4 + 1)...(x2" + 1) _ 3.'2 1 (x 2 "+' 1) - 1 Lev Davydovich Bronstein Trotsky (1879-1940) 1 -1 1. product Let p denote an odd prime and set w = f'.xp(21ri/p). Evaluate the (1.0) R(p) (W T1 + W T2 + ... + w T (p-'){2)(W n , + w n2 + ... + w n (p-'J{2), where 1'17, . . , r(p-l)/2 denote the (p - 1)/2 quadratic residues modulo p and nh"', n(p-l)/2 denote the (p 1)/2 quadratic nonresidues modulo p. Solution: We set q == (p - 1)/2 and { 0 if p == 1 (mod 4), (1.1) E = 1: if p== 3 (mod 4), and for k == 0,1,... ,p 1 let (1.2) N(k) == q L 1. itj:1 T.+n,=k (modp) 
48 SOLUTIONS SOLUTIONS 49 If k is a quadratic residue (resp. nonresidue) (mod p) {kri: i = 1,2,...,q} is a complete system of quadratic residues (resp. nonresidues) (mod p) and {knj : j 1,2,..., q} is a complete system of quadratic nonresidues (resp. residues) (mod p). Replacing ri by kri and nj by kni in (1.2), where 1 s: k s: P 1, we obtain . W'ri+nj k=O i,j=1 Ti+nj=k (roml p) (1.4) N(O) ,,} N(k) k=O N l () + ]V (I )( (,) + w 2 + . . . + wP I) N(O) 1\'(1) t.lJ. (q.. <)/'2, by (1.'1) and (Uj) , (1.3) N(k)=N(l), k-1,2,...,p-l. Next, we Ilotn that 0;;: fq, that is i,j=1 Ti=-n 1 (modp) J'(p ) { (1 p)/4, if p (1 t p)/4, if p 1 (mod 4) , 3 (mod 4) , as -1 is a quadratic residue (mod p) for p = 1 (mod 4) and -1 is a quadratic nonresidue (mod p) for p 3 (mod 4). Now as as required. ( 1.5) N(k) 10;;: q2, 2. Let k denote a positive integer. Determine the number N(k) of triples (x, y, z) ofintegcrs satisfying { ixl < k, Ivl s: k, izl s: k, (2.0) Ix - yl s: k, Iy zl s: k, Iz xl s: k. we obtain, from (1.3), (1.4), a.nd (1.5), fq + 2qN(1) q2, (1.6) N(I) (q f)/2. Solution: The required number N(k) oftriples is given by N(/d that is Finally, we have Ixl$k Iyl$k Ix-yl$k Izl$k Iy-zl$k Iz-xl$k B(p) (EW T .) (t wnj ) ;,vTi+nj k k k 0;;: 1, x=-k y=-k z=-k x-k$y5,x+k x-k$z$x+k y-k$z5,y+k 
50 SOLUTIONS SOLUTIONS 51 t1!at is (2.1) k N(k) I: I: I:1, x=-k y z -1 -I .r+k S3 I: I: I: 1 =F, x=-k y=x z=-k k x-I k .";.. L L L 1 F , , ,..=1 1/=0 ==:r.-k k -1 1I+ k Sf, L L 1 =E, X=O y=x-k z=.c-k where the second sum is taken over y = max(-k,x k) to y min (k.x +k), and the third sum is ta.ken over 2- = max(-k,x k,y k) to z min(k,x+ k,y + k). We now split the sum on th right of (2.1) into six sums SJ,... ,S6, wher(' x a.nd yare restricted as follows: O::;:x<;y, x < 0::;: y, x::;: y < 0, 0:::; y < x, y<O::;x, y<x<O, in ''''1 : in fh; in 8 3 ; in 8..; in 8 5 ; in 8 6 . /)'6 = -1 L .r=-k+l x-I L y==-k ?I+k I:1 z=-k 1 _::.: -(k 6 1 )k( 41. + 1) . Thus we have N(k) Sl + 8 2 + . . . + ''''6 !(k + l)(k + 2)(4k + 3) + k(k + 1)(2k + 1) 6 3 1 +-(k - 1)k(4k + 1) 6 4k 3 + 6k 2 + 4k + 1 (k + 1)4 k 4 . Clearly, we have k k k 51 I: I: 1 x=o II=X z=y-k k k I: I:(2k+ 1- y) x=o 1/=X 1 k = 2' L(k+ 1- x)(3k+2 - x) x=o 1 k 2' I:«k + 1)(3k + 2) - (4k + 3)x + x 2 ) x=(J  (k+1)2(3k+2)- + 2 + + k(k+12k+1» ) !(k + l)(k + 2)(4k + 3). 6 Similarly, with E denoting k(k + 1)(2k + 1)/3, w obtain 3. Let p == 1 (mod 4) be prime. It is known that there exists a unique integer tV == u:(p) such that w 2 == --1 (mod p), 0 < 1lJ < pf2. (For example, '1/)(5) = 2,w(13) = 5.) Prove that there exist integers a,b,c,d with ad bc = 1 such that pX2 + 2mXY + (w 2 + 1) y 2 == (aX + by)2 + (eX + dy)2. p S2 -1 x+k I: I: .r=-k y=o x+k I:1 z=y-/.; =E, (For example, when p = 5 we have 5X2 +4XY + y 2 == X 2 + (2X + y)2, 
52 SOLUTIONS and when p::::: 13 we have 13X 2 + 10XY + 2y 2 == (3X + y)2 + (2X + y)2.) Solution: We make use of the following property of the reals: if r is any reaJ number, and n is a positive integer, then there exists a rationaJ number h/k such that (3.1) IT - il  ken  1)' 1  k  n, GCD(h,k)::::: 1. Taking T ::::: -w(p)/p and n ::::: [y'P], we see that there are integers a and e such that I -w(p) - ! I <, 1  a < vIP. p a avIP Setting e ::::: w(p)a+pe, we see from (3.2) that lei < vIP, and so 0 < a 2 +c 2 < 2p. But c wa (mod p), and so 0,2 + e 2 == 0,2(1 + w 2 ) 0 (mod p), showing that (3.2) (3.3) p ::::: 0,2 + c 2 . As p is a prime, we see from (3.3) that GCD(a,e) ::::: 1. Hence, we can choose integers sand t such that (3.4) at-cs==l. Hence (as + et - w)( as + ct + w) ::::: (as + et)2 w 2 (0,2 + c 2 )(S2 + t 2 ) - (at _ es)2 _ w2 p(S2 + t 2 ) (1 + w 2 ) - 0 (mod p) , so that (3.5) as+ct == fw (modp), f::::::H. SOLUTIONS 53 Hence there is an integer 9 such that (3.6) as + et ::::: fw + gp . Set (3.7) b == sag, d t - eg . Then, by (a.a), (a.4), (a.6), and (3.7), we have (3.8) ab + cd == fw, ad - be = 1 . We now obtain p(b 2 + d 2 ) ::::: (a 2 + e 2 )(b 2 + d 2 ) (ab + ed)2 + (ad - be)2 ::::: w 2 +1, so that (3.9) b 2 +d 2 (w 2 +1)/p. Then, from (3.3), (3.8), and (3.9), we have (w 2 + 1) 2 (3.10) (aX + bY)2 + (eX + dy)2::::: pX 2 + 2fwXY + p Y If f 1 then (3.10) is the required identity. If f == -1, replace b,e,Y by -b, -e, -Y respectively to obtain the desired result. 4 L t d (n) r ::::: 0 1 2 3 denote the number of positive integral . e T, , , , , . . divisors of n which are of the form 4k + T. Let m denote a positIve mteger. Prove that (4.0) m 2.:)d l (n) - d 3 (n) n=l 00 " [ m ] ?:(-1)3 2j+l . 3=0 
54 SOLUTIONS Solution: We have m m L L (_1)(d-I)/2 "",1 dl" d odd L(d1(n) d 3 (n» n=l L L (-1 )(d-I)/2 ,1 tJdd 15:.{J.: 771, L(_1)(d-I)/2 cl (.,tltl L 1 I <,.k<m/J L (_1)(d-l)/2 [ 111- ] dodd d fe-oj [  ] . j=O 2J + 1 This completes the proof of (4.0). 5. Prove that the equation (5.0) y2 x:, + 23 has no solutions in integers x and y. Solution: Suppose that (x,y) is a solution of (5.0) in integers. If x o (mod 2) then (5.0) gives y2 =0 3 (mod 4), which is impossible. Hence, we must have x 1 (mod 2). If x =0 3 (mud 4) then (5.0) gives y2 =0 2 (mod 4), which is impossible. Hence, we see that x 1 (mod 4). In tliis case we have x2 - 3x +9 =0 3 (mod 4), and so there is at least one prime p 3 (mod 4) dividing x2 3x + 9. Since x2 3x + 9 is a factor of x3 + 27, we have x 3 + 27 0 (mod p). Thus by (5.0) we have y2 -4 (mod p). This congruence is insolvable as -4 is not a quadratic residue for any prime p =0 3 (mod 4), showing that (5.0) ha.'J no solutions in integers x and y. 6. Let f( x, y) ax 2 + 2hxy+ cy2 be a positive-definite quadratic. form. SOLUTIONS 55 Prove that (f(Xl, ydf(X2' 1tl))1/2 f(XI - X2, YI - Y2) (6.0) 2: (ac - ti)(XIY2 X"lytJ2, for all rea] numbers .1;], X2, Yb Y'l' Solution: First we lIote that ac - b 2 > 0 as f is positive-definitp. \Ve ue the identity (6.1) (ax + 2bxIYl + cyn(ax + 2bx2V2 + cy) :::: ( ax IX2 + bXIY2 + bX2YI + CY1Y2)2 + (ac ( 2 )(XIV2- XZJ/I?' Set E I =f(XbYtJ2':O, Ez f(X2,Y2) 2:0, F = !ax1x2 + hXIV2 + bX2Y1 + CY1Y21 2: 0, and then (6.1) becomes (6.2) EIE2 F 2 + (ac b 2 )( Xl Y2 - X2YI?' We also have (6.3) f(xi X2,Y1 V2) E 1 + E 2 :I: 2F. Hence, using (6.2) and (6.3), we obta,in 1/2 ) (f(xbvtJf(X2,Y2)) f(XI X2,YI - Y2 2': (E1E2)1/2(E1 + E 2 2F) 2: (I<J 1 Ed/ 2 (2(E 1 E2)1/2 - 2F) 2(E 1 J;2) - 2(E 1 1:;2)1/2 F 2F 2 + 2(ac b 2 )(x1Y2 - X2yJ)2 -2F(F 2 + (ac - b 2 )(XIY'l 2F 2 + 2(ac - b'l)(XIY2 X2Yt? 2 ( (ac b 2 )(XIY2 -2F 1 + F2 x2yd)1/2 . 2 ) 1/2 X2Y1) 
56 SOLUTIONS  2F'- + 2(ac b 2 )(XIY2 X2YI)2 _2p2 ( 1 + (ac- b 2 )(XIY2 - X2Yd 2 ) 2P2 (a- b 2 )(;/'IY2 - ;l;2yI)2 This cOlllIlletes tIlt; proof of (6.0). 7. Let R, ,.,', T be three rcal numb!'ts, not all thc !\<1lI1e. Give a condi- lioll which is alisll,,'rl hy tHlP >lIItJ ollly OtH' of rhe thr,".' rripl!';; { (R,S,T), (7.0) ('1', -8 + 21', R - S + 1'), (R - S + 'l','l.R S,R). Solution: We let (a,b,c) denote a.ny one of the triples in (7.0) and show that exactly one of the three triples satisfies (7.1 ) (i) a 5: b < c or (ii) a  b > c . \Ve consider six cases. Case (i): R 5: S < T. Here (a,b,c) (R,S,T) satisfies (7..l)(i) but not (7.1)(ii), while the other two triples satisfy neither (7,l)(i) nor (ii) as T < -8 + 21', -8 + 2T > 1l S + l' and R - S + T > 21l - S, 2R - S < R . Case (ii): R < T 5: S. Here (a,b,c) (1', -S + 2T.R - S + 1') satisfies (!.1 )(ii) bl1t not (7.1 )(i), while the other two triIlles satisfy neither (7.1 )(i) nor (u) as R < H, S > T and 1l - S + l' > 2R - S, 2R S < R . SOLUTIONS 57 " t. i Case (iii): S < R 5: 'T. Here (a,b,c) == (R - S + T,2R - S,ll) satisfies (7.1 )(ii) but not (7.1 )(i), while the other two triples satisfy neither (7.1)(i) nor (ii) a.<; R > S. S < l' and l' < -S+2T, -S+2T> R S+T. Case (iv): S5:T<R. Here(a,b,(')==(T.-S+2T.R ....'+nsa.li;;lie;; (7.1 )(i) but not (7,1 )(ii), whilfd,he otlll'r two tl'iples satisfy neither (7.1)(i) nor  ji) af\ R > S, S 5: T and R S + l' < 2R S, 2R - S > R . Case (v): l' 5: R < S, Here (a,b,c) == (R S + T,2R - 8,R) satisfies (7.1 )(i) but not (7.1)(ii), while the other two triples satisfy neither (7.1)(i) nor (ii) as R < S, S > l' a.nd l' > -8 + '21', -s + 21' < R S + T . Case(v-i): l' < S 5: R. Here (a,b.c):= (R,S,T) satisfies (7.1)(ii) but not (7.1)(i), while the other two triples satisfy neither (7.1)(i) nor (ii) as T > -S + 2T, -8 + 21' < ll- S + l' and R S + l' < 2R - S, 2R - S  R. 8. Let ax2 + bxy + cy2 a.nd Ax 2 + Bxy + Cy2 be two positive-definite quadratic forms, which are not proportional. Prove that tho form (8.0) (aB - bA)x 2 + 2(aC - cA)xy + (bC - cB)y2 is indefinite.  
58 SOLUTIONS Solution: As ax'}, + bxy + cy2 and Ax2 + Bx!) + Cy2 are positive-definite we have a > 0, c > 0, b 2 4ar. < 0, A > 0, C > 0, B2 - <lAC < 0 . Tb show that tIll' form {aB bil)J: 2 + 2iaC  cA):q, + (bC - rBhl is inddinite we HIIISt. "how that its disnimimmt D = 4{aC - CA)2 - 4(an - bA)(bC - cfl) i;; po!;itive. We first show that D ? O. This follows as 0,2 D (2a(aC' cA) b(aB - bA))2 - (b 2 4ar.)(aR _ bA)2 . Moreover, n > 0 unless aB bA = aC cA = 0 in wMch case a b c A B r; This does not occur as ax 2 + b;cy + cy2 and Ax2 + Bxy + Cy2 aT(' not propor- tiona.l. 9. Evaluate the limit (9.0) n n 2k lim - '" - . n_= 2 n L.J k k=1 L Solution: We show that L = 2. For n ? 3 we have n 2 k "'- 2 n L.J k k=1 n n-l 2n-k '}n L n - k - k=O SOLUTIONS ,,-nd so I nn 2 k "n Lk  k=l on-II I L 2k k=O 59 n-I J n == L 2 k 12. - k k=O r.-l 1 k ) L 2k(1+k k=O n-l 1 on-I k L 2k + L 2k(n - k )' k=O k=l == \ on-l k I t; 2 k (n - k) n-l k L 2k{12. - k) k=1 n-l k 1 '" ::; 2(11 -1) +  (k 2 - k)(n - k) 1 n-I 1 2(n 1) +(k-t)(n-k) 1 1 n-I 1 1 , +-L(-+-) 2(n - 1) '11 - 1 k=2 k - 1 n - k 1 2 n-2 1 +-L 2( n - t) n - 1 r=1 r 1 2 < + Inn. 2(n - 1) n 1 ....--L +  In n --+ 0 and so As n --+ +00, 2(n-l) (n-l) n 2 k L== lim L- n_co 2 n k k=1 n-l 1 == lim L 2 k n-+X" k=O 00 1 L 2 k k=O 2. 
60 10. (10.0) SOLUTIONS Prove that thcre does not exist a constant c  1 such that for aU pt)$itiv, integers nand m satisfying n  m. nCt/J(n)  mCd>(m), Solution: SlJ pposc then exisl s a constant c  1 such that ( 10 0 ) I ld fi all T't .losor . Po] lVe m ,e/1:el'S m and -n satisfying n > m I 1. h . '-1;'111 'f' > ,.... Thcn, w" hav': - . Je pea pflme 3 p+1 -  ( as p > 1c  4) 4 2(p- 1) > tj>(p+ 1) (as tj>(p + 1)  (p+ 1)/2, tj>(p) = p - 1) tj>(p)  (-L)" (by (10.0» .p+ 1 (1 _ 1 p+  1 c (using XC - 1  c(:c - 1), x > 0) p+] > 1- ::. p 3 > - (as p > :1c), .4 which is impossible, and no such c exists. .1.1. , Let D be a squarefree integer greater than 1 for which there exist posItIve mtegers A}, A2' B}, B 2 such that . (11.0) { D = A + B = A + B, (AI,B I ) =f. (A 2 ,B2)' J>rove that neither 2D(D + AIA2 + B I B2) SOLUTIONS ,r" "I.. nor 2D(D + AIA2 - B I B 2 ) is the square of an integer. Solution: Suppose that 2D(D + AIA2 + fB 1 1l2) X 2 , where X is an integer anti f =- :i: 1. We consider two caSE'S aCt:orrlinl!; as {) is ....1<1 or even. If P is odd, as it is squarefr<'c, 2D divides X, say X = 'lDl.', when' (J is a.n integer. and so (11.1) D + AIAz + I:;B l 1l 2 = 2DU 2 . Next we have 2D(D AIA2 - I:;B I B 2 ) = 2D (D 2 - (AIA2 + fB I B 2 ?) D + AIA2 + fB I B 2 2D(AIB2 - fA2Bd 2 =: D + AIA2 + fB I B 2 ' that is (11.2) 2D(D AIA2 - I:;B 1 B 2 ) = ( AIB2  fA 2 B 1 r . Since the left side of (11.2) is an integer and the right side is the square of a rational number, the right side of (11.2) must in fact be the square of all integer. Hence, there is an integer Z such that (11.3) 2D(D - AIA2 I:;B 1 B 2 ) Z2 , (11.4 ) AIB2 - fA2BI U Z . From (11.3), as above, we see that 2D divides Z, so there exists V such that Z 2DV. Then (11.3) and (11.4) become (11.5) D - AIA2 - fB 1 B 2 = 2DV 2 , 
62 SOLUTIONS (11.6) AlB2 f..4 2 B 1 = 2DIJV . Adding (11.1) a.nd (11.5) we obtain 2D =: 2DU 2 +2DF2, so that [[2+ V2 = 1, giving (11.7) (U,v) (:1:1,0) or (0,:1:1). Now from (11.1), (11.5) and (11.6), we have { 11/12 + (BIHz == ?(U. 2 -- V 2 ), -f H 1 .4 2 + Al U 2 = 2DIJ V . Solving thesc equations for A 2 and R 2 gives (11.8) Ih =: (U 2 - VZ)Al 2dlFB 1 , B 2 =: ZUV A 1 + f(lJ2 V2)Bl' Using the valus for (ll, IT) given in (11.7), we obtain from (11.8) (A:;;,B 2 ) = :1:( AI, fBd, which is clearly impossible .,.; Ah ..1.2, B1, B 2 ar€ positive and (AbBd f (A 2 ,B 2 ). The case when D is even can be treated similarly. 12. Let Q and R denote the fields of rational and real numbers respectively. Let K and L be t.he smallest subfields of R which wntain both Q and the real numbers / 1985 + 31 V1985 and / 3970 + 64 V1985 , respectively. Prove that K =: L. Solution: We set (12.1) { 0+ =: .j1985 + 31.Ji985  58.018, I.L /19 85 - 31 v'1985  24.573, (12.2) { ,£'J+ /.3970 + 64.Ji985  82.591, =: )3 970 - 61 v'1985  33.445 . SOLUTIONS 63 [1. is easy to check that (12.3) 32 V1985 , fhfL =: 62Vi 98& , Oi+I.L ( 12.4) { (0:+ + I.L? 3970 + 64 -J ' (a+ - I.L)2 =: 397() - My!1985 , from whidl Wf' obtain > (1Z.5) (1'++(L I'J+_ 0t-(.L=P_ W . t ' Q ( 'II 'II ) for the smallest subficld of R contaillin both Q <1[111 flIng tl,...'tn the real numbers 11,. . . ,1n, we have Q(Oi+) Q(o+,at) Q( Oi+, V1985) (by (12.1») Q(O+,Oi_) (by (12.3)) '2 Q(o+ + Oi_) :::= Q(/h) (by (12.5)) = Q(fh,{3) QU:J+, V1985 ) (by (12.2») = Q(!3+,{3-) (by (12.3)) '2 Q(/h + (L) Q(o+), (by (12.5)) so that K Q(o+) = Q(,g+) L. 13. Let k a.nd l be positive integers such tha.t GCD(k,5) = GCD(l,5) GCD(k,l):::= 1 and II _k 2 + 3kl- z2 = F 2 , where GC D(F,5) 1. Prove that the pair of equations { k = x 2 + y2, (13.0) l = x 2 + 2xy + 2 y 2, ' ....4 
64 SOLUTIONS has exactly two solutions in integers :1: aud y. Solution: We have 1'"2 = 4k 2 + 8kl + 41 2 == 4(k + l)2 (mod 5) so that F :f:2(k + I) ( d 5 ) R 1 1110 .'. Pp adTlI1; F by - 1", if 1)1'((':-:8ar , \' , we J ' 1 " ,, ' RUpp08e Lllat '  (l:1.1 ) p  '2(h + l) (mod '» Then we have (13.'2) { 4k - 1 - 2P --3k + 21 - F k + 1 + 2F o (mod 5), o (mod 5), o (mod 5), and we may define integers R, H, T by (13.3) { 5R = 58 = 5T = 4k - 1 - 2} -3k + 2l F, k + 1 + 2F . Purther, we llave 25(RT - 8 2 ) = (4k - 1- 2F)(k + 1 + 2F) (-3k + 2l F)2 = -5k 2 + 15kl 51 2 _ 5F 2 0, so that (13.'1) RT = !p . We now treat t1lree cases: (i) R = S 0, (ii) R  0, S = 0, (Hi) S  0 . Case (i): R = S = O. From (13.3) we have 'lk - 1 - 2F = 0 . d -3k 21 - F = 0, so that k = F 1 - 2F B k I ' . . .' an ,+ ,- . ut , are positIve COprIme integers, so SOLUTIONS 65 F = ], k = J, 1 2. In this case (13.0) has two solutions (x, y) ::1:(0,1). Case (ii): R  O,S O. From (13.4) we have T = 0, and so from (13.3) we obtain { -3k + 2l - F  0 " k + 1+ 21" = O. so that k = 1 = -F. As k, I are positive coprime integers we have P -1, k = 1= L In this rase (U.O) ha" two "oll1l.ions (:;::,y) ::1:(1.0). Case{iii): S  O. From (13.4) we ha.ve RT> O. If R < 0 then P < () and Wt' navf' k = H + T < 0, contradktillJ:'; k 2': 1. Ilenc!' U and Tar" l'0sitiw integers. !\i ext, ohberve tha,t (-1k 2F)(4k 1 + 2F) (1k .._lj2 - 41"2 = 5(l- 2k)2 , so that (13,5) R(4k 1 + 2F) (1- 2k)2 . Clearly, we have 'lk 1 + 2F  0, otherwise 5R -:c -4F and so 5 I P, con- t.radicting GCn(F,5) 1. Hence we may define nonnegative integers tL,b,c by (13.6) 2 a II R, 2 b II 4k - 1 + 2} 2 c III - 2k . We have from (.13.5) and (13.6) (13.7) a + b 2c and (13.8) ( l- 2k ) 2 2<.' ' 2 a where R 2 a ' are odd positive integers. Suppose that 2 C Gcn ( R 4k-l + 2F ) 1 2 a ' 2 b >. . 
66 SOLUTIONS Then there is an odd prime p which divides R/2° and ('1k -1 + 2F)/2b, and thus p divides "k 1 - 2f 4l' -1 + 2F, and 1 - 2k, giviIlg successively p ISk 2l, pi 4k l, p 12k, pi k, p' 1 , cont,ra.di<-.tiIlg fie D( /':, l) 1. lIence we have (T3})) (in lJ ( !!. 'i + }= ) = I . .2°' 2 b , Fm!l1 (I :U) a1l(1 (l;i.9) we 8<>e t 11'1,(. (13.10) .!i =X2 , 2 a for some integer X. '!\ext we show that a is even. This is clear if a = 0 so we may SUppose that a  1. Thus 21 R and so 1 is even. As GCDU",l) = 1 we have k odd. Then, taking -k 2 + 3kl 1 2 = F2 successively modulo 2,4 and 8, we get (13.11 ) F (mod 2) , (13.12) 1==2 (mod 4) , (1:!.13) 1 == 2k (mod 8) , Thus we hav(' 4/' 1:f: 2F 0 (mod .1) and so a  2,b  2. Also we have 2 mU, (a,b) I (4k -l + 2F) - (4k - 1 - 2F) 4F, and so as F is odd we have min(a, b) :::; 2. If a :::; b then we have a:::; 2, which implies that a 2. If b < a th<m b :::; 2, which implies that b 2, a = 2c _ 2. In both cases a is even as asserted. Setting a 2d, Xo = 2 d X, we have R = x. Then from (J:J.4) We deduce that '1' yiS, S :l:xoYo. Changing the sign of Xo if I1ccessary we may suppose that S = XoYo. Thus we obtain x + YiS :::: R + '1' :::: (5R + 5'1')/5 :::: (4k-1- 2F+ l: + 1 + 2F)/5 = l' and x5 +2xoYo + 2Y5:::: R+2S +2'1':::: I, so that (xo, Yo) is a solution of (13.0). J: , ! SOLUTIONS 67 I 1. , f ( 130 ) Then using (13.0) we have :\ow let (x,y) be any so u IOn 0 ,. . F 2 =-k 2 +3kl l2 (x 2 +xy_y2?, th - t (WI ' t! 1 F chosen t.o sath-.fy (13.1» .'-\0 d. j x2 + xy y2 = :1: F . ( 13.14) Solving (13.0) and (1;\.14) j{)'r :2, xy. y2, we get { £);,.2 .lh 1-1 2F, 5xy -3k + 21 :f: t', (l:tID) 5y2 = k + 1 =F 2F . As F == 2(k+ l) tc 0 (mod 5) the lower signs must hold in (13,15), and so { x2 :::: (4k-l-2F)/5, xy = (-3k + 21 P)/5, (13.16) y2 :::: (k + 1 + 2F)/5 . .. " lution of (13.0) we must have that (13.16) holds Since tillS IS true lor any so . h 1. with x, y replaced by xo, Yo respectively. ThIs means 1. a x 2 x, xy XoYo, y2 Y5, giving (X,y) (xo, Yo), or (-xo,-Yo) , . h t (13 0 ) has exactly two integral solutions. and provmg 1. a ' 14. (14.0) Let T and 8 be non-zero integers. Prove that the equation 2 2 ) 2 1 (r 2 - S2)x 2 - 4T.XY -(r - s y has no solutions in integers x and y. 
68 SOLUTIONS Solution: We SUppose that x and yare integers satisfying (14.0). Factoring the left side of (14.0), we obtain (14.1) «r - s)x (, + s)y)«r + 8)X + (, s)y) == 1. As each factor on tIle left side of (14.1) is an integer, we s that (14.2) { (r - s)x - (, + s)y == f, (,ts)x+(r-s)y f, where f =:; :1:1. Solving (H.2) for x and y, we ohtitin (14.3) ,f -Sf x-_ y_ -r 2 +s 2 ' - + Hence we have (x 2 + y2)(r 2 + 8 2 ) == 1, so that ,2 + s2 1, that is ("s) == (:1:1,0) or (0,:1:1), which is impassible as rand s are botll non-zero, thus showing that (14.0) has no integral solutions. 15. Evaluate the integral I L 1 1nXln(1_ x) dx . (15.0) Solution: The function In x In(l- x) is continuous for ° < x < 1, but is not defined at x == ° and x == 1, so that (15.1) 1== lim £-+0+ S-+o+ 1 1 - S  Inxln(l-x)dx. For x satisfying (15.2) O<f.:$x.:$1-6<1, SOLUTIONS '\ ... f J' .II; \\: .  r , ' ,. 69 11,IId n a positive integer, we have 00 xl. -In(l x) = :L k 1.=1 n 1. 00 xk-(n+1) :L  + x n +1 :L k=1 k k=n+l "nd so Iln(l n xk! x)+ k' 1:=1 00 xn+l 11.+1+ k=O n+l '" "xk .:$ L.- n+ 11.=0 xn+l (n + 1)( 1 - x) . = Thus we have 111-S Inx(ln(l (15.3) n xl. ) I x) + {; k dx -I-S xn+l 1 1 (-lnx)- ) dx. .:$ (n+ f (l-x Now, for y  1, we have l Ydt< 1 11dt y 1. (15.4) O:::;lny= 1 T- 1 Taking y == l/x in (15.4), we have ( 1 ) <1_1=1 x (15.5) 0 :::; In x = In ;; - x x . ( 5 5 ) ' ( 153) we deduce Using the inequality 1. I l n 1 1 - S ' ..;;-. 1 1 1 - S xklnxdx l  Inxln(l- x)dx +  k  1 1 1 - S :::; - xndx n + 1 f  (1 xndx < n + 1 10 1 ( n + 1)2 ' 
70 SOLUTIONS SOLUTIONS 71 a.nd letting n ---> 00, we obtain We llext show thal (15.6) j l-6 , In x In(l x)dx 'X> 1 1-6 k j xklna:dx. k=l f (15.10) lim (InE:)A(E:) := 0, (';-+0+ As ( a:k+lln:r. xk+l d.r ' + 1 (/.: + :r. k In x, (15.11) lim (In(l- o»A(I- 0):= 0, 6-->0+ hy the flludam('ntal t1worem of "akulu.-;, W'. ha,\'f' (15.12) limB(c)=O, (---)00-1 1 1-'" , xk In xdx ( 1- 0)k+1 (k+1)2 Ck+t (15.13) 11"2 lim B(l - 0) = 2 - _ 6 ' 6-->0+ so that by (15.6) so that (15.1) and (15.7) give j I-6 f Inxln(l- x)dx 00 InE:L k=l "" c k + 1 - E k(k+ 1)2 00 (1 0)k+1 00 (l 0)k+1 0) k=1 k(k + 1) + t; k( + 1)2 ' (15.14) 11"2 [=2--, 6 In(1 as asserted in the HINTS. Before proving (15.10)-(15.13) we show that that is (15.15) lim (Inc)ln(l- c):= O. (':-+0+ (15.7) 11-61nxln(l_ x)dx (InE:).l1(c) B(f) (In(l o))A(l-o)+B(1 where, for 0 < y < 1, .t1(y) and B(y) are defined by For 0 < I' < 1 we have In(1 - €) 1'2  { f+-+-+'" 2 3 > 1', < C+f2++... f 1- I' ' 0) , (15.8) A,(y) 0.0 k+J t; k(,+ 1) ' so that -€In c < (In c)ln(l- 1') < dne 1 I' (15.9) B(y) ,'.0 yk+] E k(k + 1)2 ' from which (15.15) follows, as (15.16) lim €ln I' := 0 . ,-->0+ 
72 SOLUTIONS SOLUTIONS 73 Now for 0 < t < 1 we have A(t) O k co k+1 (L L t k=1 k k=1 + 1 -dull f) + 111(1 to) + f (1- f)ln(1-/) + <:, plUving (15.13), and completing the proof of (15.H), 16. Solve the recurrence relation so thai (16.0 ) t ( n ) 0,( k) = !.:, 11 1, 2, .. .. k=l k 11. + 1 Jim (In(),,1(c) O. (. -0+ 'nis proves (lr,-,o). Kext we have, by Abel's theoreID, lim A ( 1 6 ) 6-+0+ . = (1 '- bY+! 00 1 hm L - " 6-+0+ k=1 k(k + 1) - f;;;;, k(k + 1) 1 , Solution: W('m,1kt'lh"illdlldiwhypot!i""i:;thal a(,I) ( 1)"H/(1I1 I) for all positive integers n satisfying 1 :$; 11 :$; m. This hypothesi:> is true for 'In 1 as a(l) 1/2. Now, by (16.0) and the inductive hypothesis, we have ( ) _ m + 1  ( m + 1 ) (-I)k+ 1 am+l - -L... m + 2 k=1 k k + 1 Thus we must show that so that 61!..W+ (In(1- 6)A(1 - 8) = In 1 0 . This proves (15.11). Also we have IB(f)1 :$; f L k=1 f ( m + 1 ) (-1)k+1 == m+ 1- (_l)m 1=1 k k + 1 m + 2 00 so that or equivalently lim B(t) == 0, (-+o+ proving (15.12). Finally, by Abel's theorem, we have . 00 / 1 _ C ) kH lim B(l 8) == Inn" \ " 6....0+ 6-+0+ f;;;;, k( k + 1)2 ">0 1 t; k(k + 1)2 'x- 1 I .t;( k(k + 1) - (k + 1)2 ) ( 11'2 = 1- 6-1) 11'2 = 2- 6 '  ( m+1 ) m+ 1 k=1 k k + 1 = m + 2 By the binomial theorem, we have for any real number x (16.1 ) m+l ( m + 1 ) k O+x)m+1 L k X. k=(J Integrating (16.1) with respect to x, we obtain (1 + x )m+2 m+1 ( m + 1 ) xk+1 _ -2- (16.2) m + 2 t;) k k + 1 + m + 2 . :raking x = -1 in (16,2) we have I: ( m+ 1 ) k=O k 1 =- m+2 ' 
74 SOLUTIONS SOLUTIONS 75 and so This estahlishes the existence of a pair of integers as required. ( m + 1 ) (_I)kH = 1- 1 = k==1 k k + 1 m + 2 m 1 m+2 18. Let as required. The result now follow!> by the principle of mathematical induc- tion. 111 Un"" 4n+l + 4n+a - 2n+ 2' n=O,l,... 17. Let nand k be positive integers. Let p he a prime such that Do('s the infinite s('ris I:=() 11." conv('rgp, and if so, what is it>; sum? p> (n 2 + n + kj2 + k . Solution: Let s( N) I:==() an, N = 0,1,. . . We have Prove that the sequence (17.0) n2, n 2 + 1, n 2 + 2, . . ., n 2 + 1 , s(N) Nil 1 E Cn+l + 4n+ 3 2n+2 ) N ( 1 1 1 1 1 1 ) .- n=() -i n + 1 - .1n + 2 + 4n + 3 - '1n + 4 + 4n + 2 -- 4n +4 4NH ( -1 )m-l 1 2N+2 L m +2 L m=1 m=1 where 1 that (n 2 + n + k)2 n 2 + k, contains a pair of integers (m, m + k) such ( ; ) = ( m; k ) 1. m Solution: As nand k are positive integers and p > (n 2 + n + kp + k, 2 none of the integers of the sequence (17.0) is divisible by p. If ( n:k ) 1 we can take (rn,m+k) (n2,n2+k).If(J 2+k ) 1 we can take (m, m + k) = «n + 1)2, (n + 1)2 + k). Finally, if P ( n 2 : k ) ( n + 2 + k ) -1, we can take (m,m + k) = «n 2 + n+ k)2,(n 2 + n + k)2 + k), as ( n 2 + n; k? + k ) ( n 2 + k)«: + 1)2 + k» ) ( n2:k )( n+2+k ) (-1)(-1) 1. Letting N . (X; we 11ave lim s(N) N-+-;'JC) 00 (_l)m-l L + m=1 m 3 00 (_I)m-l 2 L m ",=1 3 2 1n2 , 1 00 2 L 1n=1 m so that I:=() an converges with sum  In 2. 19. Let 11.},... ,G.", be m ( 2) real numbers. Set An 11.1 + 11.2 + ... + an, n = 1,2,..., m . 
76 SOLUTIONS SOLUTIONS 77 Prove that (19.0) Solution: and so (19.1) But as we have that is (19.2) Using (19.2) iD (19.1) we obtain f ( n r  12 f a . n2 n=l For 11. 1,2,...,m we have tha t i ( n r (a n + 11. anf 2 ( An ) 2 < 2a +2 --an n 11. 2 ( An ) 2 An 4a n +2 - -4a n -, 11. 11. (19.:-1 ) f( n r  n=l m A 2 m A2 +2;( nn ) - 2E n(nl) m A2 2" n !;t 11. 2 (11.+ 1) , m 4La n=l m 4La + n=l £:(1 n=l 2 ( n r4f>. n=l 11.+ The inequality (19.0) now follows from (19.3) by noting that 1 n =: 1, and 1 2': l for n 2': 2.  ( n r 4a + 2 ( n r -4an n . = 0 when 20. Evaluate the sum n (n) S = L ( 2nl ) k=O k -2anAn = -(A A;_1) - a  -(A - A_l) for all positive integers 11.. Solution: We have -2 n=l 11. m ( A2 _ A2 ) n n-l  n=l m-l A; -L n=l n(n+ 1) A2  m so that m A m A2 -2 L an < - L n . n=l n - n=I11.(n+l) () ek') (k:l) (k 2 ;1 ) 11.! (211.- k)! 11.! (211.- k -I)! (11.-k)!211.! (n-k-l)!2n! n!(2n-l-k)! ( 2n-k) _ (n-k» ) (11. - k)! (211. - I)! 211. 211. 1 (n) = - 2 e n ;l) , S = 2 t ( G) _ (k:1) ) k=o ek') (kl) 
78 SOLUTIONS SOL UTIONS 79 = 2 ( () e;) 2. (nl) ) (n;\) values of t satisfying (21.3). Hence we have (21.5) N(k) = [] [ 7] - >'(b,g) + 1 , and so 21. Iet II. and h be coprime positive int...!!;ers. For k a positiv(' intep;er, Id IV (k) denote the 111l1llber of integral solution" \.0 the equation IN(k) h _ [L I < 1 + 1 + 1 + 1 4, a b- giving, by (21.1), (;ll.n) It./: + by:::: h, ,r  0, !I?!J . ( ;ll.{j) I N(k) I I ,4 ---- -- -. < - k ab'-'k" 1.. lim N(k) . k--++oo k Letting k -+ +f)Q in (21.6), we obtain L = l/ab. Evaluate Ute limit 22. Let a, d and r be positive integers. For k = 0,1, ... set Solution: As a and b are coprime there are integers g and h SUell tbat (22.0) 1 Uk uk(a,d,r) = (a + kd)(a + (k + l)d)... (a + (k + r)d) . (21.1) ag+bh=k. Evaluate the sum n S = L Uk , k=O Then all solutions of ax ,I, by k are given by (21.2) where 1/. is a positive integer. x 9 + bt, y = h - at, t 0, :1:1, :1:2, .. . Thus the solutions of (21.0) are given by (21.2) for tbose integral values of t satisfying Solution: Fork -1,0,1,... we set (21.3) h g - > t > -- . 11.- - b (22.1 ) Vk = vk(a,d,r) 1 (a t (k + 1 )d) . . . (a + (k + r )d)rd ' so tbat Set ( 21 4 ) >. ( b ) = { 0, if b divides 9 , . ,g 1, if b does not divide 9 , Tben there are [] - [ 7 ] - A(b,g) + 1 Vk - Vk+l 1 ( 1 = (at(k+2)d)...(a+(ktr)dird (a+(k+1)d) - (a + (k + r + 1 )d» ) 1 = (at(k+1)d)(a+(k+2)d)"'(a+(k+r)d)(a+(k+r+1)d) ' 
80 SOLUTIONS SOLUTIONS 81 that is Vk - VkH Uk+l' Hence we bave n n-] 'n-l S = L Uk = L Uk+I = L (VI, - Vk+l) LI - 'I'" , k=O k=--.] k=-] Solution: By the Cauchy-Scbwarz incquality we have that is -' 1 ( 1 1 ) .. 'I'd a(a+,/)...(tI+('I'-nd) - (a+(n+l)d)--.(+c;+ r)d) . (24.1 ) C" ) 2 .a" 1ft 1 ) 2 1ft m 1 (a"vn y'n S na  . Xpxt, we have 23. Let xJ,... ,x" be n (> 1) rea.l numbers. Set Xij=Xi Xj (ISi<jsn). Let F be a real-valued function of the n(n - 1)/2 variables :rij such tbat the inequality  1 < 1+ {1ft d,x <1+ {"'!!!. L.. n - J] X - JI..jX n:;;:;.::l 1 + (2.jm - 2) 2..jffi - 1 < 2,fm . We obtain (24.0) by using the lattN' inequality in (24.1). (23.0) n F(Xll,XI2,...,X n _ln) S Lxi k=l 25. Prove that therr exist inJinitcly many positive integers which are not expressible in the form n 2 + p, where n is a positive integer and p is a prime. holds for all a], . . ., Xn. Prove that equality cannot hold in (23.0) if Lk=l Xk -10. Solution: Set Ai = (Xl + ... + x,,)/n, and replace each :r.i by Xi - AI in (23.0). Then (23.0) gives the stronger inequality .!. ( t Xk ) 2 . n k=] Solution: We show that the integers (3m + 2)2 , m 1,2,..., cannot be expressed in the form n 2 + p, where n ;::>: 1 and p is a prime. For suppose that " n F(xJ1, XI2,... ,X n -] n) S L(:r.k - M)2 L x k=I k=] lIellce if Xl, . . . ,.'l'n are chosen so that Lk=l x k -I 0, equality cannot hold in (23.0). {3m + 2? n 2 + p, where n ;::>: 1 and p is a prime, then (25.1 ) p (3m+2 n)(3m+2+n). Since p is a prime and 0 < 3m + 2 - n < 3m + 2 + n, we must have 24. Let a],... ,am be m (;::>: 1) real numbers which are such that L::'=I an -I O. Prove the inequality (25.2) 3m + 2 n = 1, 3m + 2 + n p . (24.0) ( fna ) / ( fa n ) 2 > 2 m ' ,,=1 n=1 V Solving (25.2) for Tn and n we get m (p - 3)/6, n (p 1)/2, 
82 SOLUTIONS SOLUTIONS 83 so that p 3(2m+ 1). As P is prime. we must have rn 0, which contradicts m1. 26. Evaluate the infinite series 27. Lei 111",.,]in d('nol. n the polynomial of degree n givcn by 1) distinct inhJ';ers and let In(:r:) be s f arctan ( 22 ) . n=1 n /,,(:1:) = (x - PI )(:r: 1'2 L.. (x - p,,). Prov{' that the polynomial Solution: For 11.  1 We have !/n(:r:)::: U:"'t:r:»2 + I arctan ( ! ) - arctan (  ) n 11.+2 arctan (  - nr ) 1 + n(n+2) arctan ( 11.: 1)2 ) , cannot be exprcssed as the product of two non-.consta,nt polynomials with integral coefficients. Solution: Suppose that Yn(;r;) <:an be expressed as the product of two non- c.onstant polynomials with integral coeHicients, say 80 that for N  2 we have t arctan ( 2 2 ) n=2 n (27.1 ) 9n(X) h(x)k(x) . 3=1 arctan ( 2 ,, ) n=1 . n + 1)- I:I ( arctan ( ! ) - arctan (  )) n=1 n 11. + 2 ::: arctan(l) + arctan () - arctan (  ) arctan (  ) . N+l Neither hex) nor k(x) has a rea.l root as 9,,(X) > 0 for aJlrcaJ. x. TIms, lLeither hex) nor k(x) <:an change sign as x takes on all real values, and we may suppose that (27.2) h(x»O, k(x»O, forallrealx. tetting N -7 00 we get E arctan ( :2 ) arctan( 1 ) + arctan G) =  + arctan G) Since gn(Pi) 1, i 1,2,...,11., we have h(Pi)::: k(p.) 1, i::: 1,2,...,11.. If the degree of either h( x) or h(:f.) were less than n, then the polynomial would have to be identically 1, which is not the case as h( x) and k( x) are nOll-constant polynomials. Hence both h( x) a.nd h( x) have degree 11., and (27.3) { hex) 1 +a(x - pd'''(x Pn), k(x)=l+b(x pIJ...(x Pn), S :::  + arcta.n(2) + arctan () ::: 3 4 7£' . for integers a and b. Thus we have (27.4) (x - pd(:/': - P'l)2 ...(:r: - Pn)2 + 1 =1+(a+b)(x pd...(:r:-Pn)+ab(x-Pl)2...(x- p,,)2. and so 
84 SOLUTIONS Equating coefficients of x 2n and x" in (27.4) we obtain (27.5) { ab 1, a+b =0. Thus we have a contradiction as no integers satisfy (27.5). 28. Two people, A and B, playa game in which the probability that A. wins is p, th( probability that B wills is q, and tilt.' probability of a draw is r. At the beginning, A has m dollars and B has n dollars. At the end of each game the winner takes a dollar from the loser. If A and B agree to play until one of them loses all his/her money, what is the probabilty of A winning all the money? Solution: Let p(k), k = 0,1,. .. , denote the probability that A wins when he/she has k dollars. Clearly, we have (28.1) p(O) = 0, p(rn + n) 1. We want to determine p(m). Consider A's chances of winning when he/she has k + 1 dollars. If A wins the next game, A's probability of ultimately winning is ap(k + 2). If A loses the next game however, A's probability of ultimately winning is bp(k), while if the game is drawn, A's probability of ultimately winning is cp(k + 1). Hence we have p(k + 1) = ap(k + 2) + bp(k) + cp(k + 1) . As a + b + c = 1 we deduce that ap(k+2) (a+b)p(k+l)+bp(k)=O. Soving this difference equation, we obtain P( k ) { A + Bk , if a = b , . = A + B(b/a)k ,if a l' b , SOLUTIONS 85 where A and B are constants to be determined. Using (28.1) we obtain { A= 0, A= -B B=I/(rn+n) ,ifa:::b, 1/(1 (b/ar+ n ) , if a =F b, so that { m/(m+n) ,if a b, p(m) = (1 _ (bitt)"'') I (1- (blar n + n ) ,if a =F b. 29 Let f(x) be a. monic polynomial of degree n  1 with complex co- efficients. Let xl:'" ,X n denote the n complex roots of f(x). The discriminant D(f) of the polynomial f( x) is the complex number (29.0) D(f) II (x. - Xj)2. l:<;;i<j:<;;n Express the discriminant of f(x 2 ) in terms of D(f) . Solution: As Xl,' . . , X" a.re the n roots of f( x), the 2n roots of f( x 2 ) are Y1 ,fiil, Y2 ..jX2,... ,v" =,,;x;; ,Yn+l = -.;xi, ... ,Y2n = -,,;x;;. Hence, the discriminant of f(3: 2 ) is II (Yi - Yj)2 1 :<;;i<j:S2n II (Yi - Yj? II (Yi - Yj)2 l:<;;i<j:<;;n l:<;;i:<;;n<i:5;2" II (Yi - y;)2 n«<;:<;;2n (.[Xi - ,jXj)2 II (VXi + .Jx;-n? 1$i$n<;2n II l:<;;i<i.:<;;n II (- .JXi-n + .JX;-n ? n< i<j :$2n II (.[Xi -,;x;l II (VXi + ..jXj)2 19<;$" : 
86 SOLUTIONS II ( Vii I9<.iSn II ( -Vii + vfXj)2 l$i<iSn vfXj)4 II (.JXi + vijl ISi<iSn II (2y'iJ2 lSiSn n n (a:, .l:J)4 2 2n n Xi I:$i<.iSn i=.l 2211.( -l)nJ(O)(l)(J)) . , O. ,.Prove tht for each positive integer n there exists a drcle in the xy-p ane wInch contams exactly n lattice points. Solution: L t P b h t;> I e. e.t e point (v 2 ,1/3). First, we show that two different attIce pomts R - ( X ) d 8 ( d' t f ' - 1, Yl an X2, Y2) must be at different 15 a ld llc i es rom P. For if Rand 8 were at qual distances from P then we wou lave ' (Xl - V2? + (VI - )2 == (X2 - V2)2 + (Y2 _ )2 , so that (30.1 ) 2(X2-:l:I ) V2 2 2 2 2 2 x2 + Y2- Xl YI + 3(Yl 112). As V2 is irrational, from (30.1) we see that x - x - 0 . d h 2 2 (Yl - Y2) = 0, that is 1 '2 - ,an ellce Y2 - Yl + (Y2 Vl)(Y2 + Yl 2/3) O. Since Y1 and Y2 are integers, 'we have Y2 + Y - 2/3 t= 0 . d contrary to the fact that 1l and 8 are a:;slJme distinct.. ' an so Yl = Y2, SOLUTIOl.'\S 87 Now let n be an arbitrary natural number. Let C be a circle with centre P and radius large enough so that C contains more than n lattice points. Clearly C contains a finite numher m (> n) of lattice points. As the distances from P to the lattice points are all different, we may arrange the lattin' points inside C in a sequence PI, 1'2, . .. 'Pm' according to their increasing: distances from P. Clearly, the circle C n with centre P, passing through Pn+1' t:ontains f>xae1.Jy u. Ja,ttifE-' points. 31. 1,('1, f1 1)(' a /!,ivf'1I 1I{)1J-IH',a.l.iv(' int.I'W'f. Ikt"l'lilinp I.h.' numlJf'f 8(n) of solutioHs of the equation (31.0) x+2v+2z=n in non-negative integers x,y,z. Solution: We have for It I < 1 0(, L 8(n)t n (1 + t + t 2 +... )(1 + t 2 + t 4 +... f' 1'1=0 = 1 (1 - t)3(1 + t)2 3/16 1/4 1/4 3/16 1/8 It + (] t)2 + (1 - t)3 + 1+t + (1 + t)2 3 ov 1 ov = 16 L t n + 4 L(n+ l)t n n=O n=O +! I: (n + l)(n+ 2) tn + 3 I:(_1)ntn 4 n=O 2 16 n=O 1 (X) +8' L( -It(n + l)t n n=O 1 "" 16 L(3 + 4(n + 1) + 2(n + l)(n + 2) + 3( -It n=O 
88 SDLUTIONS +21-1)n(n+ l))t n , giving S(n)  { n(n+ 6) -+1 , if n is even, ,Cn+ 1)(n+3) , if n is odd. R 32. Let n be a fixed integer>') D . are bounded for 0 < ' . - -. etermme all functioThs f(x). which x < a, and WhiCb satisfy the functional e(!Iation . (32.0) f(x)==: :2 (f(;) +f( x:a ) +...+f C+(:-l)a )). Solution: L t f( ) b e ;c e.a bounded function which satisfies (32.0 ) for 0 . a. As f( x) IS bounded on ( 0 a ) th ' . . ' . < x < Ii. such that . ere eXIsts a pOSItIve constant (32.1 ) If(x)1 < K, O<x<a. l'br s - 0 1 - , ,..., n - 1 we have 0< :r. so, < a, ifO<x<a, n so that by (32.1) we obtain I 'f ( X + n sa ) I < K , o .s; s .s; n - 1, 0 < x < a . Then, for 0 < x < a, we have from (32.0), 1 Jf(x)1 < 2([( + K +... + K ) n , SOLUTIONS 89 that is 1ft x)\ < J{ In. Repeatinj!; the argument with the bound J( replaced by Kin, we obtain If(x)1 < Kln 2 , O<x<a. Continuinl!: in this way Wf' get (32.2) 'f(x)1 < Kin', O<x<a, for I = 0,1,.,. , ;Hd letting 1 -,. 00 in (3:.!.2) !!;ives !(;c) 0 for () <: :r < a. 33. Let I denotl the dosed intervdJ [u,b], a < b. Two functions f( x), g( x) are said to be complctely different on I if f( x) # !I( x) for all x in L Let q(x) and 1'(X) be functions defined on I such that the diffrential (.quation dy - = y2 + q(:r)y + t'(x) dx has tbrel} solutions VI(X), V2(X), V3(X) which are pairwise completely different on I. [f z(x) is a fourtb solution such tbat the pairs of functions z(x), y,(x) a.re completely different for i 1,2,3, prove that there exists a constant [( (# 0,1) such tlta.t (33.0)  _ YI ([( Y2 - Y:3) + (1 J()Y2V3 If( - l)YI + (Y2 - KY3) Solution: Y4 are pairwise completely different 011 I, the As VI. Y2, Y3, Z function (33.1) f(x) (YI - Y2 )(Y3 Y4) (YI - Y:d(Y2 - Y4) is well-defined on I. Also, as Y1:Y2,Y3,Y4 are differentiable functions on J, f( x) is differentiable there alld its derivative is given by l(x) = g(x) (VI - Y3)2(Y2 V4)2' 
90 SOLUTIONS where g(x) (Y - Y)(YI- Y3)(Y2 - Y4)(Y3 - Y4) (YI -- Y2)(Y .- !IJ)(Y J/4)(YJ .'1.d (YI - Y2)(YI Y3)(Y -- Y;I)(Y:J - Y4) + (!II - YZ)(YI 1/:I)(Y2 !l4)(Y(, Y'l) ,\" :If. Z Y.., + IIY. r, -' I, :J,:1. 1 , we have g(x) ((YI + Y2 + q) (YI + !/3 + q) - (Y2 + 1/4 + q) + (Y3 + Y4 + q)) (YI - Y2 )(UI Y3 )(;112 - Y4 )(113 11,1\, that is g( x) = 0, and so /'( J:) := 0, showing thltt I( x) = J( on I for some COIlstant ](. Finally, (33.0) is obtain(.-'d by solving; (33.1) for z Y4. J( i= 0,1 as ;; i= Y3,!l1 respectively. 34. It. an, 1). 2, ;, , .., denote the number of ways the I Ho duct b2'" b n can be bracketed so that only two of the bi are multiplied together at anyone time. For example, a2 = 1 since b I b 2 can only be bracketed as (bJb z ), whereas a3 = 2 as b l b 2 b 3 can be bracketed in two ways, nanlely, (bI(bzb:,» and «b J b 2 )b,,). Obtain a formula for an. Solution: \Ve set al =- 1. The number of ways of bracketing b l b 2 .. . b n +1 is n I:: N(l,i) N(i + 1, n + 1), i=I where N (i, j) denotes the number of ways of b1'lcketiIlg bibi+1 . . . bi' if i < j, and N(i,j) l,ifi=j. Then (34.1) a n +l = alan + a2 a n_J + '" + a n -Ia2 + anal, n =- 1,2,... . SOLUTIONS 91 Set ( 34.2) 00 A(x) = I::anx n . n=l From (34.1) and (3-1.2) we obtain A(x)2 = (u,.i) (a)xj) ('::-.0 '.1 I::L: n=l I,J;;;;1 '+3=1'1-+1 (tiU,)X'+) '" a'a Xi+i u ') i,i=l 00 ) n+1 I::(ala n + aZUn-1 + ... + anal x n=l 00 I:: a n +1 Xn + 1 = A(x) - x , n=l that is (34.3) A(xi - A(x) + x = O. Solving the quadratif equation (34.3) for A(x), we obtain A(x) (I::1: Vl-4x )/2. As A(O) (34.4) o we must have A.(x) = (1 V1- 4x) /2 . Hy the binomial theorem we have (34.5) VI - 4x :=  e2)< -4Y'x n , so that, from (34.4) and (34.5), we obtain (34.6) A(x) -  e2)(-4Y'Xn . 
Equating coefficients of x n (n  2) in (34.6), we obtain an = -e2)(-1r22n (_1)n-I22n-I e2) (_l)'H22n-I( _1)n- I 1.3.5... (2n 3) 2 n n! { get) =tt.an 2 t-3tant+3t, g'( t) = sm} (2t - sin 2t) . cos t Hence g'(t) > , 0 <.t  1, .which implies that get) > g(O) = 0, 0 < t  1. We deduce that f IS an mcreasmg function on 0 < t  1, so that 92 thal is 35. (35.0) Solution: (35.1 ) We set and deduce that where SOLUTIONS SOLUTIONS 93 that is 1 tan t - t '3  t 3  tan(l) - 1, 0 < t  1. Since tan(l) < tan(-n-j3) = .J3 < 2 , we have tan t - t o < t 3  1, 0 < t  1 , which completes the proof of (35.1). For 0  x  7r and 0 < y  1 we have 0  sin x  1 and so 1.3.5... (2n - 3) I an = , 2 n -, n  2 . n. (35.2) o  ysinx  1 . Hence, by (35.1) and (35.2), we have Evaluate the limit ysinx  tan(ysinx)  ysinx + (ysinx)3, . I i " L = 11m - tan(ysinx) dx . ".....0 y 0 so that tan(ysinx)- ysinx 2. 3 O ysmx. y (35.3) We begin by showing that ttantt+t3, Ot1. Integrating (35.3) over 0  x  7r, we obtain (35.4) O! r(tan(ysinx)-ysinx)dxy2 rsin3xdx. y 10 10 f(t) = (tant - t)/t 3 , 0 < t  1, Letting y --+ 0+ in (35.4) we deduce that lim ! 1 " (t an( y sin x) - y sin x) dx = 0 , ".....0+ Y 0 f'(t) = g(t)/t4, 0 < t  1 , and thus lim ! 1 " tan(ysinx) dx = r sin x dx , ".....0+ Y 0 10 that is f(O+)  f(t)  f(l), 0 < t  1 , (35.5) lim ! r tan(ysinx) dx = 2. ".....0+ y 10 
U4 SOLU'I'IONS Replacing y by -y in (35.5), we see that (35.6) 1 1 " lim - tan(y sin x) dx y-+o- Y 0 2, also. lknce, from (35.5) and (35.6), we find that L = 2. 36. Let t be a rea.l !lumber with I) <: t <: 1. Prove tlmt there a.re infinitely lIla.ny integers n for which (36.0 ) cosn  1 c. Solution: According to a theorem of Hurwitz (1891): if e is an irrational number, there are infinitely many rational numbeIs afb with b > o and GCD(a,b) = 1 such that Ie I <  b2 . As w: is irrational, Hurwitz's thcorem implies that there are infinitely many rational numbers nfk with k > 0 aud GCD(n,k) = 1 such that 12w:-1< k 2 ' or equivalently (36.1) 12w:k - nl < 1f(vI5k). Let 0 < E < 1. vVe consider those integers nand k satisfying (36.1) for which k > 1fh/5£). There are dearly an infinite number of such positive integers k, and for each such k there is an integer n such that 12w:k - nl < £. For such pairs (n, k) we have 1 cosn :$ 11- coonl SOLU'I'IONS 95 21 sin ( br + %) }in (k1r  ) I < 2\sin(k'ir %)\ 1 < 21br - I '2/.:1£' - 11\ < l'l' "howiu!-!, I hal l)fJ.(J) hold" !'or inliuildy many inl('('r 11. 7 De termine aU the functions f, which are everyw here di1Terentiablp 3 . and satisfy (37.0) f(x)+f(y) f (  . +Y ) 1 - xy for aU real x and y with xy =I 1. Solution: Let f(x) satisfy (37.0). Differentating (37.0) partially with re- sped to each of x and y, we obtam 1 + y2 f' ( X + y ) rex) (J - xyp 1 - xy (37.1) and 1+X 2 , ( X+Y ) (::W.2) f'(y) = (1 xy)2 f 1 xy . . 37 1 ) d ( 3 2 ) we deduce tha.t Eliminating common terms IT! (, . an . I. , 2 " ) (37.3) (1 + x 2 )f'(x) = (1 + y )f(y . As the left side of (37.3) delJends only on x and the right side only on y, each side of (37.3) must be equal to a constant c. Thus we have c f'(X) = 1+ ill 
96 SOLUTIONS and so f(x) c arctan x + d, for sOllie constant d. However, taking y 0 in (37.0), we obtain f(:1;) + frO) = f{.I;).1'O that f(O) = 0 and d O. Clearly f(x) carctanx satisfie" (37.0), , and so aU solutions of (a7.0) are given by i f(:r) = (' antan. , \v1lcr(' c is a. const;mt, 38. A point Xis chosen inside or on a circle. Two perpendicular chords AC and ED of the circle are drawn through X, (In the case when X is on the circle, the degenerate case, when one chord is a diameter and the other is reduced to a point, is allowed.) find the greatest and least values which the sum H = 1.1C1 + IBDI can take for aU possible choices ofthe point X. Solution: We can choose an (x,y)-coordinate system in the plane so that the centre of the circle is at the origin, BD is parallel to the x-axis, AC is parallel to the y-axis, B lies to the left of D, and /1. lies above C. Let X denote a point (1',8) such that (38.1 ) 1'2 + B 2 :::: 11 2 , where R is the radius of the cirde. Then the coordinates of the points A.,B,C,D are (r, VR2 - 1'2 ), (- V R2 - s2,B), (1', - VR2 - 1'2 ), ( VR2 - s2 , ,9) respectively. Thus we have IA.CI = 2 VR2 - 1'2 , IBDI = 2 VR2 - s2 , and 1'0 S(r,s) IACI + IEDI = 2( VR2 - 1'2 + VR2 - B2) . SOLUTIONS 97 We wish to find the ma,ximum and minimum values of H(r,s) subject to the constraint (38.1). '. > . . First we determine the maximum value of S(r,s). Clearly, w(. h,1V( anti l;}riR prove!: tllat, VRZ - r Z + Jii z .9 2 :::: '2R , max 8(r.s) 8(O,O)=:W 1 .. A:-s? '5: J {l . I f S ( T S ) We haNe Finally, we deterrmm\ the mimmum va ue 0',' , . 2HZ __ (1'2 + s2) + 2ff---.;:i ,jil2' .9 2 2: 2R 2 - (r 2 + s2) 2. 2R Z _ (1'2 + s'2) + (1'2 + S2) R Z R 2 , (JR 1'2 + JR2 _ B2 )2 so that This proves that V R2 - r 2 + Jn 2 -" 8 2 2. R . min SIr,s) = S(:!:l-l.O) = 8(0,:l:R) ,,2 + s2:SR2 2R. 39. For n 1,2,... define the set A" by An = { Is it trlle that { 4 6 8 } if n 0 (mod 2) , 0,2, , , ,... , { 6 3( 1 )/ 2 } if n == 1 (mod 2) . 0,3, ,..., n-' , u ( n An+k ) n=1 k=1 n ( G r\Mk ) '! n=1 k=l 
98 SOLUTIONS SOLUTIONS 99 Solution: We set X = {O,2,4,6....} and Y = { o 3 6 9 } have ' , , ,... . Clearly, we for all n = 1,2,. . . , so that CXJ II I ell:.! c AT, c ... c U t12n+l = )' n=O D1 C91 An+k) = X U Y Hence, we see that and ,Ql ([51 A"+J,) # !.:\ (91 An+k) , ih  A4  A(; = . . . = X . lIPIIC(', WI' have for n = L 2. __. ;),8 2 Iwloll/,:s to .Y l J Y hut. dill's IIOt. belong to X nY, 'X> n ;\'n+ h h=J ..x, n An+h n k=J n+k=O (mod 2) XnB n , 0" n A nH k=l n+k=l (mod 2) 40. A sequence of repeated independent trials is pprformed. Each trial has probability p of being successful and probability q = 1 - p of failing. The trials a,re continued until an uninterrupted sequm]Cp of n successes is obtained. The variable X denotes the number of trials required to achieve this goal. If Pk = Prob(X = k), determine the probability generating function P( x) defined by where Bn = { AnH, if n == 0 An+2, if n == 1 (mod 2) , (mod 2) , and so (40.0) CXJ P(x) = LPk Xk . k=O u ( n AnH ) n=l k=J CXJ U (X n En) n=l Xn CQ En) X n C91 A 2nH ) XnY. Solution: Clearly, we have PF{ D pn qpn , k = 0,1,..., n - 1 , ,k = n, , k = (n + 1), (n + 2),..., 2n . For k > 2n we have On the other hand, we have Pk = Prob(A) Prob(E) Prob(C) , CXJ U AnH k=l CXJ U .4. nH U k=l n+k=O (mod 2) XUY CXJ U A nH k=l n+k=l (mod 2) where A, E, C represent events as follows: (A) no n consecutive successes in the first k - n - 1 trials; (E) (k - n) th trial is a failure; (C) n successes in last n trials. 
100 SOLUTIONS Then Pk (1 - rob(D))qpn, where D represents the event of 1 run of n consecutive successes in the first k n 1 t . I h a.t east one . - - rIa s, t at is Pk (1- klp.) qpn, k> 2n. Hence we have PC:!') = 1''':!''' + q1'''(x''+1 + '" + J.2n ) + ql n " k- n -l L, (1 -. L p,);/:l-, k=21<+1 '=0 and 1>0 !(x) pnxn l+q(x+'''+xn)+ q  xk-n_ q -{ k-J L, L, L, Pi xk - n k=2n+ 1 k=2n+ 1 i=O 1 (x - x"+1 ) xn+l 00 n+1 +1] (l-x)+q (l-x) -qx n + 1 LLP;x l 1=0 ;=0 (1 - x + qx) co n+l (1 - x) - qx n +1 L Lp;x l /=0 ;=n == (1 - x + qx) 00 1 (1 x) - qx n +1 L LPn+r XI 1=01'=0 . (1 - :c + qx) 00 1 - qxn+l,", '"' 1'+s (1 x) L, L, Pn+,'X 1=0 T.S=O r+s::::l (1 - x + qx) 00 'XJ == - qx n + 1 '"' '"' r+s (1 - x) L, L, Pn+,'x ,'=0 s=o (1 (1  : r) - qx ( f: pn+1'xn+r ) ( £>s ) 1'_0 8=0 P(x) (1- x + qx) P(x) pnxn (I-x) qX (l_x) ' P(x) = (1 px)pnx" . 1 - X + qpnxn+l . that is so that SOLUTIONS 101 41. A, B, C, D are four points lying on a circ1e such that AB CD ifi a convex qmldrilatera.l. Detel'lnine a. formula for the radius of the circle in term" of a 1.4BI, b == IBCI, c "- ICDI and d ID.41. Solution: We first prolle the following result: Th#> radius of the circumcircle of a !':::.L.,'l-.fN is givt'n by (.n.1) lmn In+m+n)(i+m-n)(l m+n)(-l+m+n) ' R where IMNI, m INLI, It ILM!. Let C denote the circumcentre of !':::.LM ]\1, so that ILCI 1M CI IN CI == R. Set a LMCN, f3 == LNCL, 1== LLCAf, so that 0'+ f3 + I 211'. By the sine law applied to .6.MCN we have R sinO' sin«lI' - 0')/2) so that sino . l = R-. ( /2) = 2Rsm(0'/2). cas a SimiJarly, we have m = 2Rsin(fJ/2), n = 2Rsinh/2). Thus we obtain n 2R == sin(I/2) sin (11' ( a  f3l ) 
102 SOLUTIOKS ' ( a 3 ) SIll -- +  2 2 sin(oJ2) cosCi:}/'!.) + cos(a/2) siu( 8/2) 1 /--;;;2 m r [2 2R V l 1R i + 2R Vi  11(.2 ;md so 1/ r- IV I . 1 --.... m. l I l2 - + 1f1 V I --- lFl . .j Hl ' Squa.riul!> bof.llliid,'., WI, obtain 2 2 ( m2 ) . ( z2 ) 0 ( ) ( ')). ==1 .1- 4R2 +m 1- . 1R i . +21mV I- 4H i ] i;) , an d so I -- [2 m 2 2lm ( 1 - - ) ( 1 - - ) 4R2 4R2. (n2 l2 l 2 m? 1'11. 2 )+ -, 2R2 . S(!uaring again we find that '1l 2 m 2 (1 [2 ) ( m2 ) 4R3 1 - '1R i = (n 2 1 2 2 2 [4m 4 m) +-. 4R4 2 2 .11/1. ( 2 2 2 T . R2 n - 1 '" Tn ) , giving, after some simplification (n:! - z2 - m 2 )2 _ 4z2m 2 == l 2 m. 2 n 2 whidt esl,ablishes (41.1). Returning to the original problem, we set .1: -= lAC!, and 0 '= LABC, so t,hat LC 1>.11 7r - O. B,v the cosine law in D..4BC and D.4CD, We have ( 41.2) :1: 2 == a 2 + b 2 - 2ab cos 0 SOLUTIONS 103 a.nd t 41.3) .r. 2 c 2 + d 2 - 2cdcos(7r - (I) c:! + IP + 2c(lcosO, BliIlinatinv; :1'2 fwm (.Jl.2) and (.:lI.;J), \\'(, obtain It ;- /)2 .... e 2 tLl cosO 2( ab + ('t1) liin!( this t:xpreshioll for ('os () in (.J 1.2), we 1';t't ,1: (/1'£ +11 . (''l (/,:'... Ii ab (ab + e.[) ( ar + b dJ!a d + bc) (ab + cd) d l ) so 1.11 at I «(lr + bd)( ad + b;;) X:..o; V (ab+cd) . The ra.dius T of the drcle passing through i1,B,C,D is the circllmradius of D.ABC, and so by (,iLL) is gh'en by j' -- nln: J; )( a .. - b + .1:)( -(I + b + x) J{o.+b+x)(a+b abx ,if N ext we ha.V( I 'j ,  'I' I' , ' ,i (0 +b)2 - Xl .. 2 ( ac+bd)(o,d+b.:) (a + b).- (ab + cd) ab«o+W-(c d? ) (ab + cd) +b c+ c-d2 (ab+ and similarly l To 2 ab(-a+b+c+d)(a-b+c+d) (a - b) == (ub + cd) , 
104 SOLUTIONS SOLUTION S 105 so that so that g 8 = -i(p - r), provinl!; that IF RI = IQSI a.nd P R .1 Q8. r= (ab + cd)( ac + bd)( ad t- 1)(:) ( -(1, + b + c t d)( a b + c + d)( a + b c + d)( i! -I- b + c . d) 43. Det"I:Jllinp polynomia.ls (I( J:':I/, .:, 11.» and q( x, y, ;;, II') with real ('"pfficipnt snch tha.I. ( <I:{'O) (:ry + ::: + 11')2 (;1:2.. 2::)(rP - 2w) (pI.D,If..:.W)) . (.1'2 2;:')(q(;I',If,,.w)f, 42. L<:t ..l1U' I) he a COllvex quadriJatpral. Lei, l' 1)1' tl1P pnint ullhide ABeD such that kiPI iJ'B, ani! LAPH ... !lilt'. The point q,u,s are ,.,jmilarly (!i.fined. Provp 1.lt:\1 tlw lhlPs pn alld q.i.,' arp or I'qlla.l IPHglh alld perp<:lIdicular. Solution: We cOTisider the quadrila.teral ABCD to be in the complex plane and denote the vertices A,B,C,f) hy the comlJlex nmnh"rs a, b, c, b. Then the midpoints H,I(,L,J-l of the sides /lB,BC, CD, TJA (re mpresentpd by (a + b)/2,(b + c)/2,(c + d)/2,(d + a)/2. Let p represent Ihe point r. As IPHI = IBHI a.nd PH .1 BH we ha.ve Solution: We sed, a solution of (4:3.0) of the form { 1-'V,y,z,'lI.') = ;ty + X , q(:l;,y,.:,w) = y+l', (:U) where X and Yare polynomials in X,lI', amI z. Substituting (4:U) ill (43.0) and simplifying, we obtain ( a t b ) p- - 2 i (b (al)) ('13.2) ((.:; ... tv)2 I- 2x 2 't1:) + 2x(::; + W):IJ (X2 (X2 _ 2Z)y2) + 2 (xX (:Z;2  22)\'') y, so that ( 1 - i ) p = -2- (at ib) , which givps Similarly, we find that ( 4:3.3) { X:: - (x 2 2Z)1'2 :c\" (x,2 - 2z)Y (z 1.11)2 + 2J;2 W , x(:; 1 w) . { q ["T' fie), r = i (t: lid) , s = 1 2 i (rl+ in). From the second equation in (4:3.3) we have x = (:1: 2 - 2z)Y + x(z + <1')) /J: , and, using this in the first equation ill (43.::1), we obtain after simplification From this we obtain zy2_X(Z t.())y 1.:r. 2 w=0. q (L2'i) «a - c) + i(b - d)) , oS = .( \ji )«b d)+i(c-n)) = -2 (t2' ) «a c) I i(b -- d») , Solving for Y we fiud l.hat Y = :1;1.11/::' or Y .1:. Dis,a,rdinf!: the first sob1l.ion ;,s we are seeking pnlynorniah X a.nd Y. we have p-r x = :r. 2 ;;; + W, Y ;r. , 
106 ane! so we may take p(x,y,;::,w) X y + x 2 - z + .w , <l e x ) \I' ,y,z,W. SOLUTIONS x +y. 44. Let C "(u{)te the field of t'OlllpJ"X II 11111 her,... Ld, f : C  C 1)(' a. f11l1C1iofi satis{\'inp; (-1-1.0) { f(0) :-:; 0 , If(z) f(w)1 Iz 1/)1, for aU z in C and 111 = 0, 1, ,i. Pmv. tlla.t f(z) f(1)z or f(1);) , where If 0)1 = 1. Solution: Fl'Om (,H.O) WI' haw (44,11 lJ(z)1 = I'; , (44.il If(z) -. od = Iz 11, ( 4,1.3) If(z) -iij = Iz - il , which hold for all z in C, and whore (44.4) a f(l), i3 f( i) Taking z 1,i in (44.1) and z = i in (41.2), we obtain (/14.5 ) lal 181 1, 10 - /11 = h . SOLUTIONS 107 Hence, we ha,ve ({2 +;P _ a 2 (J/1 + oCi;,P a(J( all + afl) - a(1 (na + ,r{fJ - (c.r - ,fJ)(a - i:j)) (r[1 (10'1 2 + !!W -In !W) u;}(1 + 1 ..2) (I , so that (4-'1.6) {1 = fa, ( :l:i. Next, squaring (44.2) and appealing to (44.1) and (-14.5), we obtain (44.7) af(z) + O' f(z ) z + if, for all z in C. Similarly, quaring (44.3) and a.nd appealing to (44.1), (44.5) and (44.()), we obtain (44.8) Cif(z) nf(z) = -fi.? + ci'f . Adding (4,1.7) a.nd (4'1.8), we deduce that 20.f(z) (1 - d)z + (1 + (i): , that is, as c = :l:i, Cif(z) = z or z. Hence we have f( :;) = .f(l)z or i(l)z, where If(l)! = 1, and it, easy to check that, both of these satisfy (41.0). 45. (45.0) If x and yare ra.tional numbers sueh that, tan'/!':t. = y, prove that x = k/4 for some integer k not congruent to 2 (mod 4). 'lIB I 
108 SOLUTIONS Solution: As x and yare rational numbers there are inte g ers P q . l' s such that ' , , { x=P/q, y=r/s, q>O, 8>0, GCD(p.q) = GCf)(r,s) ]. The equation (45.0) becomes (,J!).l) l' tan q S \Ve have, appealing to Dc:Moivre's theonm. ( s + r ) q s - zr ( 1 + i1'/S ) q 1-ir/s ( 1 + ita:n(7r p / q » ) 1 1 itan(7rp/q) ( cos(7r p / q ) +  sn(7rp/q» ) q cos(7rp/q) zsm(7rp/q) cos( 7'ip) + i sin ( 7rp) cos( 7rp) - i sine 7rp) ( m 1)P + i.O ( -1)p - i.O :;:: 1, so that, appealing to the binomial theorem, we have (s+ir)q (li - ir)q «s +i1') 2ir)q t ( k ) (S + ir)q-k( -2i1')k . k=O Hence, we have ( -2ir)q -  (  ) (s + ir)q-k k=l k = -(8+ir) ( k ) (S+i1')q-k-l, k=l SOLUTIONS 109 that is ( 45.2) ij 'i} , (-2ir)q = (s + ir)(x + iy), for some integers x and y, Taking the modulus of both sides of (45.2), we obtain 2 2q 1' 2q (s2 + r2)(,r,2 + y2) . Let 7' be !In odd prime dividing 8 2 + 1''l., Then p divides 2 2q r 2q a,nd so 1) divides 1'. Thus p divides s2 (8 2 + 1'2) - 1'2, tha.t is, p diddps s. This fOlItra.(lids GCD(r,s) = 1. Thus 8 2 + 1,2 has LlO odd prime divisors a,nd so Hlust be a. power of 2, say ., 'ti I I I of S2 + r 2 = 2 1 , I 2: ° . Further, if I 2: 2, then sand l' are both even, which is impossible. and so 1 = ° or 1. As s > ° we must have (r,s) == (0,1) or (:1:1,1). The first possibility gives x = k/4, where k = 4p, while the second possibility gives x = k/4, where k 1 (mod 2), thlL<; completing the proof. snch that Second solution: (due to R. Dreyer) "Ve m.ake use of the fact that there are integers c(n,r), n = 1,2,... ; l' = 0,1,...,[n/2], (45.3) [n/2] 2cosnO L c(n,1')(2cosOt- 2r ,'=0 II for a.ny rea1 number (I. The integers c(n,1') are given recursively by c(l,O) = 1, c(2,0) = 1, c(2, 1) -2, .. and for n 2:. 3 I ,I! iN, I { c(n,O) c( n, r) c(n,n/2) = 1, =c(n-1,r)-c(n 2,1'-1), 1:S:r:S(n-1)/2, = ( -1)n/22, n even. 
110 SOLUTIONS Now, as x is rational, we may write x = plq, where GCD(p,q) 1 and q > O. Furth"r, as y tan 11'X is rational, so is the quantity - 2 .) __9 (l-t<l1l211';/;) ,(I_y2) k - cOS11'X -..  - , '-= 2 .-- (1+tan 2 r.x) (1+1;2)' Al'pealin to (45..3), with 1J. c- fj and 8 = 211'X = '211'7Jlq, W0 S{'(' that :: is a, rational 1'0111. or UII' monj{' illtPgnl1 polynomial 'nIl] /(:1:) = 2..:: e(n,'I'J:t:""2r 1.. ,,,,0 Hence, z must be an integer. But Izi = 21 ws 211'xi :S 2 so that z = 0, :1:1, or :1:2, that is cos('111'plq) = 0, :1:1/'2,:1:1 , giving 211'p q .11' . 11' (2l + 1) 2 ' (31 :I: 1):1" l11', for some integer l. Thus, WE have x 11 q 2l + 1 4 ' 1 or 2 3l :f: 1 6 Only the first possibility, a,nd the third possibility with 1 even, have y tan 1IX ration<l,l, and hence :r. = k/4. where k Ii; not wngruent to 2 (mod ,f). 46. I,et P be a point inside the triangle ABC. Let AP meet BC at D, HI' meet CA. at'.B, and CP meet All at F. prove that ('16,0) IPAIIPBI IPBII1'CI IPClIPill IPnTlpi[ + iPE[ IPF I + IPF I IPDI 2: 12. Solution: Let S,Sl,!:h,S3 flenote the areas of 6..4.8C, 6.PBC', 6.PCA, 6.P AH respectively, so that S S} + S2 + 83- Since 6.,1HC and , "1111 J iir- " 11 :. / 'ilB SOLUTIONS 111 6.PBC share the side BC, we have IADI S II'D\ Sj ' "i. .J tlU 1m so that Will I ADI-I/'DI "Wi 1/'1>1 IpO[ /-'VI 8 ,f,' ... ::J\ Sl, + S';) 1..,.-.....---...- 8 1 ... 8 1 ... ,1.;1 ;' Similarly, w(' haY<' IPH.I_:,-'! + 8 1 IP /':1 - .'h )pGJ jPFI 8 1 + ,','2 S3 Hellc(', we ha,ve JPAIIFBI IJ'RIIPCJ IPClIP"11 I PJJ, I PEI + j' PR I H)FI + WI'I jl> DI (.)2 + S:!)(S3 -I HI) (5':, + 8 1 )(8 1 + 8 2 ) (HI + .'h)(S + '2:: t +---..., + c<c 5\,'h ,'i2.'h 0:,.71 ( Sa ,)'3 ' I , 8 ) ( .1.;1 -I ,'it + l + Sf ) -. -;- + -;- + + ,---;- + --;- .. . S1 5'1. 5}5 2 8 2 S3 8z8:1 ( .')2 ."/2 .. Si ) + . 5 -;- -t (-;- +1 + '] '. S ' . :I "1 . ". 1 ( ' 3 + 5 ,., > ) ' + ( ' 5 3 + :2. ) + ( I + '2 ) ' .5 1 . 3 2:! 2' 1 ( Sf s s ) +3 + 8 2 1; , + S 3 8 , + 8}S2 > 2 + 2 + 2 + :) + 3 12 , by t.he arithmdic-geom('tric mean inequality, which completes the proof of ( 46.0).  I 47. L<'t I and II. be. positive integers sllch that 1<;I<n, GCD(l,n) = 1. 
112 SOLUTIONS Define the intefJ:er k uniquely hy 1<.k<n,kl -l(modn). Let AI hI' the /... x 1 ma.trix whose (i.,JJth entry is ( i-l)1 + j . LP1. IV ht lilt' k X 1 ma.trix t()J'rnt.'l! by ta[{ing tlw (,O[I11I1T1 of :11 inl'('\'ers(' order ;111<1 wril illg lhe ,'ntl'i,s as th,' rows of y, \i\.'hal is the I't.laliolJsIJip IIdween Ihe (i.jHIJ {'ull'v or AI ami '.]1<' (i.jl-th (,Idry 01'\' !IIodul" /I':' Solution: If.4 [aij: a.nd B [bij: a,re two J.: X 1 m;;,trices, we write ,1 B (mod n) if aij bi i (mod n), i:= 1,2,.,. ,l:j j = 1,2,.., ,1. As Jd -I (mod n) we have Inodl] 10 'It I 2 1 2 1 1 I ] 1 + 1 1+2 'll 2 2l- I 2l M .- 21 + I 21 + 2 :Jl 2 3l 1 : ( k - l)l + 1 (k 1)1 + 2 J.:l- 2 kl- 1 (ld - (I.; 1))1 (kl (2k l))l (2l.: + l)l (f..' + 1)1 1 (kl (I..: 2))1 (1d (2k 2))1 (2/" + 2)l (k + 2)1 21 (/;;/ - (k 3))1 (/d - (2/..: - 3»)l (2k + 3)l (k + ;3)l 3l (kl)1 (ld -1.:)1 :3kl 2l.:1 kl from which iI is deal' that Ow (i,j)-th ent.ry of N is 1 times the (i,.i)-th entry of Af modulo n. 48. Let 11/, and It be int.egers l'\11ch that. 1 :S 'II/. < n. Let t1.ii, i 1,2, . . . ,m; j 1,2,. . . ,11, be rnn integers which are not all zero, and set a= max la,'I. l<i<,." J 15.j5.1l SOLL'TIONS 113 Prove that the :>ystem of equations .4 If (-18.0) { aU Xl + aU"'2 + . . . + atn X " :".,' t n",,+ .. . +0,,,,,, Uml:l:l + a m 2xZ + ... + a,."".x n o . =0, O. ' has a. oluti{)11 in illt./'gers ;1: I, ,1;2," , ,:/:n, not all zero, sa.tbfying J I lit '1 I£,I:S. ;1.2I1IL)" '" i, I'::: j::; I/. . l. . Solution: We fi(,1. i' N (27H)n:'..";:;;] tiO that i , N > (2na)"c:',;; 1, which implies (N + l),,-m > (1na)"'" . Hen....., we ha.ve (N+1)" > (2na)'''(N+l)'n (2naN + 2na)'" , \. that is, as a 2: 1, ( 18.1) (N + 1)" > (2naN + l)m . Set ,f. ! Li = l'i(Yt,Y2,... ,y,,) a;lYI + U,2Y2 +.., + ainJ/n, for 1 ::; i :S rn. If (Yb 11'1.,. .., Yn) is a vc'ctor of integers satisfying 0 :::: Y! ; N, 1 :'S; j::; 11, the corresponding value of Li Li(Yl, Yz" .. ,Yn), 1 ::; 1 ..::: Tn, sa,tbfies ;'1 'Ii naN :S /-i ::; naN, 1:S i :S Tn , 
114 SOLUTIONS SOLUTIONS 115 and fiO t.ht' vector (1. 1 , 1. 2 , . . . . L n ,) of integers can t.ake on at most. ('!.naN + l)m differellt. values. As t.here are (N + 1)'" choices of the vector (:t/1, Jh,... ,!IT()' hy (-IR.1) there must be t.wo distinct vectors Hence, we haV0 d t ) _:c 2 ) 1 (11.(3: e IX U (YJ,Yz,...,Yn), :l (Zl , Zz, . . . , zn) , and so say, giving rise to t.he sa.Tllt' vector (L], 1. 2 ,.. ., Lm). Set ('J9,l) !/(:J:) 2J:h(:r) I. . X,i .:7 Y.i ._" 1<:.1<n. !\b h(.!:) iti ,I, ra.tional ('tlllrt-ioll WP ma,:,' writ!' ;\, III(' two vector" 11 and l' <1)'1' dioliuct, n(J1 all tilt' ,1'., an' ZPI'O. IV1on'oV('I', as [1".:') 11(.1'.) 1'( ./:) t/( .1: I Li(Y1.!J2,'" ,J/n) Li(zl,z2,... ,zn), 1 <: i <: Tn, whe)'e p(:r.) and q(.'!:) are »olyoomiaJ:.; with q(;c) not idt'Tltica.lIy wrO, and GCD(1J(:r),q(:r)) 1. Thp.1I (J:J, :r2, . . . , :r n ) is a sol utiofL of (48.0). Finally, Ix j I <: ]V, 1 <: j <: n, follows from the fact that 0 <: l/;i,Zj <: N, 1 <: j <: n. 49. Liouville proved that if pi ( :r. )q ( ;t: ) rJ(r)ql ( ;t: ) q(..:)2 and using (49.2) and (.19.3) in (-W.l), we ohtain hl(.,!;) ( 4H.3) J f(J')eg(x)rlJ: (4HA) ll( l: )q(;r) p( X )ql(;r) 2J:p(:1' )q(;!') q( x)2 . is a.n elementary function. where f(;c) and .Q(x) are rational functions wit.h deg]'ee of g( x) > 0, then If q(x) is a constant polYllornial, sa.\' q(J:) 1.:, t.hen (41A.) bewmes pl(:r.) - 2;rp(:) I.:, J f(x)(,y(x)dx h(:!.' )t:,?!r) , J . _x 2 1 f: I X which is d('-arly impost;ible as the dt'grcc of thp- polynomial on the left. side is at. least one. Thus, q( J:) is a. non-cc.I!Ista.nt polyuomial. Let c denot.e one of its (compJex) roots, and let. '117, (2:: 1) denote the mlJlt.iplidty of c so that (:c - c)1r. 'I q(x). Then, we haw (3: c)m-I 11 (/(:1:), and tI'om (;19.4) written in the form p(;r)q'(x) (pl(:C) - 2:rp(x) q(3:»)q(X) , we see that (x c) I pel;), which contradicts GCD(p(:I:),q(:I:» = 1, atJ. d com- pletes the proof. where h(:r) is a rational function. Use Liouville's result to prove that is not. an element.ary funftion. Solution: Suppose that I e-:? doc is an dementary function. Then, by LiouvilJe's rCt;ult, there exi$h; a. rational function h(r) such that 50. The sequence xo, X],... is defined by the conditions ! dx h(:l') (50.0) Xo = 0, Xl = 1, :1: n +] n> 1. 
116 SOLUTIONS Determine L::::; lim X n . n-ov Solution: The reCUrrence relation can be written as n Xn+l - .'I'n n+1 Xn-d, n  1, so tha.t (50,1) XnH-X n (-l)n(XI a:o)::::; (_1)n n+l n+l' n1. The equation in (50.1) trivially holds for n = O. Hence, for N  1, we have N-I XN == L(x n +1 - .J;n) n=O N-I (_I)n L n + l ' n=O and 1i0 L = lim ''CN lVoo N-I lim " N_ oo 7 n + 1  (-l)n  n+l ' that is L = In 2. 5l. (51.0) Prove that the only inte g ers N > 3 W I ' th the fioll . . L _ . . OWIng property: if 1 < k.s; Nand GCD(k,N) = 1 then k is prime, a,re N == 3,4,6,8,12,18,24,30. SOLUTIONS 117 Solution: It is easy to check that :-1,4,6,8,12,18,24,30 are the only integers .s; 121 with the given property. Suppose that "v > 121 is an integer with the property (51.0). Define the positive integer It  5 by 1 5 1.1) I  p,. .s; v '\ < P,.+1 , where Pic (knott's Ihe I...t.l. primp.. From (51.1) we see tha.t p; ::; :V ,j 1,L,. .., 'Ii, aud w by proppj.ty (;)1.0) we l1lUt have 1'.i IN, lor j = 1,2,.. . , F.. As PI, . . . ,Pn are distinct primes. we must have ( 51.2) PIP2 .. . pn IN, and so, by (51.1) and (51.2), we have (51.3) ]JI]J2 ... pn .s; }v < P+l . By Bertrand's postulate, we have pnH .s; 2pn, Pn.s; 2Pn-l , and so (51.4) 2 > Pn > Pn-IPn - 2 - R Using the inequality (51A) in (51.3), we obtain I1-tP2 . . . Pn-2P+d8 < PH , that is PIP2'" Pn-2 < 8. Since PIP2 == 6. a.nd PI112P3 = :-10, we must have n - 2 .s; 2, and It .s; 4, which is impossible, proving that there a.re no integers N > 121 with property (51.0). 52. Find the sum of the infinite series 1 1 1 1 8=1- 4 + 6 - 9 + 11 1 14 + ." jlll1l 
118 SOLUTIONS Solution: WE' bf'gitl by observing tha.t 1 1 1 1 1 I S = 1 - 4 + 6 9 + 11 - 14 + ... ( (1 - :i:: 1 -I :/.:".. :1;" + .,0 If) -. .) d:/.o J(J fal (1 ,/1)( I , 1''' + ,/, W 'r ... 1 d.1' II x: s dx r 1 .1;2 + x -1 1 = 10 x 4 + a:- 1 -+ :1:"1 -+ x + f dx . Now, tl('composing into partial fractions, we have :1;2 + x + 1 ;4 + :r: i + :/,;2 + x + 1 b where +1' a H 10' I-VS C - 2 , b - - 10 , d  2 Tlms, w(' havE' S aIc + bl,l , where 1,= [ dx +c.r-l- 1 I _- 1 1 d- o dx -I- + 1 Now J dx 1 ( 'x+t' ) 2 F arctan 'r;--- , t < 1, :;: + 2t:c + 1 v 1 - t 2 . V 1 -, t2 and by th(> fundamental I heot'em of ralculll", we have [ 1 (arctan ( )1  tt 2 ) arct.an (  ) ) a,rctan ( I   : ) - 1 SOLUTIONS llU Hence, taking t (1- ,;g1/4 a.nd t (1 1- ../5)/'1, we obtain II! ) - 2Ji'; , .. ( V i) -I- 2J5 ) /.. - \/ arctan - F ( V fi Vi) and. SO\ so thai r---' '-, ( ' f l. .-----:- Ii: ' ) JI()+'1v' V" 2V,1 t,; = V:) . -- <I.rd,an ''''''-,/,-'--, fUS( 1!'/1O) (/10 1 -'1>/3 )!--1, sine 1!'/1O) (J- 1)1'1, ( ' 0 J F )/ I ill ( ;{1!' / l()1 "'- (Js+ 1 I/-I , cos(3Tt/Hl)" VII v:j" . 5 t.an(Tt/lOl= r,;:' Vi) f", + 2V: tan(,,/10) ==" h - VG Hence, we find t1lat and so as required. '--'-- 31!' / 10-2-15 I" = LO V 5' , --_ , 1!... . /10-1- 2-./5 la - HI V ,') s -.. ( [ 111 - 20 r.: /10 \- 2vG ) ..::. 3(;> f- A)\! " + (5 VG} V 'I' lOll' V .J '. .. ( 3(-./5 + 1) / 10 - 2-15 + (-./5 - 1)/ 10 1.2-15 ) 100  ( 6 l 1O + 2-15 t 2[W  2Yi ) ' 100 ,--- - ) ; ( 3 V lO + 2-./5 + y10 - 2-15 , dO 
120 SOLUTIONS SOLUTIONS 53. S<:micircle:;; are drawn externally to the sides of a p;iven triangle. The lengths of the common ta.ngents to tht:'.se semicircles are l, m, and n. Rl'late the quantity lm. mn nl - +.. +- n 1 m to tlw lell)!;thH of the ides of the tri<lUgl.... 121 54. f . if R 4 R ha.vin g the properties Determine all the unctIOns : (i) H(l,O,O,l) = 1, (ii) ll(Aa,b, Ac,d) = AH(a,b,c,d), (iii) H(a,b,c,d) == -H(b,a,d,c), (iv) H(a + c,b,c + J,d) "'" H(a,b,e. d) + H(e, b, J, d), WIHt't' (J"b,c,d,t:,f,A a,rc \"teal numhers. Solution: Lpj 111<' \('rji(',' of I.hl' giv('n tTianle b,c /1, B.C. Lel ,I'.lJ'. C' h,' 1IIP ('('II!,n", or 111" Pllli('irdp (1,,''1.-; 11".1.\,'0 Oil B('J' 1,111 rpspp('tivply. (,pj Of.',FG.HJ be tlte common tangents to /J and 'J', 'I and (x, n ;I.nd l' rcspedively. Join JJ'D,C'E a.nd draw C' [( from C ' perpeTl(u('ular to B'D. IIt:>lIc<" as 1((" RD is a rectangle, we have KC' = DE l.l,et Solution: Hy (Hi) we have H(l,l.O.O) -Tl(l,l,O,O), II(O,O,l,l) IAHI = c, IBL'I = 20" IC Iii = '2b , so that Then, we have (54.1) IR'(."I a, !lJlKj 111-cl, II(l,l,O,O) 1l(0.0,1,1) 0, 'f." ( " 1 - J 2 - (I _ ) 2 Ii.' -. a . c. , (54.2) and from (i) and (iii) we have 1I(0,1,1,0) -JI(l,O,O,l) = -1. a.nd o tlwt is RNlce, we obtain l=j(a -b+c)(a+b-..). Similarly. W{ have { :2 = IPGI = J (a + b- c)( -a + b + (:) , = IHJI = J(=a+b+c)(a b+c), a.nd EO mn T = -a+b+c, nl a b+t:, n lm a ,I, b - c, m gi \'ing (5;1.1) m.n nl lm + + m n a+b+c, that is so tha.l, t.he left. 8ide of (5;3.1) is the sem.ip(rimeter of the triangle. H(a,b,r,d) H(a,b,O,d) + II(O, b,c, d) all(l,b,O,d) + clI(O,b, I,d) -" -aH(b, l.d.O) - cIl(b,O,d, 1) = -a (lI(b, 1,0,0) + H(O, I,d, 0)) -c(lI(b,O,O, 1) + H(O,O,d, 1») -abH(1, 1, 0, 0) - adlI(O, 1, 1,0) -bcJl(1,O,O,1) c.dlI(O,O,l,l) =-ab(O) ad(-l) be(l) cd(O) ad bc, H(a,b,c,d) = I   I. H(O,O, 1,1), (hy (i')) (by (ii» (by (i'ii») (by (i,) (by (ii)) 
122 55. SOLUTIONS Let "1,.." "n be the complex roots of the quation zn + (11Z"-1 +... + {ir. 0, when' (ll,. .., an ar(' I/. (2: I) ('omplpx Humlwl's. Set A. wax Iftkl . lk:Sll, . l'rov{' thm !,..I:S'] + ,I. .7 Solution: Set I .) P'.H __ fl. f(::) z"+a.!z"-ll',,,+a.,. and suppos(' tha1: one of the Zj, j :S' j :S' n, it' such that Iz' l > 1 + A, Then , we ll<t.ve .1 () == If(zj) i :n (1 a1 a". ), ' ! "jl + .. . +- '?j .""5 7 } I  I " 1 1 + fl.l + an , . -.1 .' .... 1-. I Zj zj'; 2: I::;jr' ( 1- II 1".11 _ !zjl" (1- II - H. IZjln ( 1 _ __1.  '" - iZjt IZjln ( 1  ) ' 1,0.11 1 Iz;I" (Izj; - (.4 + 1» . IZjl I > 0, ... _ l (l"l ) IZj!n /l ) 'V :n !'-J i Iz" -..-) SOLUTIONS 123 which is impossible. Thus all the roots z}' 1 :S' .i :S' rt , of fez) must satisfy IZjl :S' 1 + A. 56. If m and n are positive integers with 1It odd, det.ermine (I G(.' D(2 m 1,2" + I) . Solution: Define integers k and I by 2'" - I = kd, 2", 1 = ld , and then. we obtain 2'" = kd + 1, 2T' = ld - 1 , and su for integers sandt we have { 2mn = (kd 11)" = ,qd + 1 2 m 'lL = (ld - 1)"" = td _.. 1, a.s m is odd, Hence, we have (,5" t)d -2, and so rl divides 2. nut dearly d is odd, so that d = 1. 57. If f( x) is a polynomial of degrp.€ 2m f- 1 with integral coefficients for which there a.re 2m + 1 intep;ers k 1 ,. . . , kZm+I such that (ii7.0) f(k 1 ) '.' = f(J'2m+IJ 1, prove that f(x) is not the product of two non-constant polynom.iab with illtep;ra.l coefficients. Solution: Suppose that f( x) is the product. of two non-consta.nt polynomi- als with int!'gral coefficients, sa.y f(:c) g(x)h(x) , 
124 SOLUTIONS where'/'= deg(g(x») and s = deg(h(x» satisfy r + s = 2m + 1, 1 .::; r .::; 8 .::; 2m . Clearly, we have r'::; m. Now, for i 1,2,..., 2m + 1, we howe, from (57.0), 1"" I(k,) g(ki}h(k,) . A;; 9(1,;;) is an integer. we must. have g(k i ) = 1, 2, .. . , 2r/1. + 1 . Clea: ly , either + 1 or -1 OCf;urs at least rn + 1 times among the values of 9( 1.,), 1 .::;,  .::; 2m + 1, and we let E denote this value. Then 9(x) - E is a polynollli al of degree at most m which vanishes for at least m + ] values of 3:. Hence the polynomial g(x) E must vanish identically, that is, g(x) is a constant polynomial, which is a contradiction. Thus there is no factorization of I( x) of the type supposed. 58. that Prove that there do not exist integers a, b, r, d (not all zero) such (58.0) 0,2 + 5b 2 2c 2 - 2rd 3d 2 0 . Solution: Suppose that (58.0) has a solution in integers a, b, e, d which are not all zero. Set { Tn = GCD(a,b,c,d) , al alrn, b l blm, CI = elm, d l dIm. Then dearly (aI, bI, CI, d l ) is a solution in integers, not all zero, of (58.0) with GCD(al, b I , CI, dd 1. SOLUTIONS 125 Hence we may suppose, without loss of generality. that (a, b, c, d) is a solution of (58.0) with GCD(a,b,c,d) 1. Then, from (58.0), we obtain (58.1) 2(a 2 + 5b 2 ) (2c + df). + 5d 2 , so that 2a 2 (<!c + d)2 (mod:J). Since 2 is a q1Jadmiic Jloll1'esidue (mod j) we must have (!).2) a 1(' + d 0 (lIIod 5) , Sd, a. = )X, 2c+rl J))' wherc X and}' are integers, so that (58.1) becomes 2(5X 2 + b 2 ) 5y2 + d 2 . Thus we have 2b 2 d 2 (mod 5). Again, as 2 is (!, quadratic nonresidue (mod 5), we deduce that (58.3) b == d 0 (mod 5) . Appealing to (58.2) and (58.3), we see that a b == c == d == 0 (mod 5), contradicting GCD( a, b, c, d) = 1. Hence the only solution of (58.0J in integers is a = b = r d = O. 59. Prove that there exist infinitely many positive integers which are not representable as sums of fewer than ten squares of odd natural numbers. Solution: Vile show that the positive integers 72k+42, k = 0,1, . . . , cannot be expressed as sums of fewer tha.n ten squares of odd natural numbers. For suppose that (59.1 ) 72k + 42 2 2 2 Xl + X2 + .,. + x 8 , for wme k > 0, where XI,''', x. are odd integers and 1 '$ s < 10. Now, x; == 1 (mod8) for i = 1,2,...,8, and so considering (59.1) as a congruence modulo 8, we have 8 == 2 (mod 8) . ii; 
fOT n 1,2,. .. , we have 1 ( 211 ) 11+1. ,11, 126 SOL{;TIONS SOLUTIONS Since 1 ::S . < 10 WI' must, have s '2 a.nd so (.59.2) 72k + 42 2 +x 2 . Tr....<!1.ing [!I!),2) a>; it congru01lCe rnollulo :J, we obtain xi + :t 0 (mod 3) . SillCf> thl' '''lilaH' of <I,ll illt(('r is <;ong;rIJ1'nt to () or ] (mud ;), we lUust have ;!:I :;:: 0 (mod :!). Finally, T P dlldng (59.2) Illodulo 9, we obta.in the rnntra.dirtion fi (I (mod !J), as rCljllired. 61. Prove bhal 60. Eva.lna,t(. the' integra.l ( 60.0) 1 '0'" sin lex rosk x l (X , o x is a.n illteer for /I . 1, '2, :,. . . . I(k) where k is a. positive int0g('r. Solution: Solution: By the binomial theof0m, we ha.ve (60.1) (t?2i'" + l)k t ( : ) (;:l1'i' . r=O As (e 2ia , + II ekio: (eio: -I- e-i,ry, (cos kx -I- i sin k:r. )2 k cos" x , the im1lginary part of (e 2ix + 1)" is 2 k sin kx ros k :1'. Equating imaginary parts in (60.1), we obtain 2 k sin k:r. cos k x k ( k ) L r sin 2rx ,'=0 k ( k ) L r sin2r.r.. ,'=1 As ( 2n ) and ( 2"+1) are both intep;ers, this shows th,t n r... was required to be' proved. () is an integer, as Thus usin g f<<> .in x dx = , Jo 3: we ha.ve 127 2;1 t ( : ) r=l ...7r ( i< 1 Zk+1 ) 7r ( 1 ' 2 1 "-'i I )' 1 ( '2n.' ) ;:-+ 1 n 21/.! 1 (1/.!)211+1 2n! + (n!F 'J , ( .....11.. '. (n! )2  2n! 2.c- (n!? -I- :2 ( 2n ) ( 2n -I- I ) n n /(k) 1  ( k ) 10 "" sin 21'x k L rl.r. 2 . 1'=1 r . 0 x Second solution: (due to S. Elnitsky) For n 1,2, . .' we have 1 ( 2n ) 2n! 1 n+l n (71T)i--1 
128 SOL UTIONS == 2n! n!(n + I)! "n' n!( 1)! (11.+1)-71.) 2n! 2n! (n!)2 (:;-:- l)!(n+ i)! (2n ) ( 2" ) \ n 71. l :b {2 , :' } ' aud ( ') 1 tl ' . ,,,:v iU'!, ,>0 I lItlegl't;" this ;.lIOW,; that I ( "" ) ' "IT .;" j an inl!'5!;pr. 62. Find the sllm of the inlinite series cv':' 2 n 8 == L  2n -/ _ 1 ' n=O wlwre a. > I. Solution: We hav for 0. > 2 n n'ln + 1 :,In(0.2n 1) 0.2n+1 1 2n+1 2 n a'ln+1 _ 1 ' ,,0 that ==  ( 2" 2'f1.+I ) S L.- - ' n=O a 2n 1 a 2n + 1 1 1 a-I' 63. Let k be an inte g er. Pr ov p . 1 t th f, 1 ' .. L la c onna power series v i + kx 1 + alX + 0,2:(;2 +... SOLUTIONS 129 has integra! co<?fficients if and only if k 0 (mod 4). Solution: If I.' == 1 (1110<1 2) thpn at '"' 1.:/2 is n01 art iute/!;er and if' k :,I (mo{l4) th,'n tzz _1.:2/iI, is not an intl\ger. Wlwl} I: - o (mod 4), we have for n == 1,2,... ( '1/2 ) I. , " o,'u  n I )( :.' ,J " , _ I .. _ (II 1\ i .  i __'.. 'L'___ ln n! 1 3 ,, ) ' --. ( " 11 - " ) (_.1)"-1 ""'-' ' )-/" . 2" n! -1 n--1 (211 - 2)! k" ( ) 22n- I n!(n-1)! ' 2 ( 1)n-11 ( 2ft - 2 ) (  ) r, n n 1 41 \vhich is a.ll integer since k/4 is an int(ger and e:'-='12) is an integer by Problem 61. 64. Let rn be a, positive integer. Eval.uatt:' the determinant of the m X Tn matrix ]"lm whose (i,j)-th entry is GCD(i,j). Solution: Let C1,..' ,Cm denote the columns ofthe matrix kIm.' \Vt' <tefitH-! N m t.o the matrix whoe columns D1"'" Dm a,re given by { D, D:n Ci, i= 1,2,...,m 1, Ld(-l)"'( dj C m /d, where the sum is taken over those squarefree integer d whil'h divide m,. Clearly, as Dm :::: C m + J, where J is a lineal" combina.tion of the C:i, 1  i  in - 1, we have det .i.'v1 m :::: det N m . . 
130 SOLUTIONS SOLUTIOKS For 1 5- i :S 'm, I.he entry in the i- tIt row of D", is (writing (i,.i) for GC D(i,.i» L (-1y(.1){i,mfd) II p"lIm L ('-1)T(d)(i,p') fd) .11f.." 131 ,( f>qnJrefr d s<{uar(:fn'f: m. :::: fil + 8(21.lv .!- 1,2) , h- t ( 6" 0 \ 1 )1ds. Thml Suppose there exist integers u a.11d l' such t a, .). i It ., we have Solution: dfm II (I,pO) -, (i,JlL>-l)) sn that m "" f)/ (mod R). Hence, we must have .. '" 2 ,.. 2 :1'2 !- :'C:IJ t 17:/}' "" 'J:C + :);J (mod x) , P'\iFlt J 0""11"" (p" - ,/'-') , if i 111, 1 (I . Ii III { d>( 111) ,f i  Tn.:; o , If 1 ":, Z :::0 m .- 1 . 1 [lat j" 4.<;2 + 'l:lJ 2 =0' 0 (mod x) , and so X" ..!- y" == 0 (mod 2) , which contnHlicts the condition that I is o,Jd. lIelIG', expanding the dderminant of I"i m hy ib; T/I.'( h columIl, wp obtain det IV m. = ,i>( '/TI) det !If'n-l a nd so 66. Let del. lU n ... (_1)"-1 !-.'--ln2. . n <i>(m)detM m _ 1 . an = 1 1 1 - !- ,- 2 . 3 Tlm'3, as <let ilh = 1 ( 1), Wt!, find tha.t dctM". =: 'Pfm);,'>(m, - 1).. .?(2)q.!l). P tl . t \',,:- a converges and determine its sum. TOV{ la L-rTf:::::l n . 65. Let land 171. bE' posit.ive integers with 1 odd a.nd for which thel'C 11,1'(; integers x and 11 with Solution: \Ve have 1 1 .. ' 1 ' ) n-l n-l) dx (1 x !-:c -...t(-. :r. i (1 1 ( 1 + (_It-IX: ) dx _ t d: 1 1 + x 10 1" X 1. 1 x" dx. o tx Hence, for any integer IV  1, we have N ,v, fl (_1)n-lxn dx L: (I,,, := L.. /0 1 j- X n=1 ,,=1 . an {m "-,,x2+y2, =: x 2 + 8.1:Y + 17y2. Provl} that there do not exist integers 'U <tnt! 'V \vith (6,').0) { I '- u 2 + 'v 2 , m= ,')v," + 16'11:11 ,. 13v". t _ (1:.£ 10 1 +;£ 
132 aHo:.! o SOLllTIONS [1 1 N 10 1 + L(_J)"-lxn dx o X n=1 [1 ( X + (_I)N+1x N +1) 10 (1 + :/")2 dx 1 1 iC N 1 1 x,'\'H ( - 1 ..- ) 2 dx + (-1) H .  dx . o . +.t 0 (1 + .1' j2 . [II,' 1 1 X I L an - d.r In=1 0 (1 + x)2 I l (t:: 1 )2 dx s 1 1 x NH d:1; J + 2' Letting N -> oc. we see that an converp;es, and has sum 1 1 X dx o (1+;r.F 1 1 C  ;1 : - (1 :.)2 ) dx In2 1/2, 67. Let A = {a; I 0 S i S 6} be a sequence of seven integers satisfying o = aD .$ 0,1 .$ ... S 0,6 .$ 6. For i -= 0, 1, . . . ,6 let N; = number of aj (0 S j S 6) such that aj == i. Determine all sequences A such that (67.0) Ni !16-i, i == 0,1, . . . ,6 . SOLUTIONS 133 Solution: Let A be a sequence of the required type satisfying (67.0) and let k denote the number of zerOs in A. As Go = 0 we have k  1, and a k = No = a6 we have k .$ 6. If k = 6 then it follows that..t {O, 0, 0, 0, 0, 0, 6}, contradicting Nn 0,0 = O. Hence, we have J S k an .$ 5, a.nd so (67.1 ) Nk  1, Nk+1 ... == N e O. TbuH, oy (67.0) and (67.1), we obtain (b7.2) lIO =.. (II ' .. ,= O';-(!:+.IJ II, tl";'k'C' and so k No 6-(k+l)+I, that is k = 3. This proves that A is of the form (67.3) A == {0,0,O,a3,a4,as,3} , where (67.4 ) 1 S 0,3 S 0,4 S as .$ 3 . Clearly, we have 0 < N 1 S 3. If Nt 0 then, by (67.0), we have the contradiction as N 1 O. If N 1 1 then, by (67.0), we have 0,5 1, and 80 (67.'1) implies tl1at a3 0,4 == as 1, giving the contradiction N I = 3. If N I = 3 then a3 a4 = as = 1 and so, by (67.0), we obtain the contradiction as Nl = 3. Hence, we see that NI = 250 that 0,3 = a4 = 1 and as = N J = 2. The resulting sequence A= {0,0,0,1,1,2,3} satisfies (67.0), and the proof shows that it is the only such sequence to do so. 68. Let G be a finite group with identity e. If G contains elements 9 and h suc.h that (68.0) g5 e, ghg- I = h 2 , II' II: 
134 determine the order of II. Solution: SOLUTIONS If h e theu tIw order of hi;; 1. Thus We' may suppose that h f c We have g211g-2 g3/ur 3 !/lhy.4 !/5I1g-5 gighg-1 )g-1 = !1(q2/lfI-2)tj-1 g(g:1hg-")g-1 .Qi.q'l h g-1 ),q- j c-: g/1 2 [1-1 ,= tjll4tF 1 !1//"g-l = ,q!JH;y-1 = (gh.q-l )2 (y/iy-l)1 :- (g/Ig-I) = (!lhtF I)'" 11 4 , -- hi! , ., h 1 (; , -'-" h'{2 . and dO, as g5 = t, we obta.in h = h 3 "2 , that. i s /1 31 ' 1. ' 1 e. " IUS th0 ord",r of h is 31 a h f f: and 31 is prime. 69. Let a and h be positive integers uch tbat. DCD(a,b) = 1, t1 =t b (mod 2) . If the set S II<IS the ioUowing two properties: (i) a,bES, (ii) ;I;,y,z E S implips x +:1+ z E S, prove t.hat evcry integer> 2llb belongs to S. Solution: IAt N he an integer> 2(lb. As GC D(a, b) -= 1 there exist integers k and 1 I'uch that aA + bl=. N Furthermore, as  _ ( l )  there exiRts an integer t such that ak + bl ab -k b U <;t < t, + 1 <; a IV I > 2, l,tJ J ,Ij SOLUTIONS U5 Define integers 11 and -v by 'II, = /... + bt, " = I - at , ;J.nd intep;prs : and y hy { : c= :: 'I' " , y = v, Y I' f/ if'll + t' = 1 (mod 2) , if /I + n '" 0 (mod 2) . It. is <'asy to ch l'ck tlHt 1. iY=:ra+yb, :1:>0, y2:0. 3;+y=l(mod2). \V(! show bdow that S cOllIains a.1l integers of the fortI! :W + yb , x? 0, y 2: 0, ;c + y =' I (mod ) , completing the proof that IV E S. For Tn an odd positive integer, let, Pm be tbe a.%ertion that XQ. + yb E S for all integers x and !I satisfying ;r: 2: O. y? 0, :r: + y 1 (mod 2), x + Y Tn . Clearly PI is true a<l a,lJ E S by (i). Assume that P,n is true and considcr an integer of the form X a + Vb, where X and Yare integers witb X ? o. Y? 0, X + Y =' 1 (mod 2) , X + Y m + 2. As m -I- 2 2: 3 at leal't one of X and Y is ? 2. Then, writing X Q + Yb iu the lorlll { ((X - 2)a + Vb) + a + a ,if X > 2 , (Xa+(Y-2)b)+b+b ,ifY?'2, we see that Xa + Yb E S, by the inductive hypothesis, and so P m +2 is true. Hence, by the }Irinciple of mathematical induction, Pm is true for all odd positive integers rn. 70. Prove that every integer can be expressed in the form XZ+y2. 5z 2 , wher!' x,y,z are integers. 
136 SOLUTIONS Solution: (due to L Smith) If rn is even, say m m (n--2}'+(211 ])2 5(n whereas if m i;, odd, "".V lit == :In + 1, then 2'11., then 1)2, m. (n+1?+(2nf--Sn2. 71. EVi:l,luatt! the sum of the infinite series In2 In3 In-i In II  a +T 5+'" Solution: .For x > I we have In x r dt < r dt J 1 t J 1 .Ji 2.J;/: 2 < 2.J;/: and -1/2:5 x [x] -. 1/2 < 1/2, so that for any a 2: 1 we have 1 " I (In.'f. I) . " (:1: - [x J ] x- 1/2)/ dx < 1 " ( 2VX + 1) 1 . 2 _ 2 ' d,c I :c <  {" dx 2 J 1 x 3 / 2 == 3 ( 2  ) 2 vf(i < 3. Thus, th integr<il 1 == r' (In x - 1) X Jl ,2 (. [x] 1/2) lix SOLUTIONS 137 is absolutely conv(rgent. Kow, one form of the E:ulel'-MacLaurin summa,tion formula. aggerts that .if f(;c) lias a continuous derivative on [1, n], where n (> ]) is a positiv( integer, then "1 f. " f. " :L I(k) == -(fen) + 1(1)) + f(.,,) d;c + J'(x)(:J: - :I:] ,- 1/'2) d;". I.,I 2 .'1 . I Taking J(x) In :,./:,:, Wf' oh1.aill th k=l Ii: lll + n +!. "Li. -Ia.,,). -.- (:1: 211 2 .1 3,2 1/2) tLL: . (a;] Setting In 2 n n In/.' E(n)==:L k k=l " L and letting n --;. 00, we see tha.I lim n -+ oc , E( n) exists and has the value -I. rhus lim (E(2n) F,(n)) 'n--O!:, exists and has the vahle O. N",xt, we have th(' following 2" ". l) T.1n r LJ-- r=2 r In 2 In 3 In 4 In 2n 2 - :C +4 .. . + 2n ( In , 21n4 In2n ) ( In2 In:) In2n ) ]+ 2+"'+ 2+ 3 +...+ (In2+1n1) (In2+ln2) 2+1n1l) n Ink + +...+ n --:L. 1. 2 k=] ( 1 J ) n.lnk 2-n In/.; In2 1+. +...+-:- +:L--:L- 2 11 kd k k=l k In 2 (] +  + ... + ) + ( E(n) + In;  ) _ (F,(2n) + Ir / 2 2n ) 
138 .--- --- SOLUTIO;'IJS SOLUTIONS 139 ( 11 :::: In 2 1 + ? + '. +  n ' ) In:.! 2 Inn - 2 + (1';(1'1.) - E(2n)) . n-I . .' I: \j vf 16A:3 + 24A2 + 91.' + 1- \mk 3 + 24k2 + 9k k=O Lettill n =, a.nd rem(m bering tbat limo ( 'f.! - In It ) = l' 11.-+<"., .A''''I k . n-1 )(Vk+1 - Vk) = ..;n. "--<I k=O lIence we may take II, H1, () 24, a.nd c 9. wlWfe l' ::::: O.577:n is Euler's conjnt, WI' ohtail' 73. Ld,lI 1)(' a poil.ive Int"I';"1' a,nd a,b in1."1!,r-J'B such that , T /Ill 2:)-1) -;""l'In2 r=2 1 " In  ') 2 - , (,'(.'/)(fI,l>,lI) = 1 Prove thaI there fxist illt,ef:',ers aI,b l with a] a (mod n), b1==b(modn), GCD(Il].bti I, 72. Determine constants a,b and c such that vii  (J;;k 3+-b k +] - -laP + "J.2 + C" . k=O . Solution: \Ve ('hoose (/.1 to be any nonz(!ro integer such that for n :::: 1,2,. .. . U:U) 0,1 == tl (mod n) . Then we set. Solution: J;or k 0,1,. ,. , 1.ve have ( Vk + ( vi;r (k + ) )Vk+l- :!(k + I)Vk + ; hlk -t 1 - k.Jf (4" + 1 )vk+l- (4A + 3)v'1.: f(4 k + -i Rk+l) ';( 4k + 3)2k Vi6k3-+24'k2 + 9k + 1 - -I 16k 3 + 24k 2 + 9k b] b + 1'1'1. , when' r is the product of those primes which divide a] but which do not divide either born. If there a.re no such primps then l' I. Clea,rly we have b] = Ii (mod n) \Ve now show that GCJ)(a1,b]) = ] . Suppose tha,t GCD( 0,] , b]) > I, Then there exisls a prime q which divides both III and b l . We consider three cases according a.s so tlJ a.t {j V I 6P -I 24i: +9k +  =- + 4 Vk + ) ._ v'k , and thu:> (i) q divides b, (ii) q docs not divide b but divides n, (Iii) q divide;; neither b nor n. 
140 SOLUTIONS Case (i): As q I b, q I b 1 and b 1 - b = m, we have q I Tn. Now, by (73.1), GCD(aI,b,n) GCD(a,b.n) = 1 . Sine" q I 0,1 and q I b w£' "ee that q does not divide n, Thus we have q I r, contradictinl!: the definition of T. Case (ii): This case clearly cannot occur as b 1 = b + rn, yet q divides both b l and n, hut does not diYit!e b. Case (iii): As q I 0,1 but does not divide born. we haW' q i T. Sinn.. q I b 1 , q I rand b l = b + 1''1) , we musl. haw' t] I b, which is impossi hl£'. This completes the solution. 74. For 11. = 1,2,... let s( rt) denote the sum of the digits of 2 n , Thw;, for example, as 2 8 = 256 we have 8(8) 2+5 +6 13. Determine an positive integers 11. such that (74.0) s( n) = s( n + 1) . Solution: 'Write 2 n = 0,11110 111 + a",_I 1Om - 1 +... + 0,110 + ao , where 0,0, 0,1, . . . , am are integers such that 1 ::; am ::; 9; 0 S ak ::; 9, 0::; k S mI. then 2 n a m +a m -l+..,+al+aO s(n) (mod 3) , and so sIn + 1) == 2n+l == 2.2 n 2 s(n) (mod 3) , Hence, if 8( n + 1) s( n), we must have 8(n) 0 (mod 3), 2 n == 0 (mod 3) , SOLUTIONS 141 which is impossible. Thus there are no positive integers satisfying (74.0), 75. Evaluate the snm of the infinite serit,s Solution: \Ve ha,ve "" 1 L m(m+n ) 7n.n=1 On the other hand, we have 00 1 L m11.(m + 11) m,n=l :1U1 00 1 mn(m+11.) . s L "",1'1:;::,] d(:L1(m,r101::;;;1 t !..-11 xm+n-l dx rn,n=l rnn 0 1 ( 00 x m ) ( ':><) x n ) dx 1 L  L-;- x o 111=1 n;::;1 rr In 2 (1 x) dx J o x 1 00 u2e-" du o (1 e-") {"'" u 2 f e- nu du Jo n=1 = f 1= u 2 e- n " d'u 71.=1 co 2 L n3 n=1 00 1 2 L n 3 ' n=1 (x = 1 - e- U ) 00 L d=1 00 L 1 rnn(m, + 11.) 1H,n=1 GClD(m,n)=d 
142 SOLUTIONS 'x' 00 L 1 ri3qr(q+ rj L d=l q,r=] GCIJ(g,T)=1 (E :3) 'oX L I qr( q + r) '1,"=1 Or' 0("",)=, ( 'X I ) S L;j:, <1=1 so that 8 2. 76. A cross-country racer ruIIS a 10-mile race in 50 minutes. Prove that somewhere alon/!; the COl1rse the racer ran 2 miles in exactly 10 rnimltes. Solution: For 0 « x :::; Slet T(:) denote the \,ime (in rnimltes) taken by the , , ". raer o 1'Iln )etween points x and J' + 2 mileii along the course. Ihe functIun 1 (x) 1M COIltmuol1s on [0,8] and has the property (7(>.1) T(O) + T(2) + T(4) + T(6) + T(S) = 50 . 1'h<, equation (76.1) shows that not all ofth<, values T(0),T(2), T(4), T(6) and !(8) ar('> greater than 10 nor a.re aU of them less than W. Hence, there exist mteg<'rs rand s with 0:::; r,.s :::; 8 such that T(r) s: 10:::: T(..,). Then, by he int.erml.'diate value t.heorem, there exists a value y, r s: ?! s: oS , snell t.hat T(y) = 10, and this proves the assertion. 77. Let AB be a line segment with midpoint O. Let R be a point OIl AB between Ii and O. Three semicircles are constructed 011 the same :;ide of AB as Iollows: 8 1 is the semicircle with centre 0 and ra.diuslOA I = l OB'. 8 . tl . . I . h " 2 IS ,Ie Sl.'tIJJclrc e WIt centre R and radius I AR I meetin g R B at e. S ' . tl , , . < -',' 3 IS . lC SOLUTIONS 143 semicircle witll centre S (tlw midpoint of CH) and radius IC81 = 18BI. The common tangent to ''''2 and 8 3 touches 8') at P and SJ at Q. The PNpendicular to AB through C meets HI at D, Prove that PCQD is a rectangle. Solution: \V" give a <;olution using coordinate geometry. The coordinat(- svstPTTl is dl<,'';''fI so 1.ha,t ,Ie ( 1,0), 0 -c: (0,0), n (1.0) Th<,n w(' h.ave It := ( -Il, U), w IWf(' 0 <. It <. I, ami hClln- C' = (1- 2a,0), S -= (I. - tt,O) Tile <,quatiolls of the t.hreE! >'t)mjdrc)(.;; arE' givE.'T! as IoJlows: !h ;r2 + !J2 = 1 5'2 (3: + a)'l + y2 = (1- a? 8:\ (:r+a__l)2+.y2=a'l The perp('uJicula.r to .41J through C meets 5'1 at D (1 2a,2 Va - a 2 ). The equation of the common tangent to 8z and 8: 3 is ;dl- 2 a)+2 y Ja-a 2 = 1-2(1.+2(12, and this line touches !h at the point P (2a? - 4a + 1,2(1 a) a 2 ) and 5'3 at tlw point Q (1- 2a?,2a va - tP ). The slope of P D is /--..,  2a\i Q. - a.-:= - l -- _ r ( ' I - . - 20. - 2a 2 ,Ii 
144 SOLUTIONS and the slope of PC if; 2(1 -, (l) 2a 2 20, _ (I a . V - a The product oflhese slopes is -1, slJOwin that PC ;}.nd l'D iU!:' perpeudic111a.r, that is IC}'!) = !.I(I". Similarl.\', L P IJq = L nq( , == I <J( . I J = !)()" , :'0 thaI. P LJ(J{' i: a H'<'tan,d". 78. Determine the inverst" of the 11 X 11 matrix (71<.0) S[;: 1] where n;?: 2. Solution: Sel., I[r:: f] u[j:: I] so that S = {j - I, U 2 = nU . For any real number e, we have (U - I)(cU l) ('.[;2 (c + 1)F + I (en. (c + 1»[' + 1. SOLUTIONS 145 Thus, if we choose en - (('. + 1) == 0, that is c 1/( n - I), we have S-l = (TJ I)-I =F 'f! - 1 I r Z-n X__l n---l ---1-. _ /1-1 1 1 L-l 2-n n::. 1 ",] 1 n.l fi-. n- 79. l':vaJHa,te the sum (79.0) n-] S " "( l) k. n f I.. / ) ,(11.) == L.,\- 10S .1'11" n , k=O wh'l'e fI. is a positive integer. Solution: Set w exp(1I"i/n) so that S(n) I:(-nk ( w k .)w-k ) T1 k=U .. Hcncf', by the binomial theorem, Wt" obtain 8(1/.) I " I: ,-l kn I: " ( n ) . k(n-ZI) -- W \,01..' 2 n 1 k=O 1=0 " ( ) n-]  I: 1/. I: w k (2Ti--Z!) 2 1=0 1 k=O 2 ((  ) 11 + (:) n) , that is S(n) = 11/2'1'1-1. 
146 SOLUTIONS SOLUTIONS 147 (80.0) 80. Determine 2 X 2 matric<:s Band C with integral elltri0S such that Solution: Let tbe two triangles be IIBC and DEF. We suppose that [ -1 ] ] o -2 B 3 +C 3 . IABI IDEI a, 1,1C 1 b, I/fCj=c, h, IDFI = c, [J>'FI = tl, awl that (HLl) (l < b . Solution: Let so that and thus giving lIenee, we have and so .1 [ 1 ., :: J 1 () ,\ j\",t/j(' a1l(] 1\/)1"1" ar" Rimila.r, w.' havt' a b t.' A 2 [ 1 ".3 ] o 4 ' b d ' c RO that /1.2 + :lA + 21 = 0 , (iiI.2) c = l? I a, Ii = b:11 a 2 . F'rom (B1.1)we ha\'e A 3 + 3A 2 +2A O. (81.:1) 1 < bin, (A + I)J A 3 + 3A + 311 + I = .4 + I , a.nd from (81.2) and the inequality c < a + b we have b 2 <a+b a A (A + 1)3 - 1 , so that and we may t.ake B /1+1 [ _], (:=-1=[ - -n.  <' 1 +  R; 1.618 . (g]..t) .,' 2 To satisfy (81.;1) and (81.4) we choose bla :3/2. say n = 2t and b = 3t. Then, by (81.2), we have 9t c = "2' d == 27t . 4 81. find two llon-conf,Tuent similar triangles with sides of integral length baving the lengtbs of two sides of olle triangle equal 10 the lengths of two sides of the other. To ensure that c a.nd d a,re integers we choose t = .J so that (I 8, b = ]2, c ]8, d 27. 
148 SOLCTIONS The triangles with sides 8,12, 18 and 12, HI, 27 respectively, meet the require- ments of the problem. 82. Let a, b, c be three real numbers with a < b < c. Th fmlctiOll f(x) is mntinuous on [a,c and differentiable on (a, c). The derivative f'(x) is strictly inereasing on ( a., c). Prove that (82.0) (c.. b)f(a) + (b - o)f(c) > (e  a)f(IJ) , Solution: By the mean-value th",orem there exist.s a, real number 'u such that f(b) - f(a) = f ' ( 'I1, ), b b-cL ,a<'I1,<, and a real number v such that f(c)-f(b) ' ( b = f v), c b<v<c. As a < 'Ii < V < (' and f' is increasing on (a, c), we have f'(-lL) < f'(v), and so f(b) f(a) < fee) - f(b) b-(! c b Rearra.nging this inequality gives (82,0). 83. The sequen.ce {am i m = 1,2,...} is such that am > a m +1 > O,m = 1,2,..., and am converges. Prove that <X> L m(a". a m +1) m=l SOLUTIONS 149 converes and determine its SI11ll. L t l) . \ . \"" a . is a conver g ent serit's of positive terms, C f > ..1 S /nt:=::;:;l Tn '_ t.here ,-,xists it positiv., integer N(c) snch that ( , ] 0< tl m + l + Um+Z + .., < (./3 , g:,. ) for a.JJ 111 2: IV( d. Let 11 2: :L!\ (d+ L If'11 is even, say 11 = 21.', whpre f.: > X( (), from P":!. II \\'t' h"vp Solution: ka2k < akH + ak+2 + . . . + a2k < C.n , so that 'WI" = 2ku2k < 2E/;J < ( . If n is odd, say 1) = 2k + 1, where k 2: N(i), from (8;J.1) we have kaZk+1 < a/,'+2 + (l,k+3 + . . . + aZk+1 < {/3 . so that. nUn .= 2kaZk+J + a2kH < 2E/3 + €/3 = ( . VV", have shown thai 0< na n < €, for all n 2: 2N(E) + 1, and thus lim na n = ° . )}.--4OC' Next., set n Sn = L k( Gok - ak+1), n 1,2, , .. . k=J vVe hay.; S" n n L kak L kak+l k=1 k=l n 71.+1 L kUk - 2)k l)ak k=1 k=l 
150 SOLUTIONS n (k - (k l))ak na n +! :=1 n  (/k na1/. T l' k=l Letting n ---, 'JC, we ee that lim,,,,:, Sn exi;,ts, anti hag the valu" Y:kl H;,., aR )0,<> nanH ,,10,: ((11 + l)ft,,+1 - (l.n+d () - 0 () . llepce, > I k(ak - Ilk+l) COli verges, aud its sum is ) ::;::;;01 Uk- 84. The continued fraction of ,(D, where D is all odd nOl1squal'e imeger > 5, has a period of length one. 'What is the length of tile period of the continued fraction of Ml + ..Ji5}? Solution: The continued fraction of yf[j is of the form ViJ' [a; b] , where a and b are positive intpgers, so that - (I 1 b I + 1 b+- b+... 1 b+..Ji5 a, giving D + a 2 - (lb 1 2a As D is not a square, ,jjj is irrational, and we mllst have b 2(1, D::: (lz + 1 _ SOLUTIONS 151 I-'urthcrmore, a. D is odd and greater than , we have a ::: 2c. c 2: 2 and D 4c 2 + 1. It is easy to check that [ (L+ !) , ] [( '""I .1] [ (u,!';) - I ] [ (-'1"'ij (2'-') ] [ L :t 2 V1J ] - I + yf[j  :!,. ,\ [ I + In ] 21" ::: [ 3.C 1 + J iJ ] 2 ::: [ 2C - 1 + -Ji5 ] 'lc 1 , == c .. 1 , I. 2c - 1 . so tha.t the nmtinueJ fraction of (1 +..[i5) is as 2(: I > ;, and its period is of length 3. 85. (85,0) Let G be a group which has the following two propertie5: (i) G has no elem<,nt. of order 2, (ii) (,,'y)2 (yx}2, for all :C,y E G. Prove that G if' ahdian. Solution: , , . For x,y E G we have ,,:2 11 ::: ((:I:y-l}y)2 y (y( xy-I »)2y (by (8!J.0)(ii) (1/:r.,Ij-l)(Y3:y-l)y, 
152 SOLUTIONS SOLUTIONS 153 that is for all i = 1,2,. . . ,n. Prove tha.t 0  det A  1. (85.1) x 2 y = yx 2 :1:.- 1 J,-1. 1 ; :r(};-1)2y- I x ;q,-l(.r,-I )2x, Solution: tet A denote 0111' of the eig'?nvahws of ct and It,1; ;!2 (Ie Q) h. an eigellvector of A r.orr<;spolHling to >., 50 that K'xt, we have (hy (Sfi.1)) (86.1) .43;= ),;r . that is SiC1- :I, 0= (:II: _.. ,.1:"f "lid choose i, 1 <:: .; < 1/., o I hat (R!).2 ) ,/,' ly-1:r:::: ;1;.tJ- l. r -1 "rtl IHax 1:/'11 f 0, l::O;.1:5 n . Similarly, we have (85.3) y- 1 x- 1 y y:r.- 1 y-1. From the i-th row of (8tU) , we obtain (p,:r.- 1 y-1 yl :::: xy( x-1y-1x )y;c-1y 1 xy(xy-1 x -1 )yx- 1 y--I (by (85.2)) xyx(y- 1 x- 1 Y)X- I y-l xyx(y:j;-ly-1 )x- 1 y-1 (by (85.;1)) (xy)2(X-1 y-1)2 (xy ?(y:t' )-2 (yx )2(yx )-2 (by (85.0)(ii» n 2.::: ai33:j :::: AXi , j==1 Then we obtain so tha.t n (>' - l)xi 2.::: aijX 3 , 3=1 #i a.nd thus :::: 1, '" I>' 111 x il 12.::: aijxj\ j=l #i and thus, as G has no elements of ordcr 2, we have xy.r:- 1 y-1 = 1 , that is ;!:y = yx, proving that G is a.beJian. n <:: 2.::: \a;. j II): 31 j=l #i 86. Let /1 :::: [aij] be an n X n real symmetric matrix whose entries satisfy n S, 1;l:i l 2.::: !aij! .io=l j#i < !.r.;!, (81:5.0) n aii = 1, 2.::: la;jl  2 , j==l showing that (86.2) \>. - 11  1 . 
154 SOLUTIONS Since A is a, real symmetric tnatrix, ). is rea] ami from (86.2) we see that (86.3) 0:SAS;2 Lei. /\1,"" An dt'liote the 'Ii eigenvalues of it. I'a.ch Aj is nonntga.tive by (81).3). Thu& we have o <: dt't il '\1'\2'" An ( I _R -( AI + ).'2 + ... + An) ) .n / ( trace A ) " ( 1 ) .. -n \n -- I < 87. Let R be a finite ring containing an dement T which is not a divisor oJ zero. Prove that R must haV( a multiplicative identity. Solution: As II is a finite ring there exist integers 111 and 'Ii such that (R7.1) 1'''' Tn, I S; Tn < n We wish to show that (87.2) l' 1'1: , for some integer A 2:: 2. If m 1 we may takt) k we have 1'(r m - 1 _ rn-l) O. As l' is not a divisor of zero, We must have n. If m 2:: 2, from (87.1), (i.:) 1'",-1 1' n - 1 = 0 . If m - 2 we may tali:e k 'Ii - 1 (2:: 2). If 'In 2:: 3, f.rom (87.3) \W have r{r m - 2 _ 1'''-2) = 0 . SOLUTIONS 155 As l' is not a divisor of zero, we must have 1' m - 2 n.-2 l' o. If m ,,_ 3 we may titke k -c /l. . - 2 (:> 2). Continuing in this wa. y .' we S('(' that (87.2) holds with k It - m+ 1 (2: 2). For any x E R. we have from (R7.2) :1'1'=,0.1. and 50 (.1' .I'l'l. 1)1' 0_ Ali r is not a, divisor of zero, we SE'€ that :1: = :1.'1'/;-1 . (87.4) Similarly, we have (87.5) x r.l"- 1 x. From (87.4) and (87.5) we selJ tha,t 1'k-l is a multiplicative identity for R. 88. Set J" {I, 2,... , n}. For eadl non-empty subset S of .In define It'( S) = max S - mill S . . BES BES Determine the averi1ge of wl.S) over a.1l non-empty subsets 5' of I n . Solution: I'or 1 < k S; 1 S; n let S( k, l) d()f\ote the set of' subsets of Jr. with miTIS BE,'; k, maxS sES 1. We have, for aU S E 5'(k,l), w(S)=l-k, and { I, ifk t, IS(k,l)1 = 1-k-1, if k < l, 'II' illl 
156 Then we hav(' L.: toeS) ','cfs.Jn SOLUTIONS L.: L.: w( S) 1 </"<1<,,, 8(;.'I(k,l) L (1- k)IS(k,l)l 1 :$k<I:5" (l I.: 1'21-;:- 1 I:$/"<::l> 11-1 11.-1 z=h"l k=1 " I L l:i 1=/;+1 n '\ L..,i :: 1 k=1 l=k+1 1"t-l L.: k=1 ('17 - 1 )2 n +1 - (k _1)2 k + 1 ) n-I L k2- k - 1 (2 n +1 - 2k+1) k=1 'fl.-I (n 1)2 11 L.: T k k=1 n-l L.:(k - 1) k=1 ('17 n-1 n-1 -2" L.: k-r k + L.: k k=1 k=1 1)2" ( 1 -  ) , 211.-1 ...2" ( 2 _ ( '17 + 1» ) + n - 1 2 n - 1 l):t' 2('17 - 1)- 211.+] + 2('17 + 1) + '17 .... 1 3)2" + (n +{), = (n = (n so that the required average is ('17 n 1,2,.... 89. Prove that die number of odd binomial coefficients in each row SOLUTIONS 157 of Pasca.l'" triangle is a. POWt'l' of 2. Solution: The ellt.ries in t.he n-th row of Pascal's triangle iln the mdtidcnts of tlit' IH)Wf'I"S of x in 1.11(' expansion of (1 + 3)". vVe write n in binary notation p.m. I ) '1/ 'c 2'" + n2 ..J . _ . + :J"'" . (9.2) W]IC1'£ 0.1,' .., f1.k arc intcgers such thai III :> (l2 :> ". :> (lk ::: [) . Now (1+;rf (I + :1:)'1 (1 + 3f - I + 2x + :1: 2 _ (:I + 3;2)2 (1 + :'1;4)2 == 1 +x 2 == 1 + :,/;" == 1+:1: 8 (mod 2), (mod 2;, (mod 21, and so generally for cl.ny nonnegative integer a we have Thus, we have (1 + ;/;)'" (I + .r. fl" == 1 + x 2 " (mod 2) , (1 + x)2"1+2"2+...+2'" (1 + x)2"1 (1 + x)2"2 ." (1 + x)2"k (1 + X 2 "1 )(1 + x2"2)... (1 + X2"k) (mod 2) 1 + (X2"1 + X2"2 + . . . + :t 2 "k) ( 2"1 +2"2 + 2"k-1 + 2 0 k ) + x +... x +... +:,/:2"1 +2"2+...+2"k (mod 2) , and the llumlJer of odd weffidents is 1 + k + () +... + G:) 2 k . 
158 SOLUTIONS 90. From tbe n x n array [ (n 1 2 3 'II n+l n+2 n+3 2n 2n";'" 1 2'11 +2 271 -L;3 3/1 I)n + 1 (n. I)n +2 (11 1)11 +;1 ') n- ". number :1:1 is sek.ctcd. The row and column cont.ainhll!, ;1:1 <'In' t.hen delet.ed. From tb., resulthg array a numtJP1' .1'2 i, s('lecjp,L and it,., 1'0\,' ilild colu1Il1l deleted a>; before. The :=:election is continued until only one number :e'l remains available for selection. Determine the sum Xl + :r2 -I- .. . .j... X n ' Solution: Suppose that :r., 1 ::; i ::; n . belongs to the t'i-th row and the -5i-th column of the array, Then Xi = (1'; .- 1)n 'S;, 1::; i ::; n , and so n LXi ;=1 n nLt'i ;0=1 n n 2 + L S; .i=l ;'oJow {1'1"", 1'n} and {Sl,.... .s,,} a.re permutations of {l, 2, . .., n} and so n n n (, J) L"'i=Ls;=Li= nn ;=1 io=l i=1 2 Thus n LXi ;=1 2 , n(n+ 1) n(n 2 -!..1) -n _._= 2 2 91. Suppose that p X's and q O's are placed on the circumference of a circle. The number of OCCIHrences of two adjacent X's is t1. and the number of occurn>nces of two adjar.ent O'sis b, Determine n b in terms of p and q. 'I :1 ,j SOLUTIO:;\J'S 159 Solution: Let N x : e , N xo , N ox , N oo denote the llnwber of oc.curreTlct's of XX. XO, OX, 00, respectively. 'fht'u dearly we have { N"x = a , lv'"" = b, T . r __. \x,. + ;\ ,,'>. - P , _'V OII -!- N o . l ; -.::: fJ  so that 0, b IV:rx - ;"V oo (N x >; ,L Noo) (N oo + N",.) + (No' .. N;",,) p - q + (N ox - Nxo) , Finally, we show that N ox = N xo , which giv<:'$ the resuU a.-b=p-q. To see that N ox = N xo we consider the values of a function S ;<1> :ve .make one clockwise tour of the circumference of the circle, starting an.d nmshmg at the , 1. I . t ' al1 m e let C'_ 0 Then as we tour the CIrcle, tbe value of same potn. 111 1 y,.., , ,J - . ' ., S is changed as follows as we pass from ear.h X or 0 to tht, next. X or 0: new value of S old value of 5 1- €, where € { - , in going form 0 to X , . in going from X to X or 0 to 0 , , in going from X to 0 . Clearly the value of S at. the end of \,he tour is N ox N,.o. owever, S must be 0 at' the end as we have returned to the starting point. ThIS completes the proof of N o3 ; = N"", <Uld the solution. 
160 SOLUTIONS 92. In the triangular array 1 1 1 1 1 '2 3 2 1 1 ;) 6 -; 6 a 1 4 10 Hi 19 16 10 4 (92.0) ('VNY (,JltfY (exc('pt tilt' t(IP 1) is tll(' slim of thp ('Id,r.... t1 illlI1H'di".!,('jy all"v!' it, and the entrieli II and c immediately to the left ,wd right of 0.. Absenee of a.1I entry indicalcs zero, Prove that every row after the secon<1 row contaiJU; an entry which is even. Solution: The first eight rows of the triangulaf array taken modulo 2 are given in (92.1). 1 1 1 1 1 0 1 0 1 (92.1) 1 ] 0 1 0 1 1 1 0 0 0 1 0 0 0 1 1 1 1 0 1 1 1 0 1 1 1 1 0 1 0 0 0 1 0 0 0 1 0 1 1 1 0 1 1 0 1 1 1 0 1 1 0 1 1 The first four entries in the fourth row of (92.1) a.re 1 1 0 1, which aJ'e exactly the S<J.llle as the first four Dutries in the eighth row. Thus the pattern (92.2) 1 1 1 101 1 1 0 1 000 1 0 o repeats itself dow1l the left-hand edge of the array. As each row of (92.2) con/.ains at least one zero, every row from the fourth 011 down contains an 161 SOLUTIONS r r........ 1 ' h ' ]  , " ompl tes the P roof, as the third row contains an even even l1I1Inbcr. "  " number. . I ; 01 I' I nb ' I '" Xl r s<I,lis!ies A sequcnce 0 n rea. mn .... ' "..,"n ' 93. i\ { Xl 0:: 0, I \ + I 2..::i::;n. i J ; X._I r, . " t ' '0 "0. 1 numb,'f IktNmilll' a,lowN hound for tlw avr-rage wlwl'<' e IS o. pOST He 1(',1.1 - . f " .' .\::; a. runc.tioll nf (' only, o Xl,.....&Jn 4:. (93,0) Solution: I t be an'i . ' rea.l number such tha.t . ,e Xn+I IXn+ll:::: \x n +cl. Then, we have :1 n+l 2 L,.; ..f. i i=l n+l 71.+1 L Ix;l2 == L !Xi-l + cl 2 i=2 i=2 n+1 L(Xi-l + ci i=2 ,,+1 n+l L X;_l + 2c L Xi-I + C 2 tl. i=2 i=2 n LxI + 2c i=I n Xi +c 2 n, so that n L 2 O <: X 2 ' 1 == 2c Xi + c 7/. , .....2 '1l+ i=l and thus (a.s c> 0) 1 n C L X i;:::-2' n i=1 
162 SOLUTIONS 94. Pro\."<) tha.1, the polynomial (9/LO) f(:I: I = ,1''' I :/:,1 -t 3: 2 .j :1: I- 5 is iITNludblt, over Z If,r n » .1. Solution: Suppw,(' f('1:) i., rl'rhldbl0 ov(']' Z T/1"1I (/1pl'l' t'xij llIonic poly- nomials f,(.r) alld h(.r) witb ;uJ"'r:.ral c()eflicj('trl, ,-,lIdl I ha', UI'LI) /, r) !J!'./';!I(:J"), dt').'; r, >' dP1', It Thus, we havE:' .') f(O) [1(0)11.(0), and, a.s 9(0), h(O) a1'(, illteg('rs <.Iud ,1 is primc, we lrav!) witJlOu1.ln,;fi ot generality 9(0) :1:1, h(O) H,. Lt ,. g(x) III:r- ii,i) _';=1 be the facf.orization or g(:/:.) over C. TlI';n, \'IrE; JJ,w(' ,. 1 19(0)1 .. II IfJ.i1 , j=] and o at least one of the 1;1 j l is less than oj' equal to 1, say /;31/  1, 1  1  r , Henc(' /.f(;ll Ii /,6 1 '/3j! + fir I ill I !')i > !) - IfJLI -- liW rfJ/ 1 3 d" :::,51]11 1 , SOLUTIONS I !II whidl t:ontr;1dicts , 163 Thb prows tlli.t /(.1') is irreducible over Z. f«(3z) g«(3/ )h(tl[) 0 hUJz) 0 . . 1 I I )Nt>rrn;ll(' 1;11<' 9 r:: I ' I (/ hp 11 ( > ,'1 \ dbt.inct r0<l !lI11Il )l'l'fi. .t). Jt- , at,..., -71 ,.._ I II 1 .' f 1 ' 11( ' ""RtA {11 of '11 2 lilH'ilr e(jlJatiolls r;c'lwra so 11 Ion 0 , . ",' ," (9,5.0) { . :"1 ..I '/'2 1"" -I- ,/'" a , IX] l rL2J:2+ ...., (t".X n a 2 . c _ , ,,2' Cl ' I ... 1- a2xn I'] "2" ". ' I ' 1'1.-3 + n-:I, a-' ;.{:] 1- (t2 ;C2 + . , . a" wn = 0, = 0, 0, = 0, ill the 11. unknowns XI,. .. ,In. Solution: Set For k 0,1,... ,n (95.1) f(x) = (;/;- al)(:r a',!)...(x-u n ). 1 the pa:rt.ia.l fraction expansion of ;r k If( x) is Xk f(a:) n ?= ;r $=1 (Ii Multiplying both sides of (95.1) by f(x), and equating coefficients of xn-l, we obtain (95.2) This shows that n a L: - f ' ( ' ) i=l '1, { 0 , k 1 ,k 0,1,...,11. - 2 , n-1 if = (f/(l) "'" f'(n) ) 
16.1 SOLCTIONS SOLUTIONS and 96. Evaluate the sum ( 0,1 an ) :/; == jl((ld "'" f'(a n ) N 2,3,... L H( r'lT) == mn are two solutions of (fl!!.O). Thl'se twn solutions a.re lint:'ady in <leJl!"lIdt'u I ; 1i:11' otherwise tht'n wouJd pxjt I'''al 11111111)(']'<; ," and i ( not both zero) such thai ji1t<n'SN nt+n>N IX' D(m ,>I )=1 -"11. + tp == (0. ...,0) . that if< (95.3) s + Ill,; == 0, i == 1,2,. . . , n . Solution: For iV :>. :1 we ha.v£' If t == 0 theIl from (95.;H we havi' B 0, which i:; a cont.radict.ion. Thus, I 1: (I and (95.3) giw;; L 1 H(.'V ) L + rnn m.1l 1m<n:$N.-1 l1tJ.<n=N rn+'1>N 1n+n>N GCV(rn,n)=1 GCV(m,n)=1 1 L 1 L r1/.n mn 111t<nN-l ln.<niV--] m+n>"'--1 m+n=.;\' GCD(m,,,)=1 GCD(m,n)=l  i==1,2,...,n, (1; -1:' which c011tradict.s the fad that. th! a; aI'e dislincL TllU tILe solutions l a.nd ]1. are linearly ind'pendent.. :'{(1xt, as the ai art" distinct, the Vandermonde determinant 1 1 1 l!l °2 {/,n-2 a? 11. 2 a_2 2 n-:1 n-3 (L= I tt 1 11. 2 I does 1101. vanish, and so the rank of the coefficient. matrix of (95.0) is 1/. - 2. Thus all solution,; of (!J5.0) are given as linear cornbinatiomi of a.ny two linearly independent solutions. Hence an solutions of (95.0) are given hy H( N - ]) L 1:$m<N/2 GCD(m,N)=1 1 L 1 N m, l:$m<N/ GCD(m,N)=l ] L + N m l:$rn<N GCD(m,N)=l S(N - 1) (Xl,...,Xn) t).JJ.. + ill ( Q + (3(1'1 Q + !3 an ) -f'(al) ,...,- f'(a) for real numbers n and (i. 165 + L m.N l'in<X GCD(rn,N)=l 1 L 1 + N m l:$m<N GCD(:r.,N)=1 ] L Ne-m N 1:$rn<N/2 GCD(m,N )=1 
5'(A' -- 1) 1 '\' 1 1 L 1 -\ 1V L-o m N m l':S:m<N 1::;'111<.N 'iCD{m,N)=l aCf)(m)\')=l Ccc S l IV - 1) , remembering that GC IJ(.N 11, N) > 1 fOJ' even ;V (2: <1). Thus, we havc 8(1\') S(A -- I: - SU\ J- 8(1)"'1/2. 97. FV<lJna,te the limit (97.0) 1 nit. L = lim '\. ''\'  "_ex, n  L-o J '2 .L 1.:2 . J=1k=1 Solution: Pa.rtition the unit squa.re [0, 1J x [0,1; into ,,2 suhsqlJ<l,r"s by tIt(' partit.ion points {Uln,k/n): 0  ),1,;  n}. Then a IU"ma,un sum of the functiorL x/(;r.2 .\. y2) for this partition is \.' jln 1 L..- Uln' ) 2 + (k/n}2 n 2 1t,"" ) "" :-- 2 1'" nL..- J + 1\'- 15:.lt k 5Jt a.n d al so ',' jln 1 11m L..- ( ,/'2 ("/F 2 n. '00. J n J' -\ : n 'it lJ,k:5n it. x 'd:t'd!J, , [O,Jlx [0,11 ;<2 .1. y- sO tha1 (9..0) becomes !o l !o l :r L = -.' dJ: d'l} , 0 ,0 X 2 .1. y2 . SOLUTIO:;\lS 167 t'!4 1 "ce cos () dl' dO .1. 2 l : ce COB e dr- de .-- 18=0 '1"=0 Jli_'Jr/4 T-O /4 1 /2 [ nO -\- cot 0 ,10 in IT/I To /-1.1 [In sin 0 ]: r./I -In(l/Ji, that is r .= 7[/1 + (1112)/2. 98. Prove th<l.t (98.0) :.lr. . 2r. / - tan .L 4 SUI Ji = " 11. . Solution: :For col1vetlien('c we let 7' = 'ir /11, a.nd set r; =: co,:;p, . = sin]'. . pi I ( + is ) l1 - .-1 that is Th.en, we ha.ve c -1- zs = co an( so (' -, 8 'I' 3 'J O 74 I If ' 2 / j s"i 11 1 ' 1 JO'  5 c.\1 02 - 165e s'  -\- ," e s ," I (. e + e S -. ;:h - :;, c _. 10 11 . 1 _ 1 " 2 fi (; 3 'J O 4"7 i + 165e3 .8 + 55r;2 .\). 11eo5 05  -. -«,) (' S " .. .. Equa.ting imaginary parts, we obtain " :; 6 5 'i30 4 7 .1 55(;2 S9 _ 05 11 0 . (fiR-I) 11e 1O s 165e s + 462c s' -. r;. . .From (mU), as . i 0, we have . 64 . 0 46 -28 JO 0 ( , 11c1O-165e8,2-\-462e's .-33e. +0;)('05 . f!8.k) ::-\I:xt, as (fl8.3) C2 1 .2, the equation (98.2) becom!'s 11 22005 2 + 123205 4 - 2816s 6 .1- 281605 8 ,- 102405 10 = 0 , (%.4) and thus ; 
168 SOLUTIONS SOLUTIONS 169 (lIs - 41.q3 - 32.'<;) 11r. 2 (1 4.q2)2 121.q2 - 968,q4 ./- 2640i; 2816,g8 + 1024,g1O -.11(1 $2)(1 8,q2+,g4) -11 + 220,q2 1232.q.l + 216,g6 2816,<;8 + 102.1s 1O () , Evaluate the sum 00 c s 1: n(n'/- 1) . n=1 by (98..1). This proves that . I. ,pt 1.: he it ! wsitive integ£'L We have SolutIon: (mu)) I J" ,I.\.g:' - ;t.4" = -L/II ('( I. k \1::.., ( ':.!>  '/I n=-I k L Cn _0' n:] n A,+l . L (.n -] n=2 n '/1('/1\ 1 n=l ('/I. \ II 1.1) Xext, we have tall 3p -I- 4 sin 2p + 8sinp cosp A' ( ) C "'\ c? :, C. 1l ---1 _  Cl+L...-- n k+1 n=2 that is, using (98.3), k 1. Ck 1/-"--- . L...- n 2 k + J n::;:::;;:l ( 98.6) tan 3p + 1 sin 2p k 1 = L -1:'+1. n=l  ( cA-lnk) L...- n 2 k + 1 n=l Ink k+1. Then, from (98.5) and (98.!i), we obt(1,in ta;n 3p + 4 sin 2p ::!:J1i. 311" . 21T r;-:; tan + 4 SID = v 11 , 11 ] 1 Letting n 00, and using tIw fact that lim (Ck --In k) k-+oo . As tan 3p > 0, sin 2p > 0, we must have exists, and al,,;o Ink lim - =0, k-+= k + 1 a s required. 99. For n 1,2, _ _ . let we lind that 00 1 1T 2 S=L n 2=fi' n=1 C n 1. 1 1. 1.+ +-+"'+-. 2 3 n 
170 100. SOLUTIONS For x > 1 determine the sum of the infinite series x   +- + ;1' + 1 (:1' + 1)(;1;2 + 1) Solution: For 11. it positive integer, BPI, S"Jx) so that .1: :C 2 :l2J"/ ;:-+1 + ( .1' + 1 :I( .l';; + 11 . . +...j C -" ..f « , 1)(.:1':' + 1)... (:1'"" + 1) .1' 1 X x 2 +--+""'+ - 1 :1: 4 - 1 ( - . -- -  ) + x - 1 x 2 - 1 1 +... + Thus, as x > 1, we have x - I x 2n +1 - 1 . giving I " Sn(X) 1 Ull- n-.oo x-I x 1 :it! :r. -. 1 1 - x 4  1 ) I :r. x 2 x + 1 +(x + 1)(x 2 + 1) +... = J1:, 8,,(.1:) = 1 .  , Problem THE SOURCES 01: Gauss, see Werke, Vol 2, Gottingen (1876), pp.11-45, showed that ( E=.!. ) 2 ,,:Tl + 1..)"2 + . . . + W"(p-l)/2 = (-1 + i 2 -jP)f2 , ( E=.!. ) 2 w''' + W1!2 + ... + w1I-(p-l)/2 = (-1 - i 2 .,jP)f2 . 04: 05: 09: This result is implicit in the work of Gauss, ('(' Wet'ke, Vol '1., Gottingen (1876), p.292. The more general equation y2 x 3 + « 4b - lYI 4a l ), whcr<. (1, has no prime factors 3 (mod 4), is treated in L.J. Mordell, Diophantine Equations, Academic Press (1969), pp.238-239. This prohlem was suggested by Problem 97 of The Green Rook. It also appears as Problem E2115 in American Mathematical Monthly 75 (1968), p.897 with a solution by G.V. lcWi1lia,ffis in American 'Mathematical \1lonthly 76 (1969), p.828. 10: This problem is due to Professor Charles A. Nicol of the University of South Carolina. 11: Another soh]tion to this problem is given in Crux \ifathemat.icorum 14 (1988), pp.19-20. 14: The more general equation dV 2 - 2eVW dW 2 1 is t.reated in K. Hardy and K.S. Williams, On the solvability of the diophantine equation dV 2 - 2eVW - dW 2 = 1, Pacific Journal of Mathematics 124 (1986), pp.JL15-158. 17: This generaJizes the well-known result that the sequence 1,2,...,10 contains a pair of consecut.ive quadratic residues modulo a prime 2:': 11. The required pair can be taken to be one of (1,2),(4,5) or (9, 10). 19: Based on Theorem A of G.H. Hardy, Notes on .'?ome points in the integral calculus, Messenger of \1Iathematics ..18 (HJI I), pp.107-1l2. 
20: This identity can be found (eqn. (4.9» on pA7 of H.W. Gould,  "; Combinatorial Identities, Morgantown. W. Va. (1972). :1 The more general equation aIxI + ... + anx n k is trcated in Hua :j LooI<eng, Introduction to Ntlmber Theory, Springer-Verlag (1!J82), see Theorem 2.1, p.276. 21: 22: Finite sums of this type are discussed extensively in Chapter 1.5 of W.L. Ferra.r, IIightT Algebra, Oxford University Press (H):;O). 25: See Problem 2 on p.113 of W. Sierpinski, Eft'menta!"?! Thmrll of Number.'!, WaIsaw (196,1). Suggested by Problem A-3 of the Forty Seventh Annual William Lowell Putnam Mathematical Competition (December 1986). The discriminant of f(x"), k  2, is given in terms of the dif;. criminant of f(x) in R.L. Goodstein, The. discriminant of a certain polynomial, Mathematical Gazette 53 (1969), pp.60-61. H. Steinhaus, Zadanie 498, Matematyka 10 (1957), No.2, p.58 (Pol- ish). 26: 29: 30: 34: This problem was given as Problem 3 in Part H of the Seventh An- nual Carleton University Mathematics Competition (1979). 37: Based on a question in the ScholaIship and Entrance Examination in Mathematics for Colleges of Oxford University (1975). 38: Based on a question in the Scholarship and Entrance Examination in Mathematics for Colleges of Oxford University (1972). 39: Based on a question in the Scholarship and Entrance Examination in Mathematics for Colleges of Oxford University (1973). 40: Based on a question in the Scholarship and Entrance F,xamination in Mathematics for Colleges of Oxford University (1973). 172 41: 45: 47: 48: 49: 52: 53: 56: 59: 62: ... " 63: :t 64: This is a classical result, see for example H.S.M. ?o:,eter and S:L. Greitzer, Geometry Revisited, Mathematical Association of AmerIca (1967), pp.57, 60. " t d b T S Chu An g les with mtional tangents, America.n imgges e y .. , . MathematicallVlonthly 57 (1950), pp.407-408. S t d I y W Cross P Hilton, J. p,'ders"II, ICY. Yap, An alflo- uggcs ,e I . 1 , . . . .. M h' t' Maga- rithm fm' 1mdlil}lication in modlLlar arzthmetzc, i at (.ma ICS l . zinc !i!) (1!)Rf5), pp.Hi7-170. d S t 3 P 8 of Th Skolem Diophantischc Gleidmngen, Base on a z on . _ , Chelsea Publishing Co., New York (1950). d E I 1 . n D G Mead Integration, American Mathe- Base on ,xamp e. I .. _ . matical Monthly 68 (1961), pp.152-156. Suggested by 5.4.5 of L.C. Larson, Problem-Solving Thr'olLgh Pr'ob- lems, Sprinf!;er-Verlag (1983). See Problem 48 of Lewis Carroll's Pillow Proble.ms. See Problem 1 on p.13 ofW. Sierpinski, Elementary Theory of Num- bers, Warsaw (1964). See Problem 12 on p.368 of W. Sicrpinski, Elementary Theory of NlLmber's, Warsaw (1964). This problem was suggested by I)roblem A-4 of the r'irty Eighth Annual William Lowell Putnam Mathematical CompetItion (Decem- ber 1977). Th ' P oblcm was shown to us by I'rofes.<;ors David Richman and IS r . C r Michael Filaseta of the University of South aro ma. Th ' It ' d e to II J S S mith On the vallLp of a certain arith- IS resu IS u . . . , . al S metical determinant, Proceedings of the London Mathematic 0- ciety 7 (1876), pp.208-212. 173 
68: This is a well-known problem, see for example 4.4.4 in L.C. Larson, Problem-Solving Through Problems, Springer-Verlag (1983). 69: This problem was suggested by Problem 3 of Part A ofthe Fifteenth Annual Carleton University Mathematics Competition (1987). 70: Forms ax 2 +by2+cz 2 which represent every integer have been charac- j..rized by L.E. Dickson, The forms a,;r2+by2+t.::;2 which rqlresent all integers, Bulletin of the American I\.1atht'matic:tJ Society, 35 (1929), 1)p.fi i 'i-59. 86: Suggested by ideas of 97.5, Estimates of characteristic roots, in L. Mirsky, An Introduction to Linear Algebra, Oxford Universit.y Press (1972). I. .. A c:(()[ ED IN SCIENCE AND MATHEMATICS  l' In 75: 82: This problem was suggested by Problem 95 of The Green Book. Suggested by K.A. Hush, On an application of the mean value theo- rem, American Mathematical Monthly 62 (1955), pp.557-578. 89: This is a well-known problem. A generalization to the multinomial theorem is given by H.D. Ruderman in Problem 1255, Mathematics Magazine 61 (1988), pp.52-54. 94: Suggested by an example given in a talk by Professor Michael Fi- laseta at Carleton University, October 1987. 95: See Problem 2 on p.219 of W.L. Ferra:r, Higher Algebra, Oxford University Press (1950). 98: See Problem 29 on p.123 of E.W. Hobson, A Treatise on Plane and Advanced Trigonometry, Dover Publications, Inc. New York (1957). 174